0% found this document useful (0 votes)
57 views473 pages

Fin 320-00 Fall 2022 Online Package R PageN

This document provides information about an online finance course taught by Dr. Terrance Jalbert at the University of Hawaii at Hilo in the fall semester of 2022. It includes details about course materials, policies, evaluation, and topics to be covered. Students are expected to have access to specific TI calculators and course materials provided by the professor. Evaluation will include homework, attendance quizzes, a midterm exam, and a final exam. Course topics range from organizational forms and agency relationships to time value of money, bond and stock valuation, and capital budgeting.

Uploaded by

Elijah Carigon
Copyright
© © All Rights Reserved
We take content rights seriously. If you suspect this is your content, claim it here.
Available Formats
Download as PDF, TXT or read online on Scribd
0% found this document useful (0 votes)
57 views473 pages

Fin 320-00 Fall 2022 Online Package R PageN

This document provides information about an online finance course taught by Dr. Terrance Jalbert at the University of Hawaii at Hilo in the fall semester of 2022. It includes details about course materials, policies, evaluation, and topics to be covered. Students are expected to have access to specific TI calculators and course materials provided by the professor. Evaluation will include homework, attendance quizzes, a midterm exam, and a final exam. Course topics range from organizational forms and agency relationships to time value of money, bond and stock valuation, and capital budgeting.

Uploaded by

Elijah Carigon
Copyright
© © All Rights Reserved
We take content rights seriously. If you suspect this is your content, claim it here.
Available Formats
Download as PDF, TXT or read online on Scribd
You are on page 1/ 473

Finance 320-002

Online
Course Materials
Dr. Terrance Jalbert

Fall Semester 2022


rev. 07-11-2022

1
Exclusive Use Agreement

The materials provided in this package of materials are for your exclusive use only. You
may not re-distribute materials contained in this package in any form.

2
Course Syllabus

3
Course Syllabus
Principles of Finance
Fin 320-002 CRN: 10109
University of Hawaii at Hilo
Fall Semester 2022

Professor: Dr. Terrance Jalbert


Meeting Place: Virtually
Meeting Time: TR 3:30-4:45
Office Hours: 10:30AM -12:00PM and 12:45 PM to 1:45 PM on Tuesday & Thursday,
My office hours will be both in-person and virtually to the extent possible.
Office: COB-111 Phone Office: 932-7249 Home 959-8156 E-mail: [email protected]

Please feel free to e-mail or phone me at home. If I am not home, please leave a message
and I will return your call.
Zoom Codes

Virtual Office Hours


Office Hours 10:30-12:00 TR and 12:45-1:45 TR
Code: 944 8544 9173
Password: 126036

Class Time Codes


Finance 320-002: TR 3:30-4:45
Code: 963 8028 3133
Password: 501843

COVID 19 Semester Notes

Given the ever-changing nature of our current environment, I reserve the right to modify
this syllabus throughout the semester as I determine appropriate.

Class Etiquette

When attending Zoom classes, please turn on your camera. When attending
Zoom classes, clothing is required. Please wear appropriate attire. Appropriate
attire requires, at minimum, a shirt and shorts. Also, please be aware of what
might be in your surroundings and displaying on your camera.

On Site Section Safety Protocols

When in close proximity to me, such as visiting during my office hours, I request
that you were a mask regardless of university policy. If you are not able or
willing to wear a mask, I will be happy to arrange a Zoom meeting with you.

4
Catalog Description: Introduction to concepts and techniques of business finance.
Topics include organizational forms, agency relationships, financial analysis and
planning, the capital formation process and capital markets, risk and return, time value of
money, stock and bond valuation, and capital budgeting.
Prerequisite Courses: ACC 201 or ACC 250

Calendar
August 22, First Day of Class
August 30, Last Day to Register or Add a Class
September 5 (Monday) Labor Day Holiday
September 13: Last Day to withdraw from a course without a grade of W
October 31: Last day to withdraw from a course with a grade of W
November 8: (Tuesday) Election Day, No Class
November 11 (Friday) Veteran’s Day, No Class
November 24, (Thursday) Thanksgiving Day Holiday
December 8: Last Day of Regular Classes
December 8: Last day to withdraw from the university with grades of W

Midterm Exam: I will announce the midterm exam date as the course progresses. I will
give you at least two weeks notice prior to any exam.

Third Exam: Your third exam will be Thursday, December 15, 4:10 PM -6:10 PM: We
will use our regular class time zoom code for the exam.

Course Learning Outcomes:

A. Describe organizational forms available to firms, the characteristics of


organizational forms, corporate governance, the taxation of firms and how
dividend policy and capital structure can be used to affect firm taxation.
B. Understand agency relationships and how to manage the firm to minimize
agency problems.
C. Understand the relationships between financial statements, Forecast financial
statements into the future and determine future financing needs.
D. Assess the financial health of firms by examining their financial statements.
E. Calculate the returns and riskiness of returns for various assets. Describe the
relationship between risk and return. Describe how to use return and risk
information to evaluate investments.
F. Describe the capital formation process and sources of funds that are available
to firms, and to estimate the cost for various sources.
G. Ability to identify and solve a broad spectrum of time value of money
problems. Ability to utilize time value of money techniques in decision
making.
H. Describe the characteristics of various financial assets.
I. Describe the source of asset value and to be able to calculate values for
various financial assets.

5
J. Assess the value of potential investments in projects, and make correct
decisions on weather or not to proceed with the investment.

Course Content

A. Course Topics:
1. Organizational Forms, corporate governance, taxation and implications for
dividend policy and capital structure.
2. Agency relationships and solutions to agency problems
3. Financial Statements, Cash Flow, and Taxes
4. Financial Statement Analysis
5. The Financial Environment: Markets, Institutions and Interest Rates
6. Risk and Return
7. Time Value of Money
8. Bonds and their Valuation
9. Stocks and their Valuation
10. Cash Flow Estimation
11. The Basics of Capital Budgeting
12. The Cost of Capital
B. Additional Topics to be covered should time permit
13. Derivative Securities
14. Capital Structure
15. Dividend Policy

Course Materials:

Recommended: Fundamental of Financial Management by Brigham and Houston, 14th


Edition, ISBN: 13:978-1-285-86797-7

Required: One of two Texas Instruments Calculators (purchase one or the other, not
both)

Alternative 1: Texas Instruments BA-II+ Financial Calculator. It should cost


about $40. This calculator has all the functions we will use
in this class.

Alternative 2: Texas Instruments BA-II+ Professional Calculator. This calculator


has a few additional functions that are not found in the BA-II+.
For those considering taking the Chartered Financial Analyst Exam
or other professional exam in finance, you should purchase the
Professional version. It should cost about $60.

One (or both) calculators are available at various retail outlets in


Hilo including, the UHH Bookstore, Office Max, Wal-Mart,
Target, Amazon.

6
It is critical that you get access to a calculator early in the semester! It is
impossible to successfully complete the course without the required
calculator. Do not wait until the last minute to purchase your calculator as
the demand for these calculators exceeds supply in Hilo during peak
purchasing times.

You must have access to the correct calculator to complete the course. You
will receive a failing grade for the course if you do not have the correct
calculator by the end of the 4th week of class.

THE TWO CALCULATORS NOTED ABOVE ARE THE ONLY


CALCULATORS YOU MAY USE FOR THE COURSE. I WILL NOT
ALLOW YOU TO USE ANY OTHER CALCULATOR ON THE EXAMS!!

If you can not afford to buy a calculator, you may be able to borrow one from a
friend. I have been requiring these calculators for some years at UH-Hilo, so
most UHH business students have one. If neither are possible, please let me
know and we will find a solution for you.

Required: You must have computer access with internet.

Required: For exams, if you are not in the classroom with me when taking the
exam, you must have a detachable webcam for your computer. The internal camera
on a laptop computer will not be sufficient.

Required: I will email you a package of materials for the course. The materials contain
class overheads, special readings, homework assignments and solutions to the homework
assignments. You should have the materials available for class each day. I strongly
encourage you to print the package. In my experience students are able to take
proper notes better, learn better, and are more successful on exams if they have the
printed package.

Teaching Methods:

The primary teaching style for this course is class lectures and outside-of-class
homework.

Evaluation Tools:

Homework:

Homework will be assigned on a regular basis. You are expected to hand in your
homework at the assigned time. However, I will accept late homework with a grading
penalty.

7
Attendance Quizzes:

I believe that you learn by attending and participating in the class. Thus, attendance is
required and expected. I expect you to arrive to the classroom at least a minute or two
prior to the class starting time. You are also expected to remain in the class for the entire
class period. Unlike some professors, my lecture is disrupted by late entrants, early
departures or leaving and re-entering of the class. As such, these activities should occur
only RARELY! If you enter class late, please enter quietly to minimize the disruption!!

There will be daily attendance quizzes. I will drop your lowest quiz score when I
compute your final grade. Please sit in the same spot each day to facilitate conducting
the quiz quickly. The goal of the quizzes is for you to be motivated to arrive to class on
time, participate positively and to remain in class for the entire class period.
You will receive credit for one quiz each day you are in class. If you arrive late, depart
early, or exit and return, you will receive ½ credit for the day. If you arrive late to class,
you are responsible to note to me after class that you entered the class to earn the ½ quiz
credit. If you miss a combined total of more than 9 classes, 18 tardy days (or any
combination of the two) regardless of reason, your grade will be reduced by an additional
1/3 letter grade.

Note that for virtual classes, turning on your camera and allowing me to see you assures
me that you are participating in class. If you do not turn your camera on, I use your
responses (or lack thereof) to my questions to determine if you are participating in class.

Grading of Homework and quizzes:

Each homework assignment and quiz receives an equal weight in determining your
overall homework score. However, due to the longer nature of the Chapter 8 homework,
it receives the weighting of two homework assignments.

Extra Credit: There will not be any extra credit work available in this course.

Group Work: You are encouraged to work on the homework and to otherwise study in
groups. However, you are responsible for the material individually on exams.

Course Grading:
Exam I 100 points
Exam II 100 points
Third Exam (partially comprehensive) 125 points
Homework and quizzes 25 points
Total 350 points
* I will give you at least two weeks notice on the exact dates of the midterm exams.
To earn an A in the course, you need to earn 318.5 points (91%) or more
To earn an A- in the course, you need to earn 311.5 points (89%) or more
To earn a B+ in the course, you need to earn 304.5 points (87%) or more

8
To earn a B in the course, you need to earn 288.75 points (82.5%) or more
To earn a B- in the course, you need to earn 276.5 points (79 %) or more
To earn a C+ in the course, you need to earn 269.5 points (77%) or more
To earn a C in the course, you need to earn 245 points (70%) or more
To earn a C- in the course, you need to earn 234.5 points (67%) or more
To earn a D in the course, you need to earn 217 points (62%) or more

Behavior Policy

You are expected to behave in a manner that is consistent with the UHH Student Code of
Conduct. You are responsible for making yourself aware of the responsibilities specified
for you in this document. The document can be accessed at:
http://hilo.hawaii.edu/studentaffairs/conduct/student_conduct.php
I specifically wish to call your attention to the following activities that are not allowed in
class. If I see you doing any of the following, you will lose your attendance points for the
day. I do not announce these penalties in class but, when applied, you will see them
reflected in your final grade. Repeat violators are subject to additional grade reductions
at my discretion.
You are disruptive to the class in any way
You are not engaged in the class
Using your computer
Use of headphones
Text messaging or otherwise using your telephone
Working on assignments, or studying for another class
Carrying on a side conversation with someone else in class. If you have
something relevant to the class to comment on, it is most welcome, but
please share with us all.
Sleeping
Determining if you have violated any of these rules will be made at the professor’s sole
discretion and may involve elements of subjective evaluation.

I reserve the right to reassign your seating location in the class.


EXAM PROTOCOL

All exams will be taken on Laulima.

For in-person students, you must bring your computer to class on the exam day. You
must have internet access to take the exam.

For those taking the exam in a virtual session. You must have a detachable webcam for
the exams. You must turn on the camera and display the video through Zoom when you
are taking the exam. The internal camera on a laptop computer is not sufficient. I
reserve the right to ask you to display your work area with your camera when taking
exams including your cell phone, tablet, computer screen, workspace and your

9
surroundings. I will expect you to do this IMMEDIATELY upon my request with no
delay. I reserve the right to ask you to place your communication devices in a secure
location for exams.

Academic Dishonesty:

Do not cheat. The student conduct code and other university documents specify
significant penalties for cheating. These penalties will be enforced if I find you have
cheated in any way.

You should specifically note that I consider possessing or using exams from prior
semesters, in any form, an act of cheating.

I do not distribute my exams. Leaving the testing room with a copy of an exam, or
portion of an exam, in any form, is an act of cheating.

RECORDING POLICY
Students may record using various media during class for their own study purposes
provided they have obtained permission from the instructor prior to recording in class.
The recordings may not be shared and may not be posted on-line, e.g., YouTube, or on
other social media. You must delete all copies of the recordings at the end of the
semester.

SUPPORT AVAILABLE FOR STUDENTS


Short link: http://go.hawaii.edu/zAf
Mobile friendly link: http://go.hawaii.edu/GQe

UH Hilo provides a wide variety of support to students including tutoring, disability


services, mental health counseling, and EEO-Title IX support:

COMPUTER SUPPORT
The UH System Information Technology Services (ITS) Academic Technologies section
provides technical support via email and telephone. Contact the ITS Help Desk for
support by calling toll free (800) 558-2669 or by sending an email to [email protected].
For more information on available services please see the ITS website at
http://www.hawaii.edu/askus/588.

DISABILITY SERVICES
Any student with a documented disability who would like to request accommodations
should contact the Disability Services Office - Student Services Center E230, 932-7623
(V), 932-7002 (TTY), [email protected] - as early in the semester as possible.

ACADEMIC ADVISING
Advising is an important resource designed to help students navigate the University and
the requirements of their individual majors. Students should consult with their advisor at
least once a semester to decide on courses, check progress toward graduation, and discuss

10
career options and other educational opportunities offered at UH Hilo. Advising is a
shared responsibility, but students have final responsibility for meeting degree
requirements.

ACADEMIC INTEGRITY
Intellectual development requires honesty, responsibility, and doing your own work. All
UH Hilo students are expected to know and abide by the Student Conduct Code
(https://hilo.hawaii.edu/studentaffairs/conduct/student_conduct.php). Acts of dishonesty,
for example, plagiarism (taking thoughts/ideas or words from others), cheating, collusion,
or other forms of academic dishonesty will possibly result in penalties, consequences,
and/or other disciplinary actions. If you have any doubts or questions about what
constitutes academic misconduct, please do not hesitate to contact me. For information
about your student rights, contact Karishma Kamath, Director of Student Conduct, 932-
7472, [email protected].

ACADEMIC SUCCESS
Kilohana: The Academic Success Center provides a range of free academic services and
resources to all currently enrolled UH Hilo students. Services include access to peer
student staff from a range of academic majors and course-related pdf resources.

Until further notice all tutorial sessions will take place over video.

You can schedule a session with one of our tutors by emailing [email protected].
Please state the course alpha and number (e.g. MATH 140) in the subject line so staff
know what you need assistance with. Kilohana is scheduled to begin service Monday,
August 31, 2020.

Please check the Kilohana website at https://hilo.hawaii.edu/kilohana/ for information or


call 932-7291. Daily subject tutoring hours are listed via subject google calendars at
https://hilo.hawaii.edu/kilohana/tutoring.php . You can also contact Karla Hayashi,
Director of the Kilohana Academic Success Center, 932-7287, [email protected] or
Corin Kim, Kilohana STEM Coordinator, 932-7294, [email protected] for more
information.

MENTAL HEALTH/SUICIDE PREVENTION


The UH Hilo community is committed to and cares about all students. Life at college can
get complicated. Students sometimes feel overwhelmed, lost, experience anxiety or
depression, struggle with relationship difficulties, family responsibilities, or diminished
self-esteem. However, supportive services are available and effective. UH Hilo
Counseling Services helps undergraduate and graduate students cope with difficult
emotions and life stressors. Counseling Services is staffed by experienced, professional
counselors, who are attuned to the diverse needs of all types of college students. The
services are FREE and completely confidential.

Find out more at https://hilo.hawaii.edu/studentaffairs/counseling or by calling 932-7465.

11
For immediate help, contact The Crisis Line of Hawaii 1-800-753-6879, the National
Suicide Prevention Hotline 1-800-273-8255 (suicidepreventionlifeline.org), or text
“Aloha” or “Hello” to the Crisis Text Line 741-741.

STUDENT CONDUCT
All members of UH Hilo have the right to pursue educational endeavors as part of the
institution's fundamental purposes of teaching, learning, and research. Students have the
responsibility to maintain standards of personal integrity (honesty, civility, and respect)
that are in harmony with the educational mission of the University; to respect the rights,
privileges, and property of others; and to observe University policies as well as national,
state, and local laws. The student conduct process holds students accountable for
violations of university policy and all UH Hilo students are expected to know and abide
by the Student Conduct Code
(https://hilo.hawaii.edu/studentaffairs/conduct/student_conduct.php).

STUDENTS OF CONCERN
University life should be challenging, not overwhelming. There may be times when
students experience severe stress or distress from adjusting to college life and adapting to
unforeseen changes/circumstances. Consider referring a friend, fellow classmate, or
yourself to the UH Hilo CARE Team who can provide information, resources, and
options for successfully navigating a challenging academic life. Typical referrals include
behavior which raise significant concerns and reasonably suggests that the behavior,
physical and/or emotional state:

Presents safety issues for yourself or others; or


May hinder one’s ability to meet expected standards of conduct or achieve academic
requirements; or
May be evidence of severe emotional distress
Anyone may use this (https://hilo.hawaii.edu/careteam/report.php) form to refer a UH
Hilo student to the University's CARE Team. All matters discussed are regarded as
highly confidential and are protected by the Federal Educational Rights and Privacy Act
(FERPA). If you have any questions about filing a report, please contact Karishma
Kamath, the CARE Team Coordinator at (808) 932-7472, or [email protected].

TITLE IX SYLLABUS
The University of Hawaii is committed to providing a learning, working and living
environment that promotes personal integrity, civility, and mutual respect and is free of
all forms of sex discrimination and gender-based violence, including sexual assault,
sexual harassment, gender-based harassment, domestic violence, dating violence, and
stalking. If you or someone you know is experiencing any of these, the University has
staff and resources on your campus to support and assist you. Staff can also direct you to
resources that are in the community. Here are some of your options:

If you wish to remain ANONYMOUS, speak with someone CONFIDENTIALLY, or


would like to receive information and support in a CONFIDENTIAL setting, contact:

12
UH Hilo Counseling Services: SSC, room E-203, 932-7465
UH Hilo Medical Services: Campus Center, room 212, 932-7369

REPORTING INCIDENTS
If you wish to REPORT an incident of sex discrimination or gender-based violence
including sexual assault, sexual harassment, gender-based harassment, domestic violence,
dating violence or stalking as well as receive information and support*, contact:

Jenna Waipa, Lead Deputy Title IX Coordinator, 932-7818, [email protected]


Kalei Rapoza, Interim Vice Chancellor for Administrative Affairs, 932-7626,
[email protected]

* Please note that you do not have to file a complaint with the University to receive
institutional support or assistance.

As a member of the University faculty, I am required to immediately report any incident


of sex discrimination or gender-based violence to the campus Title IX Coordinator.
Although the Title IX Coordinator and I cannot guarantee confidentiality, you will still
have options about how your case will be handled. My goal is to make sure you are
aware of the range of options available to you and have access to the resources and
support you need.

For more information regarding sex discrimination and gender-based violence, the
University’s Title IX resources and the University’s Policy, Interim EP 1.204, go to:
http://www.hawaii.edu/titleix

13
Formula Sheets and Review Sheets for
Exams

These review and formula sheets are samples.

I will revise them to reflect the current semester


materials.

Be sure to use the revised sheets that I distribute


in class when studying for your exam.

14
Exam 1 Review Sheet

15
Finance 320 Exam 1: SAMPLE:

Notes Regarding the Exam:

We will have an in class review for the exam on…...


I will come through the library on the evening of……, to help you out with any last minute
questions you may have for the exam. I will arrive at about …… and stay until …… or until people stop
asking me questions, whichever occurs first.

For the exam:

You will be allowed to use the formula sheet on the exam. The formula sheet will be identical to the one
that I have handed out prior to the exam. I will provide you a clean copy of the formula sheet on
the exam, so you may mark up the one I am giving you prior to the exam as you wish.

You will not be allowed to use the sample questions or review sheet on the exam.

Extra paper will not be necessary for the exam. I will provide plenty of room on the exam for you to
answer the questions.

Your exam will cover Chapters 1, 2, 4, 5, 6. You are responsible for Certifications in Finance, however, this topic
will only be tested on a surface level.

16
Fin 320 Sample Exam Questions
These questions are typical of questions that I might ask on an exam.

1. What are the four Corporate characteristics?

a. ___________
b. ___________
c. ___________
d. ___________

2) You are considering investing in a stock but you want to evaluate the expected return and
riskiness of the stock before you make the investment. Based on your best estimates, you have
accumulated the following information:

State Prob Return


Boom .3 15
Normal .2 12
Bad .5 5

Calculate the expected return on this stock

3.) The nominal risk free rate of interest has several components. These components are

a. The real risk free rate of interest and the liquidity premium
b. The inflation premium and the default risk premium
c. The real risk free rate of interest and the inflation premium
d. The liquidity premium and the inflation premium
e. The maturity risk premium and the default risk default risk premium.

17
Fin 320 Exam 1 Review SAMPLE

While I do try to make the review sheet as inclusive as possible, we may have discussed items in class
that are not covered on the review sheet. Anything that we covered in class is fair game for the exam.

Chapter 1:
a. Organizational Forms
Characteristics of organizational forms
How to determine if you are a partnership or a corporation
How you are taxed if you are a partnership or a corporation
How to calculate the amount of taxes you will have to pay if you are a partnership or a corporation
How do other organizational forms such as S corporations or limited liability companies fit into the
picture.
What are the characteristics of S corporations and limited liability companies.
b. Agency Problems
The agency problem between stockholders and managers
The agency problem between stockholders and bondholders

Readings:

#1 Does School Matter, an Empirical Examination of CEO Education, Compensation and Firm
Performance.

Chapter 2: Financial Statements and Taxes


a. Balance Sheet
b. Income Statement
c. Statement of Cash Flows
d. Statement of Retained Earnings
e. Understand the relationships between the various financial statements.
f. Financial Ratios and how to use them.
a. Ratio Calculation
i. Market Value of Equity
ii. Price to Earnings Ratio
iii. Total Asset Turnover
iv. Return on Assets
v. Profit Margin
vi. Debt to Assets
vii. Current Ratio
b. traditional measures of performance
a. time series analysis
b. industry analysis
c. new measures of firm performance
c. EVA
d. Balanced Scorecard
e. Market Value Added
f. Wealth Added Index
g. Tracking Stocks
g. How to compute Taxable Income
taxable income = salary + other income – deductions – exemptions
h. How to compute the tax owed
tax owed = tax on base amount + tax rate (taxable income – base amount)
i. How to use a business to reduce your personal taxes

18
Chapter 4: Forecasting Financial Statements
a. How to use the RMA ratios to forecast the income statement and balance sheet starting from a
owners equity contribution figure or a sales figure.

Sales-to-Total Assets = Sales/Total Assets

Total Assets = Common Stock and Ret. Earn./percent of funds from this source

Chapter 5: Financial Markets


a. Classifications of Financial Markets and the role of financial markets
b. The Capital Formation Process
c. Components of Interest Rates
k = k* + IP + DRP + LP + MRP, kRF = K* + IP
d. What is the Yield Curve and how do we graph it.
e. Theories of the Yield Curve and what they imply about if we should borrow long term or short
term
f. Forecasting Interest Rates

(1 + S 0 − L ) L = (1 + S 0 − N ) n1 (1 + X n− L ) n 2

( (1 + S 0 − L) L
(1 + S 0 − N ) n1
) 1
n2
− 1 = X n− L

Chapter 6:

Measures of Return and how to use them


How to use these returns to determine if we should make an investment.
How to use these return to determine if an investment has performed well.
How to use these returns to evaluate our decision making abilities.

1. For A Single Stock How to compute


a. Expected Return


1. Using subjective data K = ∑ PiKi

2.

Using historical data K = K avg =
∑ Kh
N

b. Required Return: Kreq = Krf + βi ( Km − Krf )

AmtMoneyEndYr − AmtMoneyBegYr + Cash Re c


c. Realized Return Kr =
AmtMoneyBegYr

19
2. For a Portfolio

a. Expected Return
   
1. Using subjective data Kp = W 1K1 + W 2 K 2 + ...WnKn


2.Using Historical data Kp = K avgp =
∑ Khp
N
where Khp = W 1Kh1 + W 2 Kh 2 + ...WnKhn

b. Required return Kreqp = W 1Kreq1 + W 2 Kreq 2 + ...WnKreqn

AmtMoneyEndYr − AmtMoneyBegYr + Cash Re c


c. Realized Return Krp =
AmtMoneyBegYr

Measures of Risk

a. Stand Alone
1. Standard deviation
 2
a. from subjective (forward looking) data σ= ∑ ( ki − k ) pi

b. from historical data S=


∑ (kh − K avg ) 2

N −1

σ S
2. Coefficient of Variation CV =  or
K K avg

b. Portfolio

How does combining stocks into a portfolio affect the risk of your investments?

1. Standard deviation

a. From historical data Sp =


∑ (khp − K avgp) 2

N −1

The role of the covariance and correlation coefficient in determining the


standard deviation on a portfolio.

Sp
2. Coefficient of Variation CVp =
K avgp

20
How does portfolio size affect risk?

a. systematic risk versus unsystematic risk.


b. measure of systematic risk

How to use the standard deviation

1. Confidence Intervals

a. Single Stock

CI 68 = K ± 1σ , orK avg ± 1S

CI 95 = K ±2σ , orK avg ± 2 S

CI 99.75 = K ±3σ , orK avg ± 3S

b. Portfolio

CIp 68 = K avgp ± 1Sp


CIp95 = K avgp ± 2 Sp
CIp99.75 = K avgp ± 3Sp

Chapter on Business Risk

1. What is Leverage and how do fixed costs affect returns


2. Break Even Analysis

3. Degree of Operating Leverage

4. Degree of Financial Leverage

5. Degree of Combined Leverage

21
Exam 1 Formula Sheet

22
Fin 320 Principles of Finance: Exam 1 Formula Sheet SAMPLE

Taxable Income = Salary + other income – deductions – exemptions


Tax Owed = Tax on Base Amount + Tax Rate (Taxable Income – Base Amount)

Financial Statement Conventions for Chapter 2

Assets
Cash
Accounts Receivable
Inventories
Total Current Assets
Gross Fixed Assets
Less Accumulated Deprecation
Net Fixed Assets
Total Assets

Liabilities and Equity


Accounts Payable
Notes Payable
Accruals
Total Current Liabilities
Long Term Debt
Total Liabilities
Common Stock
Additional Paid In Capital
Retained Earnings
Total Equity
Total Liabilities and Equity

Sales
Cost of Goods Sold (COGS)
Other Expenses
Depreciation
Total Operating Costs
EBIT
Interest Expense
EBT
Taxes (rate)
Net Income

Balance of Retained Earnings Previous Year


+ Net Income
- Dividends Paid
Balance of Retained Earnings Current Year

Step 1: Compute the Change in Cash Position

Net Income
+ Depreciation
Increases in Liabilities
+ Increases in Accounts Payable
+ Increases in Notes Payable
+ Increases in Accruals
+ Increases in Long-Term Debt

23
Increases in Assets
- Increases in Inventory
- Increases in Accounts Receivable
- Increases in other Current Assets
- Increases in Gross Fixed Assets
Cash Paid to and Received from Stockholders
+ New Investment by Stockholders
- Dividends Paid
= Change in Cash Position

Step 2: Compute the New Cash Balance

Cash Balance
+ Change in Cash Position
= Cash Balance

𝑀𝑀𝑀𝑀𝑀𝑀𝑀𝑀𝑀𝑀𝑀𝑀 𝑉𝑉𝑉𝑉𝑉𝑉𝑉𝑉𝑉𝑉 𝑜𝑜𝑜𝑜 𝐸𝐸𝐸𝐸𝐸𝐸𝐸𝐸𝐸𝐸𝐸𝐸 = 𝑆𝑆ℎ𝑎𝑎𝑎𝑎𝑎𝑎𝑎𝑎 𝑂𝑂𝑂𝑂𝑂𝑂𝑂𝑂𝑂𝑂𝑂𝑂𝑂𝑂𝑂𝑂𝑂𝑂𝑂𝑂𝑂𝑂 𝑋𝑋 𝑃𝑃𝑃𝑃𝑃𝑃𝑃𝑃𝑃𝑃 𝑃𝑃𝑃𝑃𝑃𝑃 𝑆𝑆ℎ𝑎𝑎𝑎𝑎𝑎𝑎

𝑀𝑀𝑀𝑀𝑀𝑀𝑀𝑀𝑀𝑀𝑀𝑀 𝑉𝑉𝑉𝑉𝑉𝑉𝑉𝑉𝑉𝑉 𝑜𝑜𝑜𝑜 𝐸𝐸𝐸𝐸𝐸𝐸𝐸𝐸𝐸𝐸𝐸𝐸 𝑂𝑂𝑂𝑂𝑂𝑂𝑂𝑂𝑂𝑂𝑂𝑂ℎ𝑖𝑖𝑖𝑖


𝑃𝑃𝑃𝑃𝑃𝑃𝑃𝑃𝑃𝑃 𝑡𝑡𝑡𝑡 𝐸𝐸𝐸𝐸𝐸𝐸𝐸𝐸𝐸𝐸𝐸𝐸𝐸𝐸𝐸𝐸 𝑅𝑅𝑅𝑅𝑅𝑅𝑅𝑅𝑅𝑅 =
𝑁𝑁𝑁𝑁𝑁𝑁 𝐼𝐼𝐼𝐼𝐼𝐼𝐼𝐼𝐼𝐼𝐼𝐼
𝑆𝑆𝑆𝑆𝑆𝑆𝑆𝑆𝑆𝑆
𝑇𝑇𝑇𝑇𝑇𝑇𝑇𝑇𝑇𝑇 𝐴𝐴𝐴𝐴𝐴𝐴𝐴𝐴𝐴𝐴 𝑇𝑇𝑇𝑇𝑇𝑇𝑇𝑇𝑇𝑇𝑇𝑇𝑇𝑇𝑇𝑇 =
𝑇𝑇𝑇𝑇𝑇𝑇𝑇𝑇𝑇𝑇 𝐴𝐴𝐴𝐴𝐴𝐴𝐴𝐴𝐴𝐴𝐴𝐴
𝑁𝑁𝑁𝑁𝑁𝑁 𝐼𝐼𝐼𝐼𝐼𝐼𝐼𝐼𝐼𝐼𝐼𝐼
𝑅𝑅𝑅𝑅𝑅𝑅𝑅𝑅𝑅𝑅𝑅𝑅 𝑜𝑜𝑜𝑜 𝐴𝐴𝐴𝐴𝐴𝐴𝐴𝐴𝐴𝐴𝐴𝐴 =
𝑇𝑇𝑇𝑇𝑇𝑇𝑇𝑇𝑇𝑇 𝐴𝐴𝐴𝐴𝐴𝐴𝐴𝐴𝐴𝐴𝐴𝐴
𝑁𝑁𝑁𝑁𝑁𝑁 𝐼𝐼𝐼𝐼𝐼𝐼𝐼𝐼𝐼𝐼𝐼𝐼
𝑃𝑃𝑃𝑃𝑃𝑃𝑃𝑃𝑃𝑃𝑃𝑃 𝑀𝑀𝑀𝑀𝑀𝑀𝑀𝑀𝑀𝑀𝑀𝑀 =
𝑆𝑆𝑆𝑆𝑆𝑆𝑆𝑆𝑆𝑆
𝑇𝑇𝑇𝑇𝑇𝑇𝑇𝑇𝑇𝑇 𝐿𝐿𝐿𝐿𝐿𝐿𝐿𝐿𝐿𝐿𝐿𝐿𝐿𝐿𝐿𝐿𝐿𝐿𝐿𝐿𝐿𝐿
𝐷𝐷𝐷𝐷𝐷𝐷𝐷𝐷 𝑡𝑡𝑡𝑡 𝐴𝐴𝐴𝐴𝐴𝐴𝐴𝐴𝐴𝐴𝐴𝐴 =
𝑇𝑇𝑇𝑇𝑇𝑇𝑇𝑇𝑇𝑇 𝐴𝐴𝐴𝐴𝐴𝐴𝐴𝐴𝐴𝐴𝐴𝐴

Current Assets
Current Ratio =
Current Liabilities

24
Financial Statement Conventions for Chapter 4

Sales
Cost of Goods Sold (Sales – Gross Profit)
Gross Profit (%)
Operating Expenses (%)
Earnings Before Interest and Taxes (%)
(Operating Profit)
Interest Expense (%)
(Other Expenses)
Earnings Before Tax (%)

Assets
Cash and Equivalents % $
Trade Receivables % $
Inventory % $
Other Current Assets % $
Total Current Assets % $
Fixed Assets (Net) % $
Intangibles % $
Other Non-Current Assets % $
Total Assets 100% $

Liabilities

Notes Payable % $
Current Maturity LT Debt % $
Accounts Payable % $
Income Taxes Payable % $
All Other Current % $
Total Current Liabilities % $
Long Term Debt % $
Deferred Taxes % $
All other Non-Current Liabilities % $
Common Stock and Retained Earnings % $
Total Liabilities and Equity 100%

Sales-to-Total Assets = Sales/Total Assets

Total Assets = Common Stock and Ret. Earn./percent of funds from this source

25
k = k ∗ + IP + DRP + LP + MRP , krf = k ∗ + IP

(1 + S 0 − L ) L = (1 + S 0 − N ) n1 (1 + X n− L ) n 2

(((11++SS 0−L )L
0 − N)
n1
) 1
n2
− 1 = X n− L


K = ∑ PiKi


K = K avg =
∑ Kh
N

Kreq = Krf + βi ( Km − Krf )

AmtMoneyEndYr − AmtMoneyBegYr + Cash Re c


Kr =
AmtMoneyBegYr
   
Kp = W 1K1 + W 2 K 2 + ...WnKn


Kp = K avgp =
∑ Khp where Khp = W 1Kh1 + W 2 Kh2 + ...WnKhn
N

Kreqp = W 1Kreq1 + W 2 Kreq 2 + ...WnKreqn

AmtMoneyEndYr − AmtMoneyBegYr + Cash Re c


Krp =
AmtMoneyBegYr

σ= ∑ (ki − k ) 2
pi

S=
∑ (kh − K avg ) 2

N −1

σ S
CV =  or
K K avg

Sp =
∑ (khp − K avgp) 2

N −1

Sp
CVp =
K avgp

CI 68 = K ± 1σ , orK avg ± 1S CIp 68 = K avgp ± 1Sp

26

CI 95 = K ±2σ , orK avg ± 2 S CIp95 = K avgp ± 2 Sp

CI 99.75 = K ±3σ , orK avg ± 3S CIp99.75 = K avgp ± 3Sp

^ ^

σij = ∑
[( Ki − Ki ) X ( Kj − Kj )]
N −1
σij
ρij =
σiXσj

27
Review Sheet for Exam 2

28
Review Sheet for Exam 2
Fin 320 Principles of Finance
SAMPLE

Notes Regarding the Exam

A. Your exam will be DATE.


B. Your Chapter 9 Homework is due on the Day of the Examination.
C. We will have an in class review for the exam DATE.
D. I will come through the library on the evening of DATE arriving around HOUR pm and staying until
about HOUR pm.
E. There will be approximately 20 questions on the exam. The primary focus of the exam is on problem
solving.
F. I will provide you with a clean copy of the formula sheet for the exam.
G. You may use the formula sheet and your calculator on the exam. You can not use this review sheet for
the exam.
H. Make sure to bring your calculator to the exam.
I. While I try to make the review sheet comprehensive, I do not guarantee them. Any materials
that we have covered in class, readings or in the homework are fair game for the exam.

29
Chapter 8:

Time Value of Money Terminology and tools for determining the time value of money

A. Single Sum
B. payment
C. annuity and annuity due
D. perpetuity, perpetuity due and growing perpetuity
E. uneven Cash Flow Streams
F. Complex Problems
G. combination Problems
H. amortization
I. effective annual interest rates
J. Time Lines
K. Simple interest versus compound interest
L. Formula’s for calculating the present value and the future value of a single sum.

How to calculate:

A. Present Value of a Single Sum


B. Future Value of a Single Sum
C. Present Value of an Ordinary Annuity
D. How to compute present and future values with non-annual compounding
E. Present and Future Value with continuous compounding
F. Future Value of an Ordinary Annuity
G. Present Value of an Annuity Due
H. Future Value of an Annuity Due
I. Present Value of an ordinary perpetuity
J. Present Value of a perpetuity due
K. Present Value of a growing perpetuity
L. Present value of an uneven cash flow stream
M. Future value of an uneven cash flow stream.
N. Present Value of a Combination Problem
O. Incorporating non-annual payments and compounding into the above problems
P. Effective annual interest rates
Q. How to create an amortization table
R. How to solve for the interest rate and number of years in time value of money calculations.

30
Chapter 10:

Concepts

A. What does the General Valuation Model tell us?


B. How do you determine what your company is worth?
C. Why do we need to make assumptions about dividends to calculate the value of a share of
common stocks?

Stock Calculations

A. Dividend Valuation
The constant growth dividend valuation model
The zero growth dividend valuation model
The supernormal growth dividend valuation model
A. Comparables Method
B. From Share Price to firm Value
C. Computing the Required Rate of Return
Historical method
Using the Security Market Line
Chapter 9:

Bond Concepts

A. Borrowing from a bank versus through a bond


B. Face Value
D. Maturity
E. Issue Date
F. Coupon Payment
G. Coupon Rate
H. Callable Bonds
I. Putable Bonds
J. Convertible Bond
K. Default Risk

Bond Calculations

A. Coupon Payment
B. Value of a bond today (annual and semiannual coupon payments)
C. The effect of a change in interest rates on the value of a bond.
D. Yield to Maturity
E. Yield to Call

31
Exam 2 Formula Sheet

32
Formula Sheet for Second Exam: Principles of Finance Fin 320

Stream of Cash Yes Limited Number Yes Equal Time Yes Equal Dollar Yes
Annuity
Flows? of Cash Flows? Spacing? Amount?

No No No No

Uneven Cash Uneven Cash


Single Sum
Flow Stream Flow Stream

Equal Time Yes Equal Dollar Yes


Perpetuity
Spacing? Amount?

No No

Constant Yes Growing


Percentage
Perpetuity
Growth?

No

Combination Combination
Problem Problem

A B C D E F G H
|----------|----------|----------|----------|----------|----------|----------|
First Middle Last
Cash Flow Cash Flows Cash Flow

PV PV FV FV
OA AD OA AD

Mode END BEG END BEG

(1) (2) (3) (4) (5) (6)


Month Beg Bal Payment Interest Principal End Bal

PMT 1
PVOP =
kreq

PMT 0
PVPD = + PMT 0
kreq

33
PMT 1 PMT − 0 (1 + g )
PVGOP = =
kreq − g kreq − g

PMT 0 (1 + g )
PVGPD = + PMT 0
kreq − g

^ Do(1 + g ) D1
Po = =
Kreq − g Kreq − g

^ Do
Po =
Kreq

Dt = Dt-1(1+g)

^
P o = EPS X PEC

^ ^
V o = P o X SO

Kreq = KRF + B ( Km − KRF )

34
Exam 3 Review Sheet

35
Review Sheet for Exam 3
Fin 320 Principles of Finance
SAMPLE

Notes Regarding the Exam

A. You may take your exam at the following time:

B. Your exam will cover Chapters 8, 10, 9, 14, 13 and 11, only as far as we cover in class
C. Your Chapter 13 homework is due on the day of the examination.
D. We will have an in class review for the exam during our regular class time on.
E. Class will not meet on DATE. However, I will be available in my office during our
regular class times for consultation on. We have been meeting an extra 5 minutes each day
throughout the semester so we have earned this day off.
F. I will come through the library on the evening of DATE.
G. I will be in my office on DATE from about TIME.
H. There will be approximately 20 questions on the exam. The primary focus of the exam is on
problem solving.
I. I will provide you with a clean copy of the formula sheet for the exam.
J. You may use the formula sheet and your calculator on the exam. You can not use this review
sheet for the exam.
K. Make sure to bring your calculator to the exam.
L. While I try to make the review sheet comprehensive, I do not guarantee them. Any materials
we have covered in class, reading or in the homework are fair game for the exam.
M. I will e-mail your exam grade to you if you would like. I will request your e-mail address on
the exam for that purpose.
N I do not distribute my final exams but you may view your exam in my office.

Chapter 8:
Time Value of Money Terminology and tools for determining the time value of money
A. Single Sum
B. payment
C. annuity and annuity due
H. perpetuity, perpetuity due and growing perpetuity
I. uneven Cash Flow Streams
J. Complex Problems
K. combination Problems
H. amortization
I. effective annual interest rates
K. Time Lines
L. Simple interest versus compound interest
M. The mathematical formula for computing the present value of a single sum
N. The mathematical formula for computing the future value of a single sum.

36
How to Calculate

A. Present Value of a Single Sum


B. Future Value of a Single Sum
C. Present Value of an Ordinary Annuity
D. How to compute present and future values with non-annual compounding
E. Future Value of an Ordinary Annuity
F. Present Value of an Annuity Due
G. Future Value of an Annuity Due
H. Present Value of an ordinary perpetuity
I. Present Value of a perpetuity due
J. Present Value of a growing perpetuity
K. Present value of an uneven cash flow stream
L. Future value of an uneven cash flow stream.
M. Present Value of a Combination Problem
N. Incorporating non-annual payments and compounding into the above problems
O. Effective annual interest rates
P. How to create an amortization table
Q. How to solve for the interest rate and number of years in time value of money calculations.

Chapter 10:

Concepts

A. What does the General Valuation Model tell us?


J. Why do we need to make assumptions about dividends to calculate the value of a share of
common stocks?

Stock Calculations

A. Dividend Valuation
The constant growth dividend valuation model
The zero growth dividend valuation model
The supernormal growth dividend valuation model
B. Comparables Method
C. From Share Price to firm Value
D. Computing the Required Rate of Return
Historical method
Using the Security Market Line

Chapter 9:
Bond Concepts
A. Borrowing from a bank versus through a bond
B. Face Value
K. Maturity
L. Issue Date
M. Coupon Payment
N. Coupon Rate

37
O. Callable Bonds
L. Convertible Bond
M. Default Risk

Bond Calculations
A. Coupon Payment
B. Value of a bond today (annual and semiannual coupon payments)
C. The effect of a change in the interest rate on the value of a bond.
D. Yield to Maturity
F. Yield to Call

Chapter 14:
a. Gathering information about the cash flows

b. Initial Cash Flows:


A. cost of machine and installation
B. changes in net working capital

c. Operating Cash Flows


A. Revenues and Expenses
B. Depreciation
C. Calculation of operating cash flows in each year.

d. Terminal Cash Flows


A. Recovery of Net Working Capital
B. Sales Price of machine
C. Taxes due on sale of old machine

Chapter 13
a. Mutually Exclusive versus Independent Projects
b. Normal versus non-normal projects
c. The required rate of return
d. Know how to calculate, the decision rule for, the advantages, the disadvantages, and when to use
each of the following

I. payback period
II. Net Present Value
III. Internal Rate of Return.
IV. Profitability Index

e. How do you handle projects with unequal lives?

38
Exam 3 Formula Sheet

39
Formula Sheet for Second Exam: Principles of Finance Fin 320

Stream of Cash Yes Limited Number Yes Equal Time Yes Equal Dollar Yes
Annuity
Flows? of Cash Flows? Spacing? Amount?

No No No No

Uneven Cash Uneven Cash


Single Sum
Flow Stream Flow Stream

Equal Time Yes Equal Dollar Yes


Perpetuity
Spacing? Amount?

No No

Constant Yes Growing


Percentage
Perpetuity
Growth?

No

Combination Combination
Problem Problem

A B C D E F G H
|----------|----------|----------|----------|----------|----------|----------|
First Middle Last
Cash Flow Cash Flows Cash Flow

PV PV FV FV
OA AD OA AD

Mode END BEG END BEG

(1) (2) (3) (4) (5) (6)


Month Beg Bal Payment Interest Principal End Bal

PMT 1
PVOP =
kreq

PMT 0
PVPD = + PMT 0
kreq

40
PMT 1 PMT − 0 (1 + g )
PVGOP = =
kreq − g kreq − g

PMT 0 (1 + g )
PVGPD = + PMT 0
kreq − g

^ Do(1 + g ) D1
Po = =
Kreq − g Kreq − g

^ Do
Po =
Kreq

Dt = Dt-1(1+g)

^
P o = EPS X PEC

^ ^
V o = P o X SO

Kreq = KRF + B ( Km − KRF )

Total Revenues
-Operating Costs (excluding depreciation)
-Depreciation
=Earnings before Interest and Taxes (EBIT)
-Interest
=Earnings Before Taxes (EBT)
-Tax
= Net Income
+ Depreciation
= Net Operating Cash Flows
Equipment Cost
Installation
Increase in Net Working Capital (NWC)

Sales Price
Tax on Gain on Sale
Return of Net Working Capital (NWC)

Net Cash Flows

41
DepreciableBase − SalvageValue
AnnualDepreciation =
DepreciableLife

Annual Dep. = Dep. Base X MACRS Rate

MACRS Depreciation Percentages 3 yr. 33,45,15,7: 5 yr. 20,32,19,12,11,6

Step 1: Calculate the Depreciation taken to Date


Step 2: Calculate the Book Value
Step 3: Calculate the Taxable Gain or (Loss)
Step 4: Calculate Tax Owed

WACC = Wst(Kreq) + Wd(Kd)(1-T)

− LastNegCumCashFlow
PB = BaseYear +
FollowingCashFlow

NPV
PI =
Costof Pr oject

42
The University of Hawaii at Hilo Finance Program
Rev 8/20/2020 Subject to Change
Be sure to check the University course catalog for the latest updates and additional details

COURSE OFFERING PLANS FOR FINANCE AT UH-HILO

*Finance 220: Personal Finance (Fall and Spring Semester, depends upon instructor availability)
Finance 320: Principles of Finance (Fall and Spring Semester)
Finance 321: Investment & Securities Analysis (Spring Semester)
Finance 322: Corporate Finance (Spring Semester)
Finance 370: Principles of Real Estate (Spring Semester, depends upon Instructor availability)
Finance 325: Small Business Finance (Even Year Fall Semesters)
Finance 412: Option and Futures (Odd Year Fall Semesters)

Sequencing: You must take Finance 320 first. After successfully completing Finance 320, you may take any
other finance course in any sequence. For example, you can take Finance 412 before taking Finance 321.
These course schedules change from time to time. Many factors can affect course offerings including
illnesses, sabbaticals, and faculty availability. Please check the University website for current schedules.
* Finance 220 cannot be used to satisfy upper division business elective requirements. But it does fulfill a
Global and Community Citizenship (GCC), Social Science (DS) or Quantitative Reasoning B (GQ) requirement.

CONCENTRATION ELEMENT OF THE BUSINESS ADMINISTRATION MAJOR

Bachelor of Business Administration students, with majors in business, using catalogs from 2011-2012 or
later, must earn at least one concentration (Finance, Management or Marketing) as part of the BBA major.
Students may earn multiple concentrations by completing the necessary requirements for each. Courses
counted for one concentration may be used to partially complete the requirements for another concentration.

Finance Concentration (rev 2012-2013) and later. Business Administration Students electing to concentrate
in Finance must complete the following courses with a grade of C or better.

Finance 320: Principles of Finance (required for all business and accounting majors)
Finance 321: Investment & Securities Analysis
Finance 322: Corporate Finance
Two additional upper division courses with a FIN alpha
Two additional upper division courses from any business alpha (only 1 may be Econ on some catalogs)

CERTIFICATE IN FINANCE

The certificate in finance is designed for non-business majors who wish to pursue specific study in the area of
finance. To earn the certificate in finance, students must complete the following courses each with a grade
of C or better. They must also complete a Bachelor’s degree in any area.

Accounting 201: Introduction to Financial Accounting


Math 104F Pre-calculus Math or Math 104 or Math 115, or Math 205 or higher
Economics 130: Introduction to Microeconomics
Finance 320: Principles of Finance
Finance 321: Investment & Securities Analysis (Fall Semester)
Finance 322: Corporate Finance (Spring Semester)
One 3, credit, upper division, finance elective
One 3, credit, upper division, Finance, Accounting or Economics Elective

43
CONCENTRATIONS

You must earn at least one concentration as part of the BBA Degree.

EARNING MULTIPLE CONCENTRATIONS

It is often easy to earn a second concentration. For Accounting Majors, it easy to


earn a second major with a Finance Concentration. How to earn more than one
concentration or major.

Finance Concentration Management Concentration


Finance 321 6 UD Business Elective Courses
Finance 322 No more than 2 from any alpha
UD Finance Elective
UD Finance Elective
UD BUS Elective
UD Bus Elective

EXAMPLE

Finance Concentration Management Concentration


Finance 321 Finance 321
Finance 322 Finance 322
(Finance 325) ACC 350
(Finance 412) ACC 351
(ACC 350) BUS 400
(ACC 351) MKT 315

In this example, someone earning a Finance Concentration, could add a


Management Concentration by taking only 2 extra classes.

In this example, an Accounting Major could add a Business Major with a Finance
Concentration by taking four extra classes beyond those required for the
Accounting Major. You would have a BBA Degree with Major in Accounting and
Business Administration with a Finance Concentration.

44
Licenses Certifications and Salaries in Finance

45
Salaries in Finance

Senior Financial Analyst $81,098


Director of Finance $113,000
Vice President of Finance $125,500
Chief Financial Officer $132,000
(source, Robert Half International Survey, average salaries 2018)

Financial Analyst: $84,300


Loan Officers: $64,600
Personal Financial Advisor: $90,640
(Source: Bureau of Labor Statistics, Median Salaries 2018)

Investment Banking Starting Salary


First-Year Analyst $100,000-$150,000 w/Bachelor Deg.
Third-Year Analyst $120,000-$350,000 w/Bachelor Deg.
First Year Associate $120,000-$220,000 w/Masters Deg.
Third Year Associate $260,000-500,000 w/Masters Deg.
Vice President $250,000-$1,000,000 3-6 Yrs. Exp.
Managing Director $400,000-$20,000,000 5-10 Yrs. Exp.
Department Head $800,000-$70,000,000 10+ Yrs. Exp.
(Source: Careers in Investment Banking, 2012 Data)
http://www.careers-in-finance.com/ibsal.htm

Financial Planning: Financial Advisor Trainee: $52,000


Financial Analyst $64,000
Source: Glassdoor.com
https://www.glassdoor.com/Salaries/financial-planner-salary-
SRCH_KO0,17.htm

Chartered Financial Analyst:


Average Compensation $300,000
(Source: CFA Institute)

Actuary $70,000-160000
Source (Salary.com, 2018)

Certified Financial Planner $68,883


Source (payscale.com, 2018)

46
License in Finance

In order to do many things in finance you must have a license. These license
are similar to a drivers license for your car. It says you have met the
minimum requirements to do this particular thing in finance.

Series 6: Investment Company/Variable Contracts Representative


Series 7: General Securities Representative
Series 63: Uniform Securities Agent
Series 64: Uniform Real Estate Securities Exam
Series 65: Uniform Registered Investment Advisor
There are many other licenses available

Certifications

A certification indicates you have reached a high level of competence in an


area.

Investments

Chartered Financial Analyst (CFA)


Website www.cfainstitute.org

First awarded in 1963, the CFA charter has become known as the gold standard of
professional credentials within the global investment community. Around the world,
employers and investors recognize the CFA designation as the definitive standard for
measuring competence and integrity in the fields of portfolio management and investment
analysis.

Chartered Market Technician (CMT)


http://www.mta.org/

The Chartered Market Technician (CMT) Program is a certification process in which candidates
are required to demonstrate proficiency in a broad range of technical analysis subjects.
Administered by the Accreditation Committee of the Market Technicians Association (MTA), the
CMT Program consists of three levels: Level 1 and 2 are multiple choice exams; at level 3,
candidates have the option of writing a research paper or taking a third(essay) exam

Certified Fund Specialist (CFS) www.icfs.com

The CFS is the oldest designation in the mutual funds industry. After you complete the program
your knowledge about mutual funds will be head and shoulders above the competition. The
information and marketing ideas you will learn are unparalleled. Studies research reports, industry
trends, new ideas and products, sales presentations and marketing tips among others.

The CFS program is a 60-hour self-study program. The cost of the program is $1,165. The cost
includes everything: registration, shipping, materials, testing, and the first year's certification fee.

47
Materials include: Morningstar’s Guide to Mutual Funds, Selected Readings I, Selected Readings
II, Student Manual with sample questions, and the Mutual Fund & Variable Annuity Directory.

The final exam is given at Thomson Prometric Centers which are located throughout the country.
The exam is administered by the NASD

Certified Annuity Specialist (CAS) www.ifcs.com

The Certified Mutual Fund Specialist Program gives students - most often, financial services
employees - a broad overview of the mutual fund business. The program looks at the industry, its
history, regulation, providers, structure, operations, distribution, marketing and the popularity of
open-end mutual funds in the United States. National Investment Company Service Association
(NICSA) offers the program, which is made up of three courses: Introduction to the Mutual Fund
Industry Course, Mutual Fund Operations Course, Mutual Fund Distribution & Marketing Course

Completion of the three courses earns the student a certificate from NICSA in the specific area of
study. The program is based on "Purely American Invention: The U.S. Open-End Mutual Fund
Industry," a book by Lee Gremillion. Dr. Gremillion is a former partner with
PricewaterhouseCoopers LLP, where he led the Investment Management and Securities
Operations consulting group in the Midwest. Dr. Gremillion is now a professor at Capella
University. He also holds a doctoral degree from Harvard. Like the book, the interactive courses
help students understanding the breadth of mutual fund operations and their context.

Certified in Asset Allocation (BCCA) www.icfs.com

Board Certified in Asset Allocation (BCAA) is the only designation of its kind. It was created to
address the frustration of investors, brokers, and advisors who now understand the value of a
properly-constructed portfolio. Yet, the general public remains leery of relying on in-house
solutions and proprietary products that lack objectivity.
The 60-hour self-study BCAA program is appealing for several reasons: (1) the materials are
completely neutral and have no hidden agenda, (2) content is comprehensive, logical, and easy to
understand, (3) the information gained can be applied to all of your clients, (4) it is the only asset
allocation certification program in the country, and (5) the designation will set you apart from your
present and future competition.

The three final exams are given at Thomson Prometric Centers and administered by the NASD.
Students must also complete two open-book case studies. The $1,165 BCAA program is fully
accredited and counts as four units toward the MSFS graduate degree.

Certified in Securities www.icfs.com

Many investors suffered huge losses over the past few years. Everyone is looking for answers and
guidance. Peace of mind for investors comes from finding a knowledgeable advisor they trust in
and believe. The Board Certified in Securities (BCS) designee is just such a person.

The BCS program is a 60-hour self-study program. The cost includes everything: registration,
shipping, materials, testing, and the first year's certification fee. The three final exams are given at
Thomson Prometric Centers and administered by the NASD. Students must also complete two
open-book case studies. The BCS program is fully accredited and counts as four units toward the
MSFS graduate degree.

There are only two designations for those who specialize in individual securities: Chartered
Financial Analyst (CFA) and Board Certified in Securities (BCS). The CFA program is designed

48
for portfolio managers, not advisors who deal with the general public. The CFA program costs
several thousand dollars and takes several years to complete. The BCS program was developed for
the practitioner who has a personal relationship with an individual or company. The BCS materials
can be completed in 60 hours or less and cost just $1,165.

Certified Mutual Fund Specialist

The Certified Mutual Fund Specialist Program gives students - most often, financial services
employees - a broad overview of the mutual fund business. The program looks at the industry, its
history, regulation, providers, structure, operations, distribution, marketing and the popularity of
open-end mutual funds in the United States. National Investment Company Service Association
(NICSA) offers the program, which is made up of three courses: Introduction to the Mutual Fund
Industry Course, Mutual Fund Operations Course, Mutual Fund Distribution & Marketing Course

Completion of the three courses earns the student a certificate from NICSA in the specific area of
study. The program is based on "Purely American Invention: The U.S. Open-End Mutual Fund
Industry," a book by Lee Gremillion. Dr. Gremillion is a former partner with
PricewaterhouseCoopers LLP, where he led the Investment Management and Securities
Operations consulting group in the Midwest. Dr. Gremillion is now a professor at Capella
University. He also holds a doctoral degree from Harvard. Like the book, the interactive courses
help students understanding the breadth of mutual fund operations and their context.

Personal Financial Planning

Certified Financial Planner (CFP)


www.cfp.net

Why has the CERTIFIED FINANCIAL PLANNER™ certification become so sought after by
consumers and the financial planners who serve them? The answer is simple. The public is
looking for a planner who has demonstrated a commitment to competency, and financial
professionals want an established certification that sets them apart in a globally expanding
financial planning profession. CFP Board research shows consumers increasingly rely on
credentials when selecting a financial adviser. You will be equipped to provide truly personalized
services to clients and maintain high levels of financial planning professionalism and expertise.

The CFP®, CERTIFIED FINANCIAL PLANNER™ and certification marks are the most
recognized financial planning marks in the world. Unlike an educational designation offered by a
college or university, CFP certification is based on independently established public interest
standards. As a result, it prepares you for a career-long commitment to meeting the ever-changing
needs of your clients. Examples of the benefits of CFP certification include:

Chartered Financial Consultant (ChFC)

A financial planning designation for the insurance industry awarded by the American College of
Bryn Mawr. ChFCs must meet experience requirements and pass exams covering finance and
investing. They must have at least three years of experience in the financial industry, and have
studied and passed an examination on the fundamentals of financial planning, including income
tax, insurance, investment and estate planning.

49
Accredited Financial Counselor
Accredited Financial Counselors have certified skills to assist individuals and families in the
complex process of financial decision making. The exam is a two part exam that is administered
by the Association for Financial Counseling and Planning Education. www.afcpe.org

Corporate Finance

Certified in Financial Management (CFM)


website www.imanet.org

The Certified Financial Manager (CFM) certification provides Members who are involved with
corporate cash management, financing and investment decisions, and risk management with a
means for further demonstrating an expanded skill set. This exam provides an in-depth measure of
competence in areas such as financial statement analysis, working capital policy, capital structure,
valuation issues, and risk management.

Treasury Management
Certified Cash Manager (CCM)
http://www.afpnj.org/ccm.html not the official website

The Association for Financial Professionals, AFP, has established the Certified Cash Manager
(CCM) designation to help provide it's members and the business community with a body of
knowledge necessary for functioning in the ever changing global economy which exists today.
Each June the AFP offers a standardized exam covering the various disciplines of treasury
management. The CCM designation is awarded to those individuals who display a comprehensive
understanding of this material. This material includes the banking and payment systems,
disbursements, collections and cash concentration, bank relationship management, short-term
borrowing and investing, cash flow forecasting, and international cash management. The exam is
administered by the AFP in various cities across the United States. For further information
concerning this program, please see the AFP's certification page.

Certified Treasury Professional (CTF) and Certified Treasury Associate (CTA)

www.afponline.org

The CTP sets the standard in the financial profession and is a symbol of excellence. It signifies that
an individual has demonstrated the knowledge and skills required to perform competently in today's
complex treasury environment. As a result of recent changes in corporate America, treasury and
finance professionals face more public scrutiny than ever before. They must now demonstrate a
mastery of risk management, capital structure, mergers and acquisitions and corporate
governance, in addition to cash management topics. With the CTP designation, AFP recognizes
the significant increase in knowledge required of treasury professionals on the job.

50
Valuation
Accredited Senior Appraiser (ASA)
www.appraisers.org

Each accredited member of the American Society of Appraisers has earned a professional
designation in one or more specialized areas of appraisal. To receive the accreditation, the
appraiser must pass intensive courses/written examinations, submit representative appraisal
reports, an appraisal experience log and evidence of a college degree or its equivalent.

Every accredited appraiser must start his or her ASA membership as a Candidate member. In order
to be accepted for Candidate membership, the prospective Candidate must be interviewed and
approved by his or her local chapter. Subsequently, each Candidate must pass ASA's Ethics
Examination and an examination on the Uniform Standards of Professional Appraisal Practice
(USPAP) within a specified period of time. (The Uniform Standards are published each year by
The Appraisal Foundation, authorized by Congress as the source of appraisal standards and
appraiser qualifications.)

The Candidate's technical appraisal proficiency and understanding of the fundamentals of


appraisal ethics, principles and concepts are evaluated by intensive written examinations. Copies
of the Candidate's appraisal reports are reviewed and must meet professional criteria. When the
Candidate has met all these requirements and has gained the necessary experience, he or she may
apply for advancement to Accredited Member or Accredited Senior Appraiser status.

Certified Business Appraiser (CBA)


www.go-iba.org

The Certified Business Appraiser Accreditation Program is one of the most important components
of the Institute's professional development curriculum. Members who meet established criteria are
awarded the prestigious professional designation of Certified Business Appraiser (CBA). This
distinction, which denotes a level of competence attained only by the most accomplished business
appraisers, grants its recipients special recognition and prestige among fellow appraisers, the
courts and throughout the business appraisal community.

Accreditation as a Certified Business Appraiser (CBA) is available to those members of The


Institute of Business Appraisers, Inc. who are able to demonstrate that they have attained a high
level of professional competence and conduct. The CBA accreditation designates the appraiser as
a true professional in his/her field, worthy of the admiration of fellow appraisers and deserving the
fullest confidence of clients and potential clients.

Applicant must pass a comprehensive written examination on current business valuation theory
and practice. Content and administration of the examination shall be as described in the
application handbook, under heading "Written Examination." Arrangements may be made for the
CBA examination to be supervised by a private proctor. Please contact IBA headquarters for
qualifying details.

51
Accredited Valuation Analyst (AVA)
www.nacva.com

The National Association of Certified Valuation Analysts (NACVA) trains and certifies
Accredited Valuation Analysts (AVA) to perform business valuations as a service to both the
consulting community and the users of their services. Through training and rigorous testing,
AVAs demonstrate they are qualified to provide capable and professionally executed valuation
services. NACVA requires training as a prerequisite to certification to assure that practitioners
have the knowledge and understanding necessary to perform competent services, and to assure a
level of consistency and continuity in their work product. Users of valuation services benefit by
having greater confidence that the service they receive is professional in quality, adheres to
industry standards of practice, and meets a level of expertise the Association deems credible and
worthy of one of its certified members. AVAs must hold a business degree from an accredited
institution of higher education and demonstrate substantial business valuation experience, among
other requirements.

Certified Forensic Financial Analyst


www.nacva.com

The Certified Forensic Financial Analyst (CFFA) credential is designed to demonstrate to


the legal community that the designee possesses a level of experience and knowledge
deemed acceptable by the National Association of Certified Valuation Analysts (NACVA)
and the Forensic Institute for Financial Experts (FI) to provide competent and
professional forensic financial litigation support. The training includes economics,
statistics, and calculating damages, followed by a five-day practicum in which participants
learn about commercial damages and participate in a damages study, which takes them
through the processes of deposition, mediation, and jury trial.

Credit Analysis

The Credit Analysis Certifications are all offered by the National Association of Credit
Management www.nacm.org

Credit Business Associate (CBA)

The Credit Business Associate (CBA) is an academic-based designation which signals mastery of
three business-credit related disciplines: basic financial accounting, business credit principles and
introductory financial statement analysis. There is no minimum work experience requirement for
this designation level and the course work needed to qualify for this designation can be obtained
through colleges, local NACM Affiliated Association programs, self-study or nationally-
sponsored programs.

Credit Business Fellow (CBF)

The Credit Business Fellow (CBF) Designation is an academic and participation-based


designation which illustrates that achievers are knowledgeable about and have contributed to the
field of business credit by first having earned the CBA designation as well as having completed
additional course work. The CBF signals competence in intermediate financial analysis and
business and credit law. In addition to completing course work, CBF designation applicants must
show evidence of 75 NACM Career Roadmap points.

52
Certified Credit Executive (CCE)

The Certified Credit Executive® (CCE®) is NACM's executive level designation which endorses
its achievers as capable of managing the credit function at an executive level. Candidates must
pass a rigorous, four hour exam which tests application skills in the areas of accounting, finance,
domestic and international credit concepts, management and law.

Public Finance

Certified Government Financial Manager (CGFM)


www.agacgfm.org

Since its inception in 1994, the CGFM has become the standard by which government financial
management professionals are measured. Its education, experience and ethics requirements have
served to elevate the most seasoned financial professionals. The CGFM program spans the public,
private and academic sectors in the United States and abroad. It recognizes the unique skills and
experience of government financial management professionals. It identifies those who have
knowledge in many functional areas and can apply it in a government setting.

Certified Public Finance Officer (CPFO)

The Certified Public Finance Officers Program (Certification Program) of the Government Finance
Officers Association of the United States and Canada (GFOA) is a broad educational self-study
program designed to verify knowledge in the disciplines of government finance. The Certification
Program is governed by the Council on Certification. Technical and administrative support for the
program is provided by Radford University's Governmental and Nonprofit Assistance Center.
To earn the designation of Certified Public Finance Officer (CPFO), candidates must pass a series
of five examinations covering the major disciplines of public finance.

Insurance
Certified Risk Manager (CRM)
www.scic.com

The Certified Risk Managers International (CRM) designation demonstrates that you are
knowledgeable in all areas of managing risks, hazards, and exposures. The five courses give you
in-depth knowledge about today’s highest priorities – identifying, analyzing, controlling,
financing, and administering operational risks – as well as political risks, catastrophic loss
exposures, third-party exposures, fiduciary exposures, employee injury exposures, juridical risks,
legal risks, and more – whether insurable or not. The skills you learn will make you more
proactive and valuable to your organization in discovering how risks can interrupt the flow of
earnings and how to protect against it.

Certified Insurance Counselor (CIC)


www.scic.com

The Certified Insurance Counselors (CIC) Program has been the insurance industry's premier,
proven source for practical, real-world education since 1969. Designed to cover important aspects
of the insurance field, each of five institutes is 20 hours of instruction

Fellow, Life Management Institute (FLMI)

53
Chartered Property Cacualty Underwriter (CPCU)

Registered Health Underwriter

Chartered Life Underwriter

Real Estate

ABR – Accredited Buyer Representative is a designation from the Real Estate Buyer’s Agent
Council that trains experienced REALTORS® in specifically representing the real estate
consumer.

The ABR designation is awarded to real estate practitioners who complete a comprehensive two-
day REBAC course in buyer representation, achieve a passing grade on the written examination,
demonstrate practical experience by completing and closing five real estate transactions in which
the candidate functioned as a buyer's representative and who maintain a membership in good
standing with the National Association of REALTORS® and the Real Estate BUYER'S AGENT
Council. Web page at www.rebac.net

CPM – Certified Property Manager designation is the oldest and most prestigious
achievement in property management. It is awarded to property managers whose
experience, education and ethical standards warrant such distinction. CPMs are experts
in managing apartments, office buildings, commercial centers and homeowners’
associations and are informed on tax laws insurance regulations, and critical investment
factors.

To become a CPM, one must complete a combination of core and elective requirements.
One will need 260 points --160 required points and 100 elective points - in addition to
meeting some other criteria unrelated to the point system.
Institute of Real Estate Management (IREM). Web page at www.irem.org.

54
Other

Certified Business Manager (CBM) www.apbm.org

The Certified Business Manager (CBM) program provides a generalized business education at the
graduate level, covering essential subject matter useful to business practitioners. The CBM
program is designed to complement existing specialty certifications, to validate a business
professional’s practical business experience and knowledge, and to act as an efficient preparatory
tool for a graduate business program.

The credential takes a comprehensive MBA curriculum and presents it in a certification format,
teaching and measuring the skills and knowledge required to become a successful manager.

The CBM Exam covers ten essential areas/modules of business (i.e. Accounting, Finance,
Information Technology, Marketing Management, Human Resource Management, International
Business and Quality Management) and their cross-functional applications in three, three hour-
long parts. Each part contains 150 multiple-choice questions.

Registered Employee Benefits Specialist (REBS)

The REBC designation signifies that an individual working in the employee benefits field has
successfully completed a comprehensive five-course program. The number of electives provides
an opportunity for individuals to specialize within the broader employee benefits field. The
designation is especially suitable for professionals who are selling or servicing the group
insurance, health insurance, retirement planning, and incentive compensation markets. More than
1,250 REBCs have been awarded, including those granted by the College since assuming
ownership of the designation from the National Association of Health Underwriters.

The designation requires five courses - three which are required and two electives. Students also
must meet specified experience requirements, maintain ethical standards, and agree to comply
with both The American College's Code of Ethics and Procedures and applicable continuing
education requirements.

55
CHAPTER 1

An Overview of Financial
Management

56
Organizational Forms
Selecting an organizational form is one of the first decisions you
must make if when you decide to start a company

Correct Selection of the Organizational form is important because:

a. The amount of taxes paid depends on the organizational


form selected.

b. The owners exposure to risk depends on it.

c. The cost of establishing various organizational forms


differs.

d. The ability to raise capital depends on the organizational


form selected.

57
Corporate Characteristics

1. Limited Liability –

refers to the amount of money the owners can lose if the


firm fails.

When a firm has limited liability, the most the owners


can lose is the amount of money they invest in the
firm.

When a firm does not have limited liablility, the


owners can lose what they have invested in the firm
and all of their personal wealth as well.

2. Continuity of Life

refers to what happens to the firm when one of the owners


dies.

When a firm has continuity of life and one of the


owners dies, the firm continues as if nothing happened.

When a firm does not have continuity of life and one of


the owners dies, the firm also dies.

58
3. Centralized Management

refers to who can make decisions on behalf of the firm.

When a firm has centralized management, only a select


group of owners can make decisions on behalf of the
firm (this group is called the board of directors).

When a firm does not have centralized management,


any owner can make decisions on behalf of the firm.

4. Free Transferability of Interest

refers to the permission owners must obtain before selling


their shares in the firm.

When a firm has free transferability of interest, owners


can sell their interest in the firm without obtaining the
permission of anyone else.

When a firm does not have free transferability of


interest, owners must obtain the permission from
someone else prior to selling their interest in the firm.

All four of these characteristics are desireable.

C-Corporations & S-Corporations have all four characteristics.


Limited Liability Companies have three characteristics and the
fourth characteristic on a limited basis. Sole proprietorships and
partnerships can have no more than two of the characteristics (and
these two include free transferability of interest and centralized
management).

59
Taxation

There are two basic types of taxation applied to


organizations.

1.) Partnership (or Pass Through Taxation)

2.) Corporate (or double taxation)

60
Partnership (or Pass Through Taxation)

The organization is not taxed. Earnings are passed directly through


to the owners and are taxed at the personal tax rate only.

Given:
Firm Earnings $100,000
When appropriate, corporate Tax Rate = 25%
When Appropriate Personal Tax Rate = 34%
All earnings of the firm are paid out to the owners

Because there are no corporate taxes under the pass through


taxation system, the net earnings paid to stockholders is the
full $100,000.

Step 1: Calculate the Total Tax

Earnings paid to owners $100,000


Personal Tax Rate X .34
Total Taxes Paid = $34,000

Step 2: Compute the Total Proceeds to the Owners

Earnings paid to owners $100,000


Less personal taxes paid $34,000
Net Proceeds to Owners $66,000

61
Corporate Taxation (or Double Taxation)

The corporation is taxed on its earnings. When the earnings are paid to the
owners, they are also taxed at the personal level.

Given:
Firm Earnings $100,000
When appropriate, corporate Tax Rate = 25%
When Appropriate Personal Tax Rate = 34%
All earnings of the firm are paid out to the owners

Step 1: Calculate the Corporate Taxes Paid

Earnings $100,000
Corporate Tax Rate X .25
Corporate Taxes Paid = $ 25,000

Step 2: Calculate the net earnings paid to the owners

Earnings $100,000
less Corporate Taxes Paid - $ 25,000
Earnings paid to owners = $ 75,000

Step 3: Calculate the Personal Taxes Paid

Earnings paid to owners $ 75,000


Personal Tax Rate X .34
Personal Taxes Paid = $ 25,500

Step 4: Calculate the Total Taxes Paid

Total Taxes = Corporate Tax Paid + Personal Tax Paid


= $25,000 + $25,500
= $50,500

Step 5: Compute the Net Proceeds to the Owners

Firm Earnings $100,000


Less Total Taxes Paid $50,500
Net Proceeds to the Owners $49,500

62
C-corporations have all four characteristics. However, they are
subject to double taxation.

There are two things that a C-Corporation can do to minimize its


tax disadvantage:

1. Retain Earnings within the firm.


When a company earns money, it can do one of two
things with it.
a. It can retain the money within the firm (Retained
Earnings)
b. It can give the money to the owners of the firm in the
form of a dividend.
Recall that the owners of the firm must pay taxes on the
money they receive from a C-corporation. However, if
the firm does not pay dividends the owners of the
company do not have to have to pay taxes.

2. Borrow Money
Because interest payments are a tax deductible expense
for the corporation, the effects of double taxation can be
reduced by borrowing money

3. Tax Inversion
Tax Inversions involve a corporation merging with
another foreign corporation to avoid U.S. taxes.

63
S-Corporations

1. Have all four characteristics

2. Are subject to pass-through taxation

3. Earnings are subject to personal level taxation whether


distributed to stockholders or retained within the firm.

4. Are subject to the following restrictions

a. May have no more than 100 stockholders


b. May have no Foreign Stockholders (US citizens and
permanent residents only)
c. May have no Corporate Stockholders
d. May have only one class of stock

64
Limited Liability Company (LLC)

1. The newest organizational form.

a. First created by the state of Wyoming in 1977.


b. Today they are allowed in every state
c. Became legal in Hawaii in 1997
(Hawaii was the last state in the Union to adopt a LLC
Statute)

2. Has the following Characteristics


a. limited liability
b. centralized management
c. continuity of life
d. Are automatically terminated if more than 50 percent of
ownership changes in any 12 month period.

3. Can select corporate taxation or pass through taxation

4. May be problematic when conducting business across state


lines because the statutes are not nationally driven, but rather
created by the individual states.

65
Agency Problems
An Agency Problem Occurs when two people work together
but their goals are not necessarily the same.

I. Agency Problem between Stockholders (owners) and


Managers

1. The Owners of the company would like the managers to


maximize their wealth.

2. The managers of the company are more interested in


maximizing their own wealth.

So there is a conflict between the two parties


We call this conflict an agency problem

3. Things managers can do to maximize their own wealth at


the expense of the owners.

a. Take Perks
b. Expand the firm to a large size
c. Reduce the risk of the firm
d. Oppose takeover threats
e. Keep a failing company alive longer than is optimal

4. Methods to reduce the conflict between owners and


managers

a. Provide managers more of their compensation in the form


of stock
b. Monitor the managers through audits
c. Increase the threat of firing
d. Increase the threat of takeover

66
II. Agency Problems between the owners and the bondholders
(lenders).

1. Bondholders (lenders) have a fixed claim.

So regardless of how well the firm performs, bonholders get


the same amount of money. So bondholders would like the
firm to minimize the amount of risk that it takes.

2. The owners of the firm have a residual claim.

They get whatever is left over after the bondholders have


Been paid. So the owners have a motivation to take on risk
in order to earn as much as they can.

3. Methods owners can maximize their wealth at the expense of


bondholders

a. Once the company has locked in its cost of debt, the


stockholders have a motivation to increase the risk of the
compaies investments, and consequently the expected
return.

Stockholders benefit because they are getting a bargain on


their cost of debt.

Bondholders are upset because the probablility of


bankruptcy has increased.

67
4. Methods to reduce the conflict between owners and
bondholders.

a. Protective covenants limit the activities of the firm thereby


protecting the Bondholders For example: restrictions on
investments, restricting dividends, making payments
directly to suppliers, and utilization of an Escrow
Company

b. Register Ownership to limit tranfer.

Car ownership is registered with the state. By


indicating on the title that there is a leinholder, transfer
can be limited. That is: the motor vehicle department
will not allow the owner to transfer ownership without
first paying off the leinholder.

c. Personal Guarantees

Recall that many organizational forms have limited


liability. Sometimes banks will require the owners to
sign a personal guarantee for loans to the firm. That is:
if the firm does not pay off its loan, the owners will be
personally held liable for the debt. In this case, the
owners of the firm are selectively giving up their
limited liability.

d. Require Collateral

Generally banks will require collateral. In this case, the


company must pledge some specific assets to the bank.
These assets will be given to the bank in the event that the
company cannot pay off its loans.

68
Business Application Forms for the State of Hawaii

To start a business in Hawaii you will work with the


Department of Commerce and Consumer Affairs.

http://hawaii.gov/dcca/

Step 1: To create a limited liability company, you will


need to file a State of Hawaii LLC-1 form.

http://hawaii.gov/dcca/breg/registration/dllc/

If you wish to establish a different organizational


form you can obtain the appropriate form from:

http://hawaii.gov/dcca/breg/registration/registratio
n-form-info.html

Once you have your organizational form established, the


State of Hawaii has established a website with a wizard
that will walk you through the other steps necessary to
start a business in the State of Hawaii. You can access the
website at the following address:

https://hbe.ehawaii.gov/BizEx/home.eb

If you are starting a business in Hawaii you will need to


file a BB-1 form.

http://portal.ehawaii.gov/help-center-questions/file-
bb1.html

69
70
71
72
73
74
75
76
77
Chapter 1 Homework Assignment and Solution

78
Chapter 1
Homework Assignment

1. List and describe the four corporate characteristics.

2. Give an example of pass-through taxation and double taxation.

3. Which characteristics does a sole proprietorship or partnership have and how are
they taxed?

4. Which of the corporate characteristics does an S-corporation have and


how are they taxed?

5. What limitations are S-corporations subject to?

6. Which corporate characteristics does a Limited Liability Company Have


and how are they taxed?

7. Discuss the agency problem between the firm’s owners and the firm’s
managers. How can the firms’ owners minimize this agency problem?

79
Chapter 1 Homework Solutions

1. List and describe the four corporate characteristics

a. Limited Liability – refers to how much money can potentially be lost by the
owners of the company if the company fails. If the company has limited
liability the loss is limited to the amount of money invested in the firm. If the
company does not have limited liability, the owners can lose their investment
in the firm and all of their personal wealth as well.

b. Continuity of Life – refers to what happens when one of the owners dies. If a
firm has Continuity of life, when one of the owners dies the company
continues without interruption. If the firm does not have continuity of life,
when one of the owners dies, the business is dissolved.

c. Centralization Management – refers to decision making. When a company


has centralized management, the only the board of directors can make
decisions for the firm. When a company does not have centralized
management, any owner can make decisions on behalf of the firm.

d. Free transferability of Interest – refers to the ability of an owner to sell his


shares without permission. If a company has free transferability of interest, an
owner can sell his interest in the firm without getting permission. If a
company does not have free transferability of interest, an owner must obtain
permission before selling his interest in the firm.

80
2. Give and example of pass-through taxation and double taxation.

Example of Pass-Through Taxation

Given:
Corporate Earnings $200,000
Corporate Tax Rate 30%
Personal Tax Rate 36%
All earnings of the corporation are paid out to the owners

Since there is no corporate taxes under the pass through taxation system, the net
earnings paid to the stockholders is the full $200,000.

Step 1: Calculate the Total Tax

Earning paid to owners $200,000


Personal Tax Rate x .36
Total Tax Paid = $72,000

Step 2: Calculate the Proceeds to the Owners


Company Earnings $200,000
Total Taxes Paid - $72,000
Net Proceeds to Owners = $128,000

81
Example of Double Taxation (using the same information from the previous
example).

Step 1: Calculate the Corporate Taxes Paid

Earnings $200,000
Corporate Tax Rate x .30
Corporate Taxes Paid = $60,000

Step 2: Calculate the net earnings paid to the owners

Earnings $200,000
Less Corporate Taxes Paid - 60,000
Earnings paid to the owners = $140,000

Step 3: Calculate the Personal Taxes Paid

Earnings paid to owners $ 140,000


Personal Tax Rate x .36
Personal Taxes Paid = $50,400

Step 4: Calculate the Total Taxes Paid

Corporate Taxes Paid $60,000


Personal Taxes Paid $50,400
Total Taxes Paid $110,400

Step 5: Calculate the net proceeds to the owners


Earnings of the Company $200,000
Total Taxes Paid - $110,400
Net Proceeds to Owners = $89,600

3. Which characteristics does a sole proprietorship or partnership have and how are
they taxed?

Sole proprietorships have centralized management and free transferability


of interest. They are subject to pass-through taxation.

4. Which of the corporate characteristics does an S-corporation have and


how are they taxed?

S-corporations have all four of the corporate characteristics and are


subject to pass-through taxation.

82
5. What limitations are S-corporations subject to?

a. May have no more than 100 stockholders


b. May have no Foreign Stockholders (US citizens and
permanent residents only)
e. May have no Corporate Stockholders
f. May have only one class of stock

6. Which corporate characteristics does a Limited Liability Company Have


and how are they taxed?

An LLC has limited liability, continuity of life and centralized management.


However free transferability is limited such that no more than 50 percent of the
ownership interest can change hands in any 12 month period.

7. Discuss the agency problem between the firm’s owners and the firm’s
managers. How can the firms’ owners minimize this agency problem?

An Agency Problem that occurs when two people who work together but have
differing goals. An agency problem occurs between the stockholders of the firm
and the owners of the firm. The owners of the company would like the managers
to maximize their wealth. The managers of the company are more interested in
maximizing their own wealth. So here is a conflict between the two parties and
we called this conflict an agency problem.

1. Things managers can to maximize their own wealth at the expense of the
owners.

a. Take Perks
b. Expand the firm to a large scale
c. Reduce the risk of the firm
d. Oppose takeover threats
e. Keep a failing company alive longer than is optimal

2. Methods to reduce the conflict between owners and managers.

a. Provide managers more of their compensation in the form of stock.


b. Monitor the managers through audits
c. Increase the threat of firing
d. Increase the threat of takeover.

83
CHAPTER 2

Financial Statements, Cash


Flows and Taxes

84
Chapter 2: Financial Statements,
Cash Flows and Taxes

In this chapter we will discuss Financial Statements and Taxes.

There are many issues associated with Financial Statements and


taxes: This is intended only as an introduction.

Goal #1: To make you aware of the various financial statements


Goal #2: To show you how various financial statements fit
together
Goal #3: To show you how to evaluate financial statements
Goal #4: Introduction to Taxation

85
Four Basic Financial Statements:
1. Balance Sheet:

Shows you everything you own, everything you owe, and the
net worth of your company on a specific day.

In most instances, it does not tell you what you should be


willing to pay to purchase a business.

2. Income Statement:

Shows you the performance of your company over some time


Period.

Shows income, expenses and profits

Typically monthly, quarterly and annual

3. Statement of Cash Flows

Shows you the sources and uses of cash over some time
period.

One of two links between the income statement and


balance sheet.

4. Statement of Retained Earnings

Shows you what was done with the earnings of the company.

One of two links between the income statement and balance


sheet.

86
Finacial Statement Example This is for a Taxi Cab Business. For simplicity, assume that the cars are magical and never wear out.

Balance Sheet

Balance Sheet
Balance Sheet

Balance Sheet
Purchase Taxi

Work for one

Work for one


Statement at

Statement at

Another Car
12-31-2014

Purchasing
12/31/2013
Investment

Retained

Retained
Earnings

Earnings

Dividend
$10,000
after the
Cab for
$9,000

Yr End

yr end
Initial

Year

After
year
Action
Balance Sheet 1-Jan-13 2-Jan-13 12/31/2013 12/31/2014
Assets
Cash 10000 1000 11000 31000 22000 12000
Cars 0 9000 9000 9000 18000 18000
Total Assets 10000 10000 20000 40000 40000 30000
Liabilities and Equity
Common Stock 10000 10000 10000 10000 10000 10000
Retained Earnings 0 0 10000 30000 30000 20000

Total Liabilities and Equity 10000 10000 20000 40000 40000 30000
Income Statement
Sales 50000 60000
other Expenses Including Tax and Interest -40000 -40000
Net Income 10000 20000
Statement of Retained Earnings
Retained Earnings Previous Year 0 10000
+ Net Income for This year 10000 20000
- Dividends paid 0 0
= New Retained Earnings 10000 30000
Cash Flow Statement
Net Income 10000 20000
+- Adjustments 0 0
= Change in Cash Position 10000 20000
Old Cash 1000 11000
+ Change in Cash Position 10000 20000
= New Cash 11000 31000

87
Hilo Clothing Market
Balance Sheet
12-31-2012 12-31-2013
Assets
Cash $100,000 $193,200
Accounts Receivable $75,000 $175,000
Inventories $150,000 $175,000
Total Current Assets $325,000 $543,200
Gross Fixed Assets $800,000 $1,000,000
Less Accumulated Deprecation $100,000 $225,000
Net Fixed Assets $700,000 $775,000
Total Assets $1025,000 $1,318,200

Liabilities and Equity


Accounts Payable $100,000 $133,000
Notes Payable $200,000 $225,000
Accruals $50,000 $40,000
Total Current Liabilities $350,000 $398,000
Long Term Debt $150,000 $150,000
Total Liabilities $500,000 $548,000
Common Stock $200,000 $200,000
Additional Paid In Capital $100,000 $100,000
Retained Earnings $225,000 $470,200
Total Equity $525,000 $770,200
Total Liabilities and Equity $1,025,000 $1,318,200

*100,000 shares of stock are issued and outstanding

88
Hilo Clothing Market
Income Statement
Year Ending Year Ending
12-31- 2012 12-31-2013

Sales $2,100,000 $2,300,000


Cost of Goods Sold (COGS) $1,200,000 $1,250,000
Other Expenses $400,000 $450,000
Depreciation $100,000 $125,000
Total Operating Costs $1,700,000 $1,825,000
EBIT $400,000 $475,000
Interest Expense $40,000 $48,000
EBT $360,000 $427,000
Taxes (40% rate) $144,000 $170,800
Net Income $216,000 $256,200

* Note: The firm paid a total of $11,000 in dividends in 2013


* Note: The stock price at the end of 2013 is $36.15 per share

89
Hilo Clothing Market
Statement of Retained Earnings for Year Ending 2013

Balance of Retained Earnings 12-31-2012 $225,000


+ Net Income 2013 $256,200
- Dividends Paid in 2013 $11,000
Balance of Retained Earnings 12-31-2013 $470,200

Hilo Clothing Market


Statement of Cash Flows For Year Ending 12-31-2013

Step 1: Compute the Change in Cash Position

Net Income $256,200


+ Depreciation $125,000
Increases in Liabilities
+ Increases in Accounts Payable (133,000-$100,000) $33,000
+ Increases in Notes Payable (225,000-200,000) $25,000
+ Increases in Accruals ($40,000-$50,000) -$10,000
+ Increases in Long-Term Debt ($150,000-$150,000) $0
Increases in Assets
- Increases in Inventory (175,000-150,000) $25,000
- Increases in Accounts Receivable ($175,000-$75,000) $100,000
- Increases in other Current Assets $0
- Increases in Gross Fixed Assets ($1,000,000-$800,000) $200,000
Cash Paid to and Received from Stockholders
+ New Investment by Stockholders $0
- Dividends Paid $11,000
= Change in Cash Position $93,200

Step 2: Compute the New Cash Balance

Cash Balance 12-31-2012 $100,000


+ Change in Cash Position 2013 $93,200
= Cash Balance 12-31-2013 $193,200

90
Evaluation of Financial Statements:
Ratio Analysis

Assorted Ratios can be used to analyze the firms performance

Market Value of Equity (Market Capitalization)

The market value of equity tells you how much money it would cost
to purchase the entire equity stake in the company.

𝑀𝑀𝑀𝑀𝑀𝑀𝑀𝑀𝑀𝑀𝑀𝑀 𝑉𝑉𝑉𝑉𝑉𝑉𝑉𝑉𝑉𝑉 𝑜𝑜𝑜𝑜 𝐸𝐸𝐸𝐸𝐸𝐸𝐸𝐸𝐸𝐸𝐸𝐸 = 𝑆𝑆ℎ𝑎𝑎𝑎𝑎𝑎𝑎𝑎𝑎 𝑂𝑂𝑂𝑂𝑂𝑂𝑂𝑂𝑂𝑂𝑂𝑂𝑂𝑂𝑂𝑂𝑂𝑂𝑂𝑂𝑂𝑂 𝑋𝑋 𝑃𝑃𝑃𝑃𝑃𝑃𝑃𝑃𝑃𝑃 𝑃𝑃𝑃𝑃𝑃𝑃 𝑆𝑆ℎ𝑎𝑎𝑎𝑎𝑎𝑎

= 100,000 X 36.15

= $3,615,000

Price to Earnings Ratio (PE)

The P/E ratio tells us that if earnings do not change from their current levels,
how long it will take in years before the firm will earn back the amount of
money that we paid for the stock.

𝑀𝑀𝑀𝑀𝑀𝑀𝑀𝑀𝑀𝑀𝑀𝑀 𝑉𝑉𝑉𝑉𝑉𝑉𝑉𝑉𝑉𝑉 𝑜𝑜𝑜𝑜 𝐸𝐸𝐸𝐸𝐸𝐸𝐸𝐸𝐸𝐸𝐸𝐸 𝑂𝑂𝑂𝑂𝑂𝑂𝑂𝑂𝑂𝑂𝑂𝑂ℎ𝑖𝑖𝑖𝑖


𝑃𝑃𝑃𝑃𝑃𝑃𝑃𝑃𝑃𝑃 𝑡𝑡𝑡𝑡 𝐸𝐸𝑎𝑎𝑟𝑟𝑟𝑟𝑟𝑟𝑟𝑟𝑟𝑟𝑟𝑟 𝑅𝑅𝑅𝑅𝑅𝑅𝑅𝑅𝑅𝑅 =
𝑁𝑁𝑁𝑁𝑁𝑁 𝐼𝐼𝐼𝐼𝐼𝐼𝐼𝐼𝐼𝐼𝐼𝐼
3,615,000
=
256,200

= 14.11 times

The P/E ratio tells us that if earnings do not change from their current levels,
it will take 14.11 years before the firm will earn back the amount of money
that we paid for the stock.

91
Total Asset Turnover

Tells you how many dollars of sales the company produced for
each dollar of assets.

𝑆𝑆𝑆𝑆𝑆𝑆𝑆𝑆𝑆𝑆
𝑇𝑇𝑇𝑇𝑇𝑇𝑇𝑇𝑇𝑇 𝐴𝐴𝐴𝐴𝐴𝐴𝐴𝐴𝐴𝐴 𝑇𝑇𝑇𝑇𝑇𝑇𝑇𝑇𝑇𝑇𝑇𝑇𝑇𝑇𝑇𝑇 =
𝑇𝑇𝑇𝑇𝑇𝑇𝑇𝑇𝑇𝑇 𝐴𝐴𝐴𝐴𝐴𝐴𝐴𝐴𝐴𝐴𝐴𝐴
2,300,000
𝑇𝑇𝑇𝑇𝑇𝑇𝑇𝑇𝑇𝑇 𝐴𝐴𝐴𝐴𝐴𝐴𝐴𝐴𝐴𝐴 𝑇𝑇𝑇𝑇𝑇𝑇𝑇𝑇𝑇𝑇𝑇𝑇𝑇𝑇𝑇𝑇 =
1,318,200

𝑇𝑇𝑇𝑇𝑇𝑇𝑇𝑇𝑇𝑇 𝐴𝐴𝐴𝐴𝐴𝐴𝐴𝐴𝐴𝐴 𝑇𝑇𝑇𝑇𝑇𝑇𝑇𝑇𝑇𝑇𝑇𝑇𝑇𝑇𝑇𝑇 = 1.748

So in this case the company had $1.748 of sales for each dollar of
assets that it had.

Inventory Turnover

Tells you how many dollars of sales the company produced for
each dollar of inventory.

𝑆𝑆𝑆𝑆𝑆𝑆𝑆𝑆𝑆𝑆
𝐼𝐼𝐼𝐼𝐼𝐼𝐼𝐼𝐼𝐼𝐼𝐼𝐼𝐼𝐼𝐼𝐼𝐼 𝑇𝑇𝑇𝑇𝑇𝑇𝑇𝑇𝑇𝑇𝑇𝑇𝑇𝑇𝑇𝑇 =
𝐼𝐼𝐼𝐼𝐼𝐼𝐼𝐼𝐼𝐼𝐼𝐼𝐼𝐼𝐼𝐼𝐼𝐼

2,300,000
𝐼𝐼𝐼𝐼𝐼𝐼𝐼𝐼𝐼𝐼𝐼𝐼𝐼𝐼𝐼𝐼𝐼𝐼 𝑇𝑇𝑇𝑇𝑇𝑇𝑇𝑇𝑇𝑇𝑣𝑣𝑒𝑒𝑒𝑒 =
175,000

𝐼𝐼𝐼𝐼𝐼𝐼𝐼𝐼𝐼𝐼𝐼𝐼𝐼𝐼𝐼𝐼𝐼𝐼 𝑇𝑇𝑇𝑇𝑇𝑇𝑇𝑇𝑇𝑇𝑇𝑇𝑇𝑇𝑇𝑇 = 13.14

In this case, the company had $13.14 of sales for each dollar of
inventory it had.

92
Return on Assets (ROA)

Return on Assets tells us how much net income the company


produced for each dollar of assets that it has.

𝑁𝑁𝑁𝑁𝑁𝑁 𝐼𝐼𝐼𝐼𝐼𝐼𝐼𝐼𝐼𝐼𝐼𝐼
𝑅𝑅𝑅𝑅𝑅𝑅𝑅𝑅𝑅𝑅𝑅𝑅 𝑜𝑜𝑜𝑜 𝐴𝐴𝐴𝐴𝐴𝐴𝐴𝐴𝐴𝐴𝐴𝐴 =
𝑇𝑇𝑇𝑇𝑇𝑇𝑇𝑇𝑇𝑇 𝐴𝐴𝐴𝐴𝐴𝐴𝐴𝐴𝐴𝐴𝐴𝐴

256,200
𝑅𝑅𝑅𝑅𝑅𝑅𝑅𝑅𝑅𝑅𝑅𝑅 𝑜𝑜𝑜𝑜 𝐴𝐴𝐴𝐴𝐴𝐴𝐴𝐴𝐴𝐴𝐴𝐴 =
1,318,200

= 0.1944

The company had $0.19 of net income for each dollar of assets.

93
Return on Equity (ROE)

Tells you how much net income you had for each dollar of
equity invested in the company.

𝑁𝑁𝑁𝑁𝑁𝑁 𝐼𝐼𝐼𝐼𝐼𝐼𝐼𝐼𝐼𝐼𝐼𝐼
𝑅𝑅𝑅𝑅𝑅𝑅𝑅𝑅𝑅𝑅𝑅𝑅 𝑜𝑜𝑜𝑜 𝐸𝐸𝐸𝐸𝐸𝐸𝐸𝐸𝐸𝐸𝐸𝐸 =
𝐶𝐶𝐶𝐶𝐶𝐶𝐶𝐶𝐶𝐶𝐶𝐶 𝐸𝐸𝐸𝐸𝐸𝐸𝐸𝐸𝐸𝐸𝐸𝐸

The difficulty in calculating Return on Equity is determining what


is debt, and what is equity? It can be difficult to make this
distinction for large firms with many different accounts. In these
cases, you must refer to the notes to the balance sheet to make a
determination.

Common Equity includes:


Common Stock
Retained Earnings
Additional Paid in Capital
Paid in Capital in Excess of Par Value

Common Equity does not include:


Any type of debt
Preferred Stock

256,200
𝑅𝑅𝑅𝑅𝑅𝑅𝑅𝑅𝑅𝑅𝑅𝑅 𝑜𝑜𝑜𝑜 𝐸𝐸𝐸𝐸𝐸𝐸𝐸𝐸𝐸𝐸𝐸𝐸 = = 0.3326
770,200

This says the company earned $0.3326 for each dollar of common
equity utilized.

94
Equity Multiplier

The Equity Multiplier tell you how many dollars of assets you are
controlling for each dollar of equity you have invested in the firm.

𝑇𝑇𝑇𝑇𝑇𝑇𝑇𝑇𝑇𝑇 𝐴𝐴𝐴𝐴𝐴𝐴𝐴𝐴𝐴𝐴𝐴𝐴
𝐸𝐸𝐸𝐸𝐸𝐸𝐸𝐸𝐸𝐸𝐸𝐸 𝑀𝑀𝑀𝑀𝑀𝑀𝑡𝑡𝑡𝑡𝑡𝑡𝑡𝑡𝑡𝑡𝑡𝑡𝑡𝑡 =
𝐶𝐶𝐶𝐶𝐶𝐶𝐶𝐶𝐶𝐶𝐶𝐶 𝐸𝐸𝐸𝐸𝐸𝐸𝐸𝐸𝐸𝐸𝐸𝐸

$1,318,200
𝐸𝐸𝐸𝐸𝐸𝐸𝐸𝐸𝐸𝐸𝐸𝐸 𝑀𝑀𝑀𝑀𝑀𝑀𝑀𝑀𝑀𝑀𝑀𝑀𝑀𝑀𝑀𝑀𝑀𝑀𝑀𝑀 = = 1.7115
$770,200

So the firm has accumulated $1.7115 of total assets for each dollar
of common equity invested in the firm.

95
Profit Margin (Sometimes called Net Profit Margin)

Profit Margin tells you how much net income the firm is producing
for each dollar of sales.

𝑁𝑁𝑁𝑁𝑁𝑁 𝐼𝐼𝐼𝐼𝐼𝐼𝐼𝐼𝐼𝐼𝐼𝐼
𝑃𝑃𝑃𝑃𝑃𝑃𝑃𝑃𝑃𝑃𝑃𝑃 𝑀𝑀𝑀𝑀𝑀𝑀𝑀𝑀𝑀𝑀𝑀𝑀 =
𝑆𝑆𝑆𝑆𝑆𝑆𝑆𝑆𝑆𝑆

$256,200
𝑃𝑃𝑃𝑃𝑃𝑃𝑃𝑃𝑃𝑃𝑃𝑃 𝑀𝑀𝑀𝑀𝑀𝑀𝑀𝑀𝑀𝑀𝑀𝑀 =
$2,300,000

𝑃𝑃𝑃𝑃𝑃𝑃𝑃𝑃𝑃𝑃𝑃𝑃 𝑀𝑀𝑀𝑀𝑀𝑀𝑀𝑀𝑀𝑀𝑀𝑀 = 0.1114

So the firm produces $0.1114 of net income for each dollar of


sales.

Debt to Assets

Debt to Assets tells which portion of the firm's assets are financed
by debt.
𝑇𝑇𝑇𝑇𝑇𝑇𝑇𝑇𝑇𝑇 𝐿𝐿𝐿𝐿𝐿𝐿𝐿𝐿𝐿𝐿𝐿𝐿𝐿𝐿𝐿𝐿𝐿𝐿𝐿𝐿𝐿𝐿
𝐷𝐷𝐷𝐷𝐷𝐷𝑡𝑡 𝑡𝑡𝑡𝑡 𝐴𝐴𝐴𝐴𝐴𝐴𝐴𝐴𝐴𝐴𝐴𝐴 =
𝑇𝑇𝑇𝑇𝑇𝑇𝑇𝑇𝑇𝑇 𝐴𝐴𝐴𝐴𝐴𝐴𝐴𝐴𝐴𝐴𝐴𝐴

548,000
𝐷𝐷𝐷𝐷𝐷𝐷𝐷𝐷 𝑡𝑡𝑡𝑡 𝐴𝐴𝐴𝐴𝐴𝐴𝐴𝐴𝐴𝐴𝐴𝐴 =
1,318,200

𝐷𝐷𝐷𝐷𝐷𝐷𝐷𝐷 𝑡𝑡𝑡𝑡 𝐴𝐴𝐴𝐴𝐴𝐴𝐴𝐴𝐴𝐴𝐴𝐴 = 0.4157

So, 41.57 percent of the firm's assets have been financed with debt
and the remainder is financed with equity.

96
Payout Ratio

The payout ratio address the issue of what you did with the money
you earned.

𝐷𝐷𝐷𝐷𝐷𝐷𝐷𝐷𝐷𝐷𝐷𝐷𝐷𝐷𝐷𝐷𝐷𝐷 𝑃𝑃𝑃𝑃𝑃𝑃𝑃𝑃 𝐷𝐷𝐷𝐷𝐷𝐷𝐷𝐷𝐷𝐷𝐷𝐷𝐷𝐷𝐷𝐷𝐷𝐷 𝑝𝑝𝑝𝑝𝑝𝑝 𝑆𝑆ℎ𝑎𝑎𝑎𝑎𝑎𝑎


𝑃𝑃𝑃𝑃𝑃𝑃𝑃𝑃𝑃𝑃𝑃𝑃 𝑅𝑅𝑅𝑅𝑅𝑅𝑅𝑅𝑅𝑅 = =
𝑁𝑁𝑁𝑁𝑁𝑁 𝐼𝐼𝐼𝐼𝐼𝐼𝐼𝐼𝐼𝐼𝐼𝐼 𝑁𝑁𝑁𝑁𝑁𝑁 𝐼𝐼𝐼𝐼𝐼𝐼𝐼𝐼𝐼𝐼𝐼𝐼 𝑝𝑝𝑝𝑝𝑝𝑝 𝑆𝑆ℎ𝑎𝑎𝑎𝑎𝑎𝑎

• Net Income per share is often referred to as Earnings per


share. They refer the same thing.

$11,000
𝑃𝑃𝑃𝑃𝑃𝑃𝑃𝑃𝑃𝑃𝑃𝑃 𝑅𝑅𝑅𝑅𝑅𝑅𝑅𝑅𝑅𝑅 = = 0.0429
$256,200

This indicates that we paid 4.29 percent of our net income to the
stockholders as a dividend.

97
The Current Ratio

The current ratio tells us the ability of the firm to pay its bills in the
short term.

Current Assets
Current Ratio =
Current Liabilities

The current ratio in 2012 is

$325,000
Current Ratio = = 0.929
$350,000

In 2013 the current ratio is

$543,200
Current Ratio = =1.365
$398,000

In 2012, the company had $0.929 in current assets for each dollar
of current liabilities.

98
Basic Dupont Analysis

In the 1920’s, the DuPont company developed a method for


analyzing ratios.

The analysis is based on the firm’s ROE, which is argued to be the


most important financial ratio. ROE gathers information contained
in all other ratios. The idea is to expand the ROE formula into its
components. By examining the components, we can identify how
various elements contribute to ROE.

Recall that:

𝑁𝑁𝑁𝑁𝑁𝑁 𝐼𝐼𝐼𝐼𝐼𝐼𝐼𝐼𝐼𝐼𝐼𝐼
𝑅𝑅𝑅𝑅𝑅𝑅𝑅𝑅𝑅𝑅𝑅𝑅 𝑜𝑜𝑜𝑜 𝐸𝐸𝐸𝐸𝐸𝐸𝐸𝐸𝐸𝐸𝐸𝐸 =
𝐶𝐶𝐶𝐶𝐶𝐶𝐶𝐶𝐶𝐶𝐶𝐶 𝐸𝐸𝐸𝐸𝐸𝐸𝐸𝐸𝐸𝐸𝐸𝐸

256,200
𝑅𝑅𝑅𝑅𝑅𝑅𝑅𝑅𝑅𝑅𝑅𝑅 𝑜𝑜𝑜𝑜 𝐸𝐸𝐸𝐸𝐸𝐸𝐸𝐸𝐸𝐸𝐸𝐸 = = 0.3326
770,200

We can expand the ROE equation to:

𝑅𝑅𝑅𝑅𝑅𝑅 = 𝑃𝑃𝑃𝑃𝑃𝑃𝑃𝑃𝑃𝑃𝑃𝑃 𝑀𝑀𝑀𝑀𝑀𝑀𝑀𝑀𝑀𝑀𝑀𝑀 𝑋𝑋 𝑇𝑇𝑇𝑇𝑇𝑇𝑇𝑇𝑇𝑇 𝐴𝐴𝐴𝐴𝐴𝐴𝐴𝐴𝐴𝐴 𝑇𝑇𝑇𝑇𝑇𝑇𝑇𝑇. 𝑋𝑋 𝐸𝐸𝐸𝐸𝐸𝐸𝐸𝐸𝐸𝐸𝐸𝐸 𝑀𝑀𝑀𝑀𝑀𝑀𝑀𝑀𝑀𝑀𝑀𝑀𝑀𝑀.

𝑅𝑅𝑅𝑅𝑅𝑅 = 0.1114 𝑋𝑋 1.7448 𝑋𝑋 1.7115 = 0.3326

99
And further expand it to:

𝑁𝑁𝑁𝑁𝑁𝑁 𝐼𝐼𝐼𝐼𝐼𝐼𝐼𝐼𝐼𝐼𝐼𝐼 𝑆𝑆𝑆𝑆𝑆𝑆𝑆𝑆𝑆𝑆 𝑇𝑇𝑜𝑜𝑜𝑜𝑜𝑜𝑜𝑜 𝐴𝐴𝐴𝐴𝐴𝐴𝐴𝐴𝐴𝐴𝐴𝐴


𝑅𝑅𝑅𝑅𝑅𝑅 = 𝑋𝑋 𝑋𝑋
𝑆𝑆𝑆𝑆𝑆𝑆𝑆𝑆𝑆𝑆 𝑇𝑇𝑇𝑇𝑇𝑇𝑇𝑇𝑇𝑇 𝐴𝐴𝐴𝐴𝐴𝐴𝐴𝐴𝐴𝐴𝐴𝐴 𝐶𝐶𝐶𝐶𝐶𝐶𝐶𝐶𝐶𝐶𝐶𝐶 𝐸𝐸𝐸𝐸𝐸𝐸𝐸𝐸𝐸𝐸𝐸𝐸

256,200 2,300,000 1,318,200


𝑅𝑅𝑅𝑅𝑅𝑅 = 𝑋𝑋 𝑋𝑋 = 0.3326
2,300,000 1,318,200 770,200

The first component indicates the contribution from the basic


earning power of the firm.

The second component tells us the contribution associated with the


speed of selling products.

The third component tell us the contribution associated with


borrowing money.

100
Two methods for determining if a ratio is good bad or average

1. Time Series Analysis

In Time Series Analysis, we are evaluating our firm as it


changes over time.

Our current ratio increased from 0.929 to 1.365. Most likely


this is a good sign, but it is not entirely clear

2. Industry Analysis (Benchmarking)

In Industry Analysis or Benchmarking, we are comparing our


firm to the average for firms in the same industry as we are in.

Step 1: Identify which industry that we operate in.

Suppose that we operate in the Textile Industry.

Step 2: Obtain industry average information for the industry


that you operate in.

A. Risk Management Associates


B. Value Line
C. Industry Associations
D. Private Studies

Suppose that you find that the average Current


Ratio for firms operating in the Textile industry
are 1.40.

101
Step 3: Compare your firm to the Industry

In 2012, our firm’s current ratio was substantially lower


than the industry average. The magnitude of this
difference would suggest some concern.

In 2013, our firm’s current ratio is nearly at the industry


average. The remaining difference would appear to be
is not large. Thus is appears as though the necessary
corrective actions have been taken by the firm.

New Measures of Firm Performance

Recently an assortment of new measures of firm performance


have been developed.

Developed to address various weaknesses of traditional


financial performance measures.

These new measures are becoming increasingly popular

1. Economic Value Added (EVA) (Stern Stewart and Company)

Attempts to make adjustments to financial statements


because accounting information can be misleading about the
actual performance of the firm.

For example a firm that expenses large amounts of


Research and Development Expenses.

A B
Sales $3,000,000 $3,000,000
Operating Expenses $1,600,000 $1,400,000
R&D Expenditures $0 $300,000
EBIT $1,400,000 $1,300,000

102
If we examine EBIT on its face, it appears as though
Company A is performing better than company B,
when company B is actually performing better.

EVA corrects this problem, and other problems by


making various accounting adjustments.

Some have criticized EVA as providing little new


information over other measures of firm
performance and being difficult to compute resulting
in large errors.

2. Balanced Scorecard

Balanced Scorecard argues that Financial Information


provides only a partial view of the performance of a firm.

It argues that other elements should be considered when


evaluating overall firm performance.

Balanced scorecard provides an integrated framework to help


managers implement strategies, measure performance, and
compensate employees.

It does this by developing goals and measures along different


perspectives and linking them together with the vision,
mission and strategy of the firm.

3. Market Value Added


4. Wealth Added Index
5. Tracking Stocks

103
Taxes

“The Only Two Things that you can be sure of in life are Death
and Taxes” Unknown

We will focus here on two issues.

#1. How to compute your taxes

#2. How you can use a business to reduce your personal


taxes.

104
Computing Your Taxes

Step 1: Compute Taxable Income

Salary
+ Investment Income
+- Gain or Loss on Business
- Contributions to Tax Deferred Accounts (IRA, 401K)
- Deductions
- Exemptions
= Taxable Income

2013 Standard Deduction for Married People is $12,100.


2013 Exemption Rate $3,900 per individual in Household.

Suppose Peggy Sherven works at the University of


Minnesota. She made $116,000 last year from her job as a
high-level executive at the University. She is a married
individual with one child. Her husband stays home to take
care of their child, so he does not work. She put $6000 into a
401K account and $2,000 into an IRA this year. She operates
a hotdog stand outside her home on Saturdays when there are
major football games at the nearby stadium. She earned
$4,000 this year from the hotdog stand. Compute Peggy’s
taxable income.

Salary $116,000
+ Investment Income $0
+ Gain from Business $4,000
- Contributions to 401K $6,000
- Contributions to IRA $2,000
- Deductions $12,100
- Exemptions 3X 3,900 $11,700
= Taxable Income $88,200

105
Step 2: Gather data from Tax Table

#1. Find the Range of Income from Column 1

in this case our income falls between $43,850 and


$105,950. This is our range

#2. Find the Base Amount from Column 2

in this case our base amount is $43,850.

#3. Find the Tax on the Base Amount from Column 3

in this case the tax on the base amount is $6,577.50

#4. Find the Tax Rate from Column 4

in this case, the tax rate is 28 percent.

Step 3: Solve for the Tax owed by using the following formula:

Tax = Tax on Base Amt + Tax Rate (Taxable Inc - Base Amt)

= $6577.50 + 0.28(88,200-43,850)

= $18,995.50

106
Tax Table

Single Individuals
Range Base Amt Tax on Tax Rate Avg. Rate at
Base Amt Top of
Bracket

0 to $26,250 $0 $0 15% 15%


$26,250-$63,650 $26,250 $3937.50 28% 22.6%
$63,650-$132,600 $63,650 $14,381.50 31% 27%
$132,600-$288.350 $132,600 $35,787 36% 31.9%
Above 288,350 $288,350 $91,587 39.6% 39.6%

Married Couples Filing a Joint Return


Range Base Amt Tax on Tax Rate Avg. Rate at
Base Amt Top of
Bracket

0 to $43,850 $0 $0 15% 15%


$43,850-$105,950 $43,850 $6577.50 28% 22.6%
$105,950-$161,450 $105,950 $23,965.50 31% 27%
$161,450-$288.350 $161,450 $41,170.50 36% 31.9%
Over 288,350 $288,350 $86,854.50 39.6% 39.6%

Corporations
Range Base Amt Tax on Tax Rate Avg. Rate at
Base Amt Top of
Bracket
0 to $50,000 $0 $0 15% 15%
$50,000-$75,000 $50,000 $7,500 25% 18.3%
$75,000-$100,000 $75,000 $13,750 34% 22.3%
$100,000-$335,000 $100,000 $22,250 39% 34.0%
$335,000-$10mil $335,000 $113,900 34% 34.0%
$10mil-$15mil $10mil $3,400,000 35% 34.3%
$15mil-$18.33mil $15mil $5,150,000 38% 35%
above 18.33 mil 18.33mil $6,416,667 35% 35%

107
Using a Business to Reduce your Personal Taxes
Through careful planning, a business can be used to offset income
that you earn from your job.

Suppose that you work at a job where you earn $45,000 per year.
You are single and you have no children. You enjoy fishing and
go fishing almost every weekend. You eat the fish that you catch.

You will have to pay taxes as follows:

Salary $45,000
+ other income -0-
- Deductions $6,050
- Exemptions $3,900
Taxable Income $35,050

Tax = tax on base amount + tax rate (taxable income – base


amount)

= $3937.50 + 0.28 ($35,050 - $26,250)

= $6,401.50

108
Now suppose that rather than eating the fish you catch, you obtain
a commercial fisherman’s license and sell the fish that you catch to
the local fish distributor. Because you have a profit motive, you
are in business and can deduct all expenses associated with the
activity.

For the business you purchase a truck, a boat and a computer.


They will cost $70,000. They will wear out by $10,000 per year.
You will operate the business out of your home where you will use
the garage to store your boat and truck and will have an office in
one room. You will use 1/2 of your house for the business and the
other half for your residence. Your house rent is $1,000 per month.
Because you use ½ of your house for business, ½ of your rent is a
tax-deductible expense.

Suppose that you go out fishing and you catch $47,000 of fish.
The operating expenses associated with catching the fish (gas, bait
and so forth) total $37,000. So you have a gross profit of $10,000
($47,000 - $37,000). However, because you have a business many
additional expenses are tax deductible.

You can re-compute your taxes in light of the business as follows:

109
Step 1: Compute the Taxable profit or loss from the Business

Sales $47,000
Operating Expenses $37,000
Gross Profit $10,000
Other Expenses
Depreciation on Boat, Truck
And Computer $10,000
½ of house rent $6,000
Internet Connection $250
Travel Expenses
(to Los Angeles to investigate the purchase of a new boat
and then to visit your friend Sandy) $2,000
Taxable Profit -$8,250

Step 2: Re-compute your taxes

Salary $45,000
+ other income -8,250
- Deductions $6,050
- Exemptions $3,900
Taxable Income $26,800

Tax = tax on base amount + tax rate (taxable income – base


amount)

= $3937.50 + 0.28 ($26,800 - $26,250)

= $4,091.50

Thus, by selling your fish at the market rather than eat them, you
reduce your taxes by:

$6,401.50 - $4,091.50 = $2,310

110
Chapter 2 Homework and
Solutions
Problem #1. You have been assigned the problem of creating financial statements for a company for the
next year. You have the following information. The company was started on December 31,
2012.

Balance Sheet as of December 31, 2012

Assets
Cash $2,000
+ Inventory $20,000
Buildings and Equipment $10,000
Less Accumulated Depreciation $0
+ Net Buildings and Equipment $10,000
Total Assets $32,000

Liabilities
Bank Loan $10,000

Owners Equity
Common Stock $22,000
Retained Earnings $0

Total Liabilities and Owners Equity $32,000

You have created the following income statement for the year ending December 31, 2013.

Sales $80,000
- Cost of Goods Sold $20,000
- Administrative Expenses $5,000
- Labor $30,000
- Depreciation $1,000
=Earnings Before Interest and Tax $24,000
- Interest $1,000
=Earnings before Taxes $23,000
- Taxes @ 20 percent $4,600
=Net Income $18,400

Other information:
All sales are cash sales
All expenses except depreciation are cash expenses
No principal payments were paid on the bank loan.
The company paid $5,000 in dividends from cash.
Inventory Levels do not change during the year.

Your job is to create a retained earnings statement for the year ending December 31, 2013, and a balance
sheet for December 31, 2013.

Note: For this problem, the cash balance for the December 31, 2013 balance sheet is $16,400.

111
Problem 2: Today is December 31, 2018. Today, we started the Hilo Tourism Company. To start the
business, the owners have invested $25,000 into the business from their own pocket. In addition,
the company borrowed $15,000 at a 10 percent simple interest rate. Thus, the company has
$40,000 to work with. The company will use $10,000 of this money for operating cash. The
company will use the remaining money to purchase a tourism van for $30,000.

The Hilo Tourism Company will contract our services to Royal Cruise Line who has agreed to pay
us $160,000 in the year 2019 for providing tours to high-class customers from the Hilo dock to the
Volcano. All sales are cash sales. Hilo Tourism Company will have the following expenses in the
year 2019. The van will wear out by $10,000 per year. Insurance will cost $12,000 per year. It
will cost $20,000 per year for the driver and $16,000 per year for the tour guide. Fuel and repairs
combined will cost $40,000 per year. All operating expenses except depreciation are cash
operating expenses. The company is in a 10 percent tax bracket. One half of the firm’s 2019 net
income will be paid out to the owners as a dividend and the other half of the firm’s net income
will be retained within the firm. The company will make interest payments on the loan only.

A. Create a Balance Sheet for December 31, 2018


B. Create an Income Statement for the year ending December 31, 2019
C. Create a Statement of Cash Flows for the year ending December 31, 2019
D. Create a Statement of Retained Earnings for the year ending December 31, 2019

Problem 3: Today is December 30, 2012. We will start a staple gun manufacturing company. To
start the company we needed $20,000 to purchase equipment. We need $5,000 for
operating funds. Thus to start the company we needed a total of $25,000. We obtained the
money to start the business by investing $15,000 from our pocket and by borrowing the
remaining $10,000 at a 5 percent simple interest rate. In addition to paying the interest
payments on the loan each year, we will pay off $1,000 of the amount that we owe on the
loan each year (the first payment on the loan will be made one year from today).

The firm will sell 200,000 staplers each year at a price of $3.00 each. Cost of
producing the staplers will be $2.00 each. In addition, sales and administrative
expenses will be $0.18125 per stapler. All sales are cash sales. All operating expenses
are cash operating expenses. The machine will wear out buy $2,000 per year. The
company is in a 10 percent tax bracket. One-fourth of the firm’s net income will be
paid out to the owners as a dividend and the other three-fourths of the firms net income
will be retained within the firm.

A. Create a balance sheet for December 31, 2012.

B. Create an income statement for the year ending December 31, 2013.

C. Create a statement of cash flows for the year ending December 31, 2013.

D. Create a statement of retained earnings for the year ending December 31, 2013.

E. Create a balance sheet for December 31, 2013.

F. Create an income statement for the year ending December 31, 2014

G. Create a statement of retained earnings for the year ending December 31, 2014.

H. Create a statement of cash flows for the year ending December 31, 2014

I. Create a balance sheet for December 31, 2014.

112
Problem 4: Today is December 30, 2012. Terry and Mercedes started a tourism company called Hilo
Tours today. In order to start the company we needed $53,000 to purchase a van and an additional
$5,000 for operating funds. Thus to start the company we needed a total of $58,000. We got the
money to start the business by investing $15,000 from our pocket and by borrowing the remaining
$43,000 at a 10 percent simple interest rate. In addition to paying the interest payments on the
loan each year, we will pay off $10,000 of the amount that we owe on the loan each year (the first
payment on the loan will be made one year from today).

Hilo Tours will contract our services to another company called Valley Tours. Valley Tours has
agreed to pay us $275 per day for providing tours from Hilo to the Waipio Valley. Valley Tours
will provide us with work 250 days per year. The van is expected to wear out by $8,000 per year.
We are responsible for providing the van and all fuel, a driver, a tour guide, and a snack for the
passengers. The van will drive 200 miles per working day. It will cost $40 per working day for
the driver and $30 per working day for the tour guide. It will cost $15 per day to provide the
snack. Fuel will cost $.10 per mile and repairs for the van will be $.20 per mile. All operating
expenses are cash operating expenses. The company is in a 10 percent tax bracket. One half of
the firm’s net income will be paid out to the owners as a dividend and the other half of the firms
net income will be retained within the firm.

A. Create a balance sheet for December 31, 2012.

B. Create an income statement for the year ending December 31, 2013.

C. Create a statement of cash flows for the year ending December 31, 2013.

D. Create a statement of retained earnings for the year ending December 31, 2013.

E. Create a balance sheet for December 31, 2013.

F. Create an income statement for the year ending December 31, 2014

G. Create a statement of retained earnings for the year ending December 31, 2014.

H. Create a statement of cash flows for the year ending December 31, 2014

I. Create a balance sheet for December 31, 2014.

113
Problem 5: You are provided the following balance sheet and income statement. In addition, you are
informed that the current stock price of the firm is $10 per share and there are 5,000 shares
outstanding. Use this information to compute the following:

A. Market Value of Equity


B. Price to Earnings Ratio
C. Inventory Turnover
D. Total Asset Turnover
E. Return on Assets
F. Return on Equity
G. Profit Margin
H. Debt to Assets
I. Equity Multiplier
J. Dupont Analysis

Assets
Cash $2,000
+ Inventory $20,000
Buildings and Equipment $10,000
Less Accumulated Depreciation $0
+ Net Buildings and Equipment $10,000
Total Assets $32,000

Liabilities
Bank Loan $10,000

Owners Equity
Common Stock $22,000
Retained Earnings $0

Total Liabilities and Owners Equity $32,000

You have created the following income statement for the year ending December 31, 2010.

Sales $80,000
- Cost of Goods Sold $20,000
- Administrative Expenses $5,000
- Labor $30,000
- Depreciation $1,000
=Earnings Before Interest and Tax $24,000
- Interest $1,000
=Earnings before Taxes $23,000
- Taxes @ 20 percent $4,600
=Net Income $18,400

114
Problem 6: You are provided the following Income Statement and Balance Sheet for Microsoft.

MICROSOFT CORP
BALANCE_SHEET
Period End: Jun 30, 2016
(In Millions)
2016 2015
Assets
Current assets:
Cash and cash equivalents $ 6,510 $ 5,595
Short-term investments (including securities loaned of $204 and $75) 106,730 90,931
------------------------------------------------------------------------------------------------------------------------------- -----------------
Total cash, cash equivalents, and short-term investments 113,240 96,526

Accounts receivable, net of allowance for doubtful accounts 18,277 17,908


Inventories 2,251 2,902
Other 5,892 5,461

Total current assets 139,660 122,797


Property and equip., net of accum. depreciation of $19,800 and $17,606) 18,356 14,731
Equity and other investments 10,431 12,053
Goodwill 17,872 16,939
Intangible assets, net 3,733 4,835
Other long-term assets 3,642 3,117

Total assets $ 193,694 $ 174,472

Liabilities and stockholders' equity


Current liabilities:
Accounts payable $ 6,898 $ 6,591
Short-term debt 12,904 4,985
Current portion of long-term debt 0 2,499
Accrued compensation 5,264 5,096
Income taxes 580 606
Short-term unearned revenue 27,468 23,223
Securities lending payable 294 92
Other 5,949 6,555
------------------------------------------------------------------------------------------------------------------------------- -----------------
Total current liabilities 59,357 49,647
Long-term debt 40,783 27,808
Long-term unearned revenue 6,441 2,095
Deferred income taxes 1,476 1,295
Other long-term liabilities 13,640 13,544

Total liabilities 121,697 94,389

Stockholders' equity:
Common stock and paid-in capital - shares authorized 24,000;
outstanding 7,808 and 8,027 68,178 68,465
Retained earnings 2,282 9,096
Accumulated other comprehensive income 1,537 2,522

Total stockholders' equity 71,997 80,083


------------------------------------------------------------------------------------------------------------------------------- -----------------
Total liabilities and stockholders' equity $ 193,694 $ 174,472

115
MICROSOFT CORP
INCOME_STATEMENT
Period End: Jun 30, 2016

(In millions, except per share amounts)


----------------------------------------------------------------------------------------------------------------------------

Year Ended June 30, 2016 2015 2014

Revenue:
Product $ 61,502 $ 75,956 $ 72,948
Service and other 23,818 17,624 13,885
------------------------------------------------------------------------------------------ ---------------- ----------------
Total revenue 85,320 93,580 86,833
------------------------------------------------------------------------------------------ ---------------- ----------------
Cost of revenue:
Product 17,880 21,410 16,681
Service and other 14,900 11,628 10,397
------------------------------------------------------------------------------------------ ---------------- ----------------
Total cost of revenue 32,780 33,038 27,078
------------------------------------------------------------------------------------------ ---------------- ----------------
Gross margin 52,540 60,542 59,755
Research and development 11,988 12,046 11,381
Sales and marketing 14,697 15,713 15,811
General and administrative 4,563 4,611 4,677
Impairment, integration, and restructuring 1,110 10,011 127
------------------------------------------------------------------------------------------ ---------------- ----------------
Operating income 20,182 18,161 27,759
Other income (expense), net (431) 346 61
------------------------------------------------------------------------------------------ ---------------- ----------------
Income before income taxes 19,751 18,507 27,820
Provision for income taxes 2,953 6,314 5,746
------------------------------------------------------------------------------------------ ---------------- ----------------
Net income $ 16,798 $ 12,193 $ 22,074
----------------- ---------------- ----------------

Earnings per share:


Basic $ 2.12 $ 1.49 $ 2.66
Diluted $ 2.10 $ 1.48 $ 2.63

Weighted average shares outstanding:


Basic 7,925 8,177 8,299
Diluted 8,013 8,254 8,399

Cash dividends declared per common share $ 1.44 $ 1.24 $ 1.12


----------------------------------------------------------------------------------------------------------------------------

116
In addition to the above data, you have checked online and found that the current price of Microsoft stock
is $69.08. Using the Microsoft Data, calculate the following ratios for 2016:

A. Market Value of Equity


B. Price to Earnings Ratio
C. Total Asset Turnover
D. Inventory Turnover
E. Return on Assets
F. Return on Equity
G. Profit Margin
H. Debt to Assets
I. Payout Ratio
J. Current Ratio
K. Equity Multiplier
L. Dupont Analysis

7. Much of the HTTP company’s financial records were destroyed in a recent fire. You have recovered
some information and wish to use this information to recreate as many account balances as possible.
You have recovered the following information:

The Return on Assets Ratio = 0.50.


The Net Income = $500,000
Total Asset Turnover = 2.5
Return on Equity = 0.25
Current Ratio = 4.0
Current Liabilities = $25,000
Inventory Turnover = 3

With this information, your job is to calculate the following values:

a. Total Assets
b. Sales
c. Profit Margin
d. Common Equity
e. Current Assets
f. Beginning of year Inventory

117
Chapter 2 Homework Solutions

Solution to Problem 1

Retained Earnings Statement for the year ending December 31, 2013

Retained Earnings 12-31-12 0


+ net income 2013 $18,400
- dividends paid $5,000
Retained Earnings 2013 $13,400

Statement of Cash Flows

Step 1: Compute the Change in Cash Position

Net Income $18,400


+ Depreciation $1,000
Increases in Liabilities
+ Increases in Accounts Payable
+ Increases in Notes Payable
+ Increases in Accruals
+ Increases in Long-Term Debt
Increases in Assets
- Increases in Inventory
- Increases in Accounts Receivable
- Increases in other Current Assets
- Increases in Gross Fixed Assets
Cash Paid to and Received from Stockholders
+ New Investment by Stockholders
- Dividends Paid $5,000
= Change in Cash Position $14,400

Step 2: Compute the New Cash Balance

Old Cash Balance $2,000


+ Change in Cash Position $14,400
= New Cash Balance $16,400

Balance Sheet as of December 31, 2013

Assets
Cash $16,400
Buildings and Equipment $10,000
Less Accumulated Depreciation $1000
+ Net Buildings And Equipment $9,000
+ Inventory $20,000
= Total Assets $45,400

Liabilities
Bank Loan $10,000

Owners Equity

Common Stock $22,000


Retained Earnings $13,400
Total Liabilities and Equity $45,400

118
Solution to Problem #2

A. Balance Sheet as of December 31, 2018

Assets:

Cash $10,000
Van $30,000

Total Assets $40,000

Liabilities and Equity

Bank Loan $15,000


Common Stock $25,000
Retained Earnings $0

Total Liabilities and Equity $40,000

B. Income Statement for Year Ending December 31, 2019

Sales $160,000
- Driver $20,000
- Tour Guide $16,000
- Fuel and Repairs $40,000
- Insurance $12,000
- Depreciation $10,000

= Earnings Before Interest and Taxes $62,000

- Interest Expenses $1,500

= Earnings Before Taxes $60,500

- Taxes $6,050

= Net Income $54,450

119
C. Statement of Cash Flows for Year Ending December 31, 2019

Step 1: Compute the Change in Cash Position

Net Income $54,450


+ Depreciation $10,000
Increases in Liabilities
+ Increases in Accounts Payable
+ Increases in Notes Payable
+ Increases in Accruals
+ Increases in Long-Term Debt
Increases in Assets
- Increases in Inventory
- Increases in Accounts Receivable
- Increases in other Current Assets
- Increases in Gross Fixed Assets
Cash Paid to and Received from Stockholders
+ New Investment by Stockholders
- Dividends Paid $27,225
= Change in Cash Position $37,225

Step 2: Compute the New Cash Balance

Old Cash Balance $10,000


+ Change in Cash Position $37,225
= New Cash Balance $47,225

D. Statement of Retained Earnings for Year Ending December 31, 2019

Retained Earnings Beginning of the Year 0


+ Net Income $54,450
- Dividends $27,225
= Retained Earnings End of the Year $27,225

120
Solution to Problem #3

A. Balance Sheet as of December 31, 2012

Assets:

Cash $5,000
Equipment $20,000
Accumulated. Depreciation 0
Net Equipment $20,000
Total Assets $25,000

Liabilities and Equity

Bank Loan $10,000


Common Stock $15,000
Retained Earnings $0

Total Liabilities and Equity $25,000

B. Income Statement for Year Ending December 31, 2013

Sales $600,000
-Cost of Production $400,000
-Sales and Adm. $36,250
- Depreciation $2,000

= Earnings Before Interest and Taxes $161,750

- Interest Expenses $500

= Earnings Before Taxes $161,250

- Taxes $16,125

= Net Income $145,125

121
C. Statement of Cash Flows for Year Ending December 31, 2013

Step 1: Compute the Change in Cash Position

Net Income $145,125


+ Depreciation $2,000
Increases in Liabilities
+ Increases in Accounts Payable
+ Increases in Notes Payable
+ Increases in Accruals
+ Increases in Long-Term Debt -$1,000
Increases in Assets
- Increases in Inventory
- Increases in Accounts Receivable
- Increases in other Current Assets
- Increases in Gross Fixed Assets
Cash Paid to and Received from Stockholders
+ New Investment by Stockholders
- Dividends Paid $36,281.25
= Change in Cash Position $109,843.75

Step 2: Compute the New Cash Balance

Old Cash Balance $5,000


+ Change in Cash Position $109,843.75
= New Cash Balance $114,843.75

D. Statement of Retained Earnings for Year Ending December 31, 2013

Retained Earnings Beginning of the Year 0


+ Net Income $145,125
- Dividends $36,281.25
= Retained Earnings End of the Year $108,843.75

122
E. Balance Sheet as of December 31, 2013

Assets:

Cash $114.843.75
Equipment $20,000
Accumulated. Depreciation $2,000
Net Equipment $18,000
Total Assets $132,843.75

Liabilities and Equity

Bank Loan $9,000


Common Stock $15,000
Retained Earnings $108,843.75

Total Liabilities and Equity $132,843.75

F. Income Statement for Year Ending December 31, 2014

Sales $600,000
-Cost of Production 400,000
-Sales and Admin $36,250
- Depreciation $2,000

= Earnings Before Interest and Taxes $161,750

- Interest Expenses $450

= Earnings Before Taxes $161,300

- Taxes $16,130

= Net Income $145,170

G. Statement of Retained Earnings for Year Ending December 31, 2014

Retained Earnings Beginning of the Year $108,843.75


+ Net Income $145,170
- Dividends $36,292.50
= Retained Earnings End of the Year $217,721.25

123
H. Statement of Cash Flows for Year Ending December 31, 2014

Step 1: Compute the Change in Cash Position

Net Income $145,170


+ Depreciation $2,000
Increases in Liabilities
+ Increases in Accounts Payable
+ Increases in Notes Payable
+ Increases in Accruals
+ Increases in Long-Term Debt -$1,000
Increases in Assets
- Increases in Inventory
- Increases in Accounts Receivable
- Increases in other Current Assets
- Increases in Gross Fixed Assets
Cash Paid to and Received from Stockholders
+ New Investment by Stockholders
- Dividends Paid $36,292.50
= Change in Cash Position $109,877.50

Step 2: Compute the New Cash Balance

Old Cash Balance $114,843.75


+ Change in Cash Position $109,877.50
= New Cash Balance $224,721.25

I. Balance Sheet as of December 31, 2014

Assets:

Cash $224,721.25
Equipment $20,000
Accumulated. Depreciation $4,000
Net Equipment $16,000
Total Assets $240,721.25

Liabilities and Equity

Bank Loan $8,000


Common Stock $15,000
Retained Earnings $217,721.25

Total Liabilities and Equity $240,721.25

124
Solution to Problem #4

A. Balance Sheet as of December 31, 2012

Assets:

Cash $5000
Equipment $53,000
Accumulated. Depreciation 0
Net Equipment $53,000
Total Assets $58,000

Liabilities and Equity

Bank Loan $43,000


Common Stock $15,000
Retained Earnings $0

Total Liabilities and Equity $58,000

B. Income Statement for Year Ending December 31, 2013

Sales $68,750
Operating Expenses
Driver $10,000
Guide $7,500
Repairs $10,000
Fuel $5,000
Meals $3,750
- Operating Expenses $36,250

- Depreciation $8,000

= Earnings Before Interest and Taxes $24,500

- Interest Expenses $4,300

= Earnings Before Taxes $20,200

- Taxes $2,020

= Net Income $18,180

125
C. Statement of Cash Flows for Year Ending December 31, 2013

Step 1: Compute the Change in Cash Position

Net Income $18,180


+ Depreciation $8,000
Increases in Liabilities
+ Increases in Accounts Payable
+ Increases in Notes Payable
+ Increases in Accruals
+ Increases in Long-Term Debt -$10,000
Increases in Assets
- Increases in Inventory
- Increases in Accounts Receivable
- Increases in other Current Assets
- Increases in Gross Fixed Assets
Cash Paid to and Received from Stockholders
+ New Investment by Stockholders
- Dividends Paid $9,090
= Change in Cash Position $7,090

Step 2: Compute the New Cash Balance

Old Cash Balance $5,000


+ Change in Cash Position $7,090
= New Cash Balance $12,090

D. Statement of Retained Earnings for Year Ending December 31, 2013

Retained Earnings Beginning of the Year 0


+ Net Income $18,180
- Dividends $9,090
= Retained Earnings End of the Year $9,090

126
E. Balance Sheet as of December 31, 2013

Assets:

Cash $12,090
Equipment $53,000
Accumulated. Depreciation $8,000
Net Equipment $45,000
Total Assets $57,090

Liabilities and Equity

Bank Loan $33,000


Common Stock $15,000
Retained Earnings $9,090

Total Liabilities and Equity $57,090

F. Income Statement for Year Ending December 31, 2014

Sales $68,750
Operating Expenses
Driver $10,000
Guide $7,500
Repairs $10,000
Fuel $5,000
Meals $3,750
- Operating Expenses $36,250

- Depreciation $8,000

= Earnings Before Interest and Taxes $24,500

- Interest Expenses $3,300

= Earnings Before Taxes $21,200

- Taxes $2,120

= Net Income $19,080

G. Statement of Retained Earnings for Year Ending December 31, 2014

Retained Earnings Beginning of the Year $9,090


+ Net Income $19,080
- Dividends $9,540
= Retained Earnings End of the Year $18,630

127
H. Statement of Cash Flows for Year Ending December 31, 2014

Step 1: Compute the Change in Cash Position

Net Income $19,080


+ Depreciation $8,000
Increases in Liabilities
+ Increases in Accounts Payable
+ Increases in Notes Payable
+ Increases in Accruals
+ Increases in Long-Term Debt -$10,000
Increases in Assets
- Increases in Inventory
- Increases in Accounts Receivable
- Increases in other Current Assets
- Increases in Gross Fixed Assets
Cash Paid to and Received from Stockholders
+ New Investment by Stockholders
- Dividends Paid $9,540
= Change in Cash Position $7,540

Step 2: Compute the New Cash Balance

Old Cash Balance $12,090


+ Change in Cash Position $7,540
= New Cash Balance $19,630

I. Balance Sheet as of December 31, 2014

Assets:

Cash $19,630
Equipment $53,000
Accumulated. Depreciation $16,000
Net Equipment $37,000
Total Assets $56,630

Liabilities and Equity

Bank Loan $23,000


Common Stock $15,000
Retained Earnings $18,630

Total Liabilities and Equity $56,630

128
Solution to Problem 5:

A. Market Value of Equity

𝑀𝑀𝑀𝑀𝑀𝑀𝑀𝑀𝑀𝑀𝑀𝑀 𝑉𝑉𝑉𝑉𝑉𝑉𝑉𝑉𝑉𝑉 𝑜𝑜𝑜𝑜 𝐸𝐸𝐸𝐸𝐸𝐸𝐸𝐸𝐸𝐸𝐸𝐸 = 𝑆𝑆ℎ𝑎𝑎𝑎𝑎𝑎𝑎𝑎𝑎 𝑂𝑂𝑂𝑂𝑂𝑂𝑂𝑂𝑂𝑂𝑂𝑂𝑂𝑂𝑂𝑂𝑂𝑂𝑂𝑂𝑂𝑂 𝑋𝑋 𝑃𝑃𝑃𝑃𝑃𝑃𝑃𝑃𝑃𝑃 𝑃𝑃𝑃𝑃𝑃𝑃 𝑆𝑆ℎ𝑎𝑎𝑎𝑎𝑎𝑎

= 5,000 X $10 = $50,000

If you wanted to purchase the entire company, it would cost $50,000.

B. Price to Earnings Ratio

𝑀𝑀𝑀𝑀𝑀𝑀𝑀𝑀𝑀𝑀𝑀𝑀 𝑉𝑉𝑉𝑉𝑉𝑉𝑉𝑉𝑉𝑉 𝑜𝑜𝑜𝑜 𝐸𝐸𝐸𝐸𝐸𝐸𝐸𝐸𝐸𝐸𝐸𝐸 𝑂𝑂𝑂𝑂𝑂𝑂𝑂𝑂𝑂𝑂𝑂𝑂ℎ𝑖𝑖𝑖𝑖


𝑃𝑃𝑃𝑃𝑃𝑃𝑃𝑃𝑃𝑃 𝑡𝑡𝑡𝑡 𝐸𝐸𝐸𝐸𝑟𝑟𝑟𝑟𝑟𝑟𝑟𝑟𝑟𝑟𝑟𝑟 𝑅𝑅𝑅𝑅𝑅𝑅𝑅𝑅𝑅𝑅 =
𝑁𝑁𝑁𝑁𝑁𝑁 𝐼𝐼𝐼𝐼𝐼𝐼𝐼𝐼𝐼𝐼𝐼𝐼

$50,000
=
$18,400

= 2.717 times

The P/E ratio tells us that if earnings do not change from their current levels, it will take 2.717
years before the firm will earn back the amount of money required to purchase the company today.

C. Inventory Turnover

𝑆𝑆𝑆𝑆𝑆𝑆𝑆𝑆𝑆𝑆
𝐼𝐼𝐼𝐼𝐼𝐼𝐼𝐼𝐼𝐼𝐼𝐼𝐼𝐼𝐼𝐼𝐼𝐼 𝑇𝑇𝑇𝑇𝑇𝑇𝑇𝑇𝑇𝑇𝑇𝑇𝑇𝑇𝑇𝑇 =
𝐼𝐼𝐼𝐼𝐼𝐼𝐼𝐼𝐼𝐼𝐼𝐼𝐼𝐼𝐼𝐼𝐼𝐼

80,000
𝐼𝐼𝐼𝐼𝐼𝐼𝐼𝐼𝐼𝐼𝐼𝐼𝐼𝐼𝐼𝐼𝐼𝐼 𝑇𝑇𝑇𝑇𝑇𝑇𝑇𝑇𝑇𝑇𝑇𝑇𝑇𝑇𝑇𝑇 = =4
20,000

Inventory Turnover tells is that we have 4 dollars of sales for each dollar of inventory.

D. Total Asset Turnover

𝑆𝑆𝑆𝑆𝑆𝑆𝑆𝑆𝑆𝑆
𝑇𝑇𝑇𝑇𝑇𝑇𝑇𝑇𝑇𝑇 𝐴𝐴𝐴𝐴𝐴𝐴𝐴𝐴𝐴𝐴 𝑇𝑇𝑇𝑇𝑇𝑇𝑇𝑇𝑇𝑇𝑇𝑇𝑇𝑇𝑇𝑇 =
𝑇𝑇𝑇𝑇𝑇𝑇𝑇𝑇𝑇𝑇 𝐴𝐴𝐴𝐴𝐴𝐴𝐴𝐴𝐴𝐴𝐴𝐴

$80,000
𝑇𝑇𝑇𝑇𝑇𝑇𝑇𝑇𝑇𝑇 𝐴𝐴𝐴𝐴𝐴𝐴𝐴𝐴𝐴𝐴 𝑇𝑇𝑇𝑇𝑇𝑇𝑇𝑇𝑇𝑇𝑇𝑇𝑇𝑇𝑇𝑇 =
32,000

𝑇𝑇𝑇𝑇𝑇𝑇𝑇𝑇𝑇𝑇 𝐴𝐴𝐴𝐴𝐴𝐴𝐴𝐴𝑡𝑡 𝑇𝑇𝑇𝑇𝑇𝑇𝑇𝑇𝑇𝑇𝑇𝑇𝑇𝑇𝑇𝑇 = 2.5

So in this case the company had $2.50 of sales for each dollar of assets that it had.

129
E. Return on Assets

𝑁𝑁𝑁𝑁𝑁𝑁 𝐼𝐼𝐼𝐼𝐼𝐼𝐼𝐼𝐼𝐼𝐼𝐼
𝑅𝑅𝑅𝑅𝑅𝑅𝑅𝑅𝑅𝑅𝑅𝑅 𝑜𝑜𝑜𝑜 𝐴𝐴𝐴𝐴𝐴𝐴𝐴𝐴𝐴𝐴𝐴𝐴 =
𝑇𝑇𝑇𝑇𝑇𝑇𝑇𝑇𝑇𝑇 𝐴𝐴𝐴𝐴𝐴𝐴𝐴𝐴𝐴𝐴𝐴𝐴

18,400
𝑅𝑅𝑅𝑅𝑅𝑅𝑅𝑅𝑅𝑅𝑅𝑅 𝑜𝑜𝑜𝑜 𝐴𝐴𝐴𝐴𝐴𝐴𝐴𝐴𝐴𝐴𝐴𝐴 =
32,000

= 0.575

So the company had $0.575 of net income for each dollar of assets.

F. Return on Equity

𝑁𝑁𝑁𝑁𝑁𝑁 𝐼𝐼𝐼𝐼𝐼𝐼𝐼𝐼𝐼𝐼𝐼𝐼
𝑅𝑅𝑅𝑅𝑅𝑅𝑅𝑅𝑅𝑅𝑅𝑅 𝑜𝑜𝑜𝑜 𝐸𝐸𝐸𝐸𝐸𝐸𝐸𝐸𝐸𝐸𝐸𝐸 =
𝐶𝐶𝐶𝐶𝐶𝐶𝐶𝐶𝐶𝐶𝐶𝐶 𝐸𝐸𝐸𝐸𝐸𝐸𝐸𝐸𝐸𝐸𝐸𝐸

18,400
𝑅𝑅𝑅𝑅𝑅𝑅𝑅𝑅𝑅𝑅𝑅𝑅 𝑜𝑜𝑜𝑜 𝐸𝐸𝐸𝐸𝐸𝐸𝐸𝐸𝐸𝐸𝐸𝐸 =
22,000

𝑅𝑅𝑅𝑅𝑅𝑅𝑅𝑅𝑅𝑅𝑅𝑅 𝑜𝑜𝑜𝑜 𝐸𝐸𝐸𝐸𝐸𝐸𝐸𝐸𝐸𝐸𝐸𝐸 = 0.8364

The company produced $0.8364 of net income for each dollar of common equity employed.

G. Profit Margin
𝑁𝑁𝑁𝑁𝑁𝑁 𝐼𝐼𝐼𝐼𝐼𝐼𝐼𝐼𝐼𝐼𝐼𝐼
𝑃𝑃𝑃𝑃𝑃𝑃𝑃𝑃𝑃𝑃𝑃𝑃 𝑀𝑀𝑀𝑀𝑀𝑀𝑀𝑀𝑀𝑀𝑀𝑀 =
𝑆𝑆𝑆𝑆𝑆𝑆𝑆𝑆𝑆𝑆

$18,400
𝑃𝑃𝑃𝑃𝑃𝑃𝑃𝑃𝑃𝑃𝑃𝑃 𝑀𝑀𝑀𝑀𝑀𝑀𝑀𝑀𝑀𝑀𝑀𝑀 =
$80,000

𝑃𝑃𝑃𝑃𝑃𝑃𝑃𝑃𝑃𝑃𝑃𝑃 𝑀𝑀𝑀𝑀𝑀𝑀𝑀𝑀𝑀𝑀𝑀𝑀 = 0.23

The firm produces $0.23 of net income for each dollar of sales.

130
H. Debt to Assets

𝑇𝑇𝑇𝑇𝑇𝑇𝑇𝑇𝑇𝑇 𝐿𝐿𝐿𝐿𝐿𝐿𝐿𝐿𝐿𝐿𝐿𝐿𝐿𝐿𝐿𝐿𝐿𝐿𝐿𝐿𝐿𝐿
𝐷𝐷𝐷𝐷𝐷𝐷𝐷𝐷 𝑡𝑡𝑡𝑡 𝐴𝐴𝐴𝐴𝐴𝐴𝐴𝐴𝐴𝐴𝐴𝐴 =
𝑇𝑇𝑇𝑇𝑇𝑇𝑇𝑇𝑇𝑇 𝐴𝐴𝐴𝐴𝐴𝐴𝐴𝐴𝐴𝐴𝐴𝐴
10,000
𝐷𝐷𝐷𝐷𝐷𝐷𝐷𝐷 𝑡𝑡𝑡𝑡 𝐴𝐴𝐴𝐴𝐴𝐴𝐴𝐴𝐴𝐴𝐴𝐴 =
32,000

𝐷𝐷𝐷𝐷𝐷𝐷𝐷𝐷 𝑡𝑡𝑡𝑡 𝐴𝐴𝐴𝐴𝐴𝐴𝐴𝐴𝐴𝐴𝐴𝐴 = 0.3125

31.25 percent of the firm's assets have been financed with debt and the remainder is financed
with equity.

I. Equity Multiplier

𝑇𝑇𝑇𝑇𝑇𝑇𝑇𝑇𝑇𝑇 𝐴𝐴𝐴𝐴𝐴𝐴𝐴𝐴𝐴𝐴𝐴𝐴 32,000


𝐸𝐸𝐸𝐸𝐸𝐸𝐸𝐸𝐸𝐸𝐸𝐸 𝑀𝑀𝑀𝑀𝑀𝑀𝑀𝑀𝑀𝑀𝑀𝑀𝑀𝑀𝑀𝑀𝑀𝑀𝑀𝑀 = = = 1.4545
𝐶𝐶𝐶𝐶𝐶𝐶𝐶𝐶𝐶𝐶𝐶𝐶 𝐸𝐸𝐸𝐸𝐸𝐸𝐸𝐸𝐸𝐸𝐸𝐸 $22,000

The company has $1.4545 of total assets for each dollar of common equity.

J. Dupont Analysis

𝑁𝑁𝑁𝑁𝑁𝑁 𝐼𝐼𝐼𝐼𝐼𝐼𝐼𝐼𝐼𝐼𝐼𝐼 𝑆𝑆𝑆𝑆𝑆𝑆𝑆𝑆𝑆𝑆 𝑇𝑇𝑇𝑇𝑇𝑇𝑇𝑇𝑇𝑇 𝐴𝐴𝐴𝐴𝐴𝐴𝐴𝐴𝐴𝐴𝐴𝐴


𝑅𝑅𝑅𝑅𝑅𝑅 = 𝑋𝑋 𝑋𝑋
𝑆𝑆𝑆𝑆𝑆𝑆𝑆𝑆𝑆𝑆 𝑇𝑇𝑇𝑇𝑇𝑇𝑇𝑇𝑇𝑇 𝐴𝐴𝐴𝐴𝐴𝐴𝐴𝐴𝐴𝐴𝐴𝐴 𝐶𝐶𝐶𝐶𝐶𝐶𝐶𝐶𝐶𝐶𝐶𝐶 𝐸𝐸𝐸𝐸𝐸𝐸𝐸𝐸𝐸𝐸𝐸𝐸

18,400 80,000 32,000


𝑅𝑅𝑅𝑅𝑅𝑅 = 𝑋𝑋 𝑋𝑋
80,000 32,000 22,000

𝑅𝑅𝑅𝑅𝑅𝑅 = 0.23 𝑋𝑋 2.5 𝑋𝑋1.454545455

𝑅𝑅𝑅𝑅𝑅𝑅 = 0.8364

131
Solution to Problem 6:

A. Market Value of Equity (Market Capitalization)

𝑀𝑀𝑀𝑀𝑀𝑀𝑀𝑀𝑀𝑀𝑀𝑀 𝑉𝑉𝑉𝑉𝑉𝑉𝑉𝑉𝑉𝑉 𝑜𝑜𝑜𝑜 𝐸𝐸𝐸𝐸𝐸𝐸𝐸𝐸𝐸𝐸𝐸𝐸 = 𝑆𝑆ℎ𝑎𝑎𝑎𝑎𝑎𝑎𝑎𝑎 𝑂𝑂𝑂𝑂𝑂𝑂𝑂𝑂𝑂𝑂𝑂𝑂𝑂𝑂𝑂𝑂𝑂𝑂𝑂𝑂𝑂𝑂 𝑋𝑋 𝑃𝑃𝑃𝑃𝑃𝑃𝑃𝑃𝑃𝑃 𝑃𝑃𝑃𝑃𝑃𝑃 𝑆𝑆ℎ𝑎𝑎𝑎𝑎𝑎𝑎

= 7,925,000,000 X $69.08

= $547,459,000,000

Which is what it would cost to buy the entire company. (Note that we are assuming the 7,925
figure indicated for shares outstanding applies to 2016.

B. Price to Earnings Ratio (PE)

The price to earnings ratio for 2016 equals

𝑀𝑀𝑀𝑀𝑀𝑀𝑀𝑀𝑀𝑀𝑀𝑀 𝑉𝑉𝑉𝑉𝑉𝑉𝑉𝑉𝑉𝑉 𝑜𝑜𝑜𝑜 𝐸𝐸𝐸𝐸𝐸𝐸𝐸𝐸𝐸𝐸𝐸𝐸 𝑂𝑂𝑂𝑂𝑂𝑂𝑂𝑂𝑂𝑂𝑂𝑂ℎ𝑖𝑖𝑖𝑖


𝑃𝑃𝑃𝑃𝑃𝑃𝑃𝑃𝑃𝑃 𝑡𝑡𝑡𝑡 𝐸𝐸𝐸𝐸𝐸𝐸𝐸𝐸𝐸𝐸𝐸𝐸𝐸𝐸𝐸𝐸 𝑅𝑅𝑅𝑅𝑅𝑅𝑅𝑅𝑅𝑅 =
𝑁𝑁𝑁𝑁𝑁𝑁 𝐼𝐼𝐼𝐼𝐼𝐼𝐼𝐼𝐼𝐼𝐼𝐼
547,459,000,000
=
16,798,000,000

= 32.5907 times

The P/E ratio tells us that if earnings do not change from their current levels, it will take 32.5907
years before the firm will earn back the amount of money that we paid for the stock.

C. Total Asset Turnover

85,320,000,000
𝑇𝑇𝑇𝑇𝑇𝑇𝑇𝑇𝑇𝑇 𝐴𝐴𝐴𝐴𝐴𝐴𝐴𝐴𝐴𝐴 𝑇𝑇𝑇𝑇𝑇𝑇𝑇𝑇𝑇𝑇𝑇𝑇𝑇𝑇𝑇𝑇 =
193,694,000,000

𝑇𝑇𝑇𝑇𝑇𝑇𝑇𝑇𝑇𝑇 𝐴𝐴𝐴𝐴𝐴𝐴𝐴𝐴𝐴𝐴 𝑇𝑇𝑇𝑇𝑇𝑇𝑇𝑇𝑇𝑇𝑇𝑇𝑇𝑇𝑇𝑇 = 0.4405

So in this case the company had $0.4405 of sales for each dollar of assets that it had in 2016.

D. Inventory Turnover

85,320,000,000
𝐼𝐼𝐼𝐼𝐼𝐼𝐼𝐼𝐼𝐼𝐼𝐼𝐼𝐼𝐼𝐼𝐼𝐼 𝑇𝑇𝑇𝑇𝑇𝑇𝑇𝑇𝑇𝑇𝑇𝑇𝑇𝑇𝑇𝑇 =
2,251,000,000

= 37.90

In this case, the company had $37.90 of sales for each dollar of inventory it had.

E. Return on Assets

𝑁𝑁𝑁𝑁𝑁𝑁 𝐼𝐼𝐼𝐼𝐼𝐼𝐼𝐼𝐼𝐼𝐼𝐼
𝑅𝑅𝑅𝑅𝑅𝑅𝑅𝑅𝑅𝑅𝑅𝑅 𝑜𝑜𝑜𝑜 𝐴𝐴𝐴𝐴𝐴𝐴𝐴𝐴𝐴𝐴𝐴𝐴 =
𝑇𝑇𝑇𝑇𝑇𝑇𝑇𝑇𝑇𝑇 𝐴𝐴𝐴𝐴𝐴𝐴𝐴𝐴𝐴𝐴𝐴𝐴

132
16,798,000,000
𝑅𝑅𝑅𝑅𝑅𝑅𝑅𝑅𝑅𝑅𝑅𝑅 𝑜𝑜𝑜𝑜 𝐴𝐴𝐴𝐴𝐴𝐴𝐴𝐴𝐴𝐴𝐴𝐴 =
193,694,000,000

= 0.0867

The company had $0.0867 of net income for each dollar of assets.

F. Return on Equity

Tells you how much net income you had for each dollar of equity invested in the company.

16,798,000,000
𝑅𝑅𝑅𝑅𝑅𝑅𝑅𝑅𝑅𝑅𝑅𝑅 𝑜𝑜𝑜𝑜 𝐸𝐸𝐸𝐸𝐸𝐸𝐸𝐸𝐸𝐸𝐸𝐸 =
71,997,000,000

𝑅𝑅𝑅𝑅𝑅𝑅𝑅𝑅𝑅𝑅𝑅𝑅 𝑜𝑜𝑜𝑜 𝐸𝐸𝐸𝐸𝐸𝐸𝐸𝐸𝐸𝐸𝐸𝐸 = 0.2333

This says the company earned $0.2333 for each dollar of common equity utilized.

G. Profit Margin

Profit Margin tells you how much net income the firm is producing for each dollar of sales.

𝑁𝑁𝑁𝑁𝑁𝑁 𝐼𝐼𝐼𝐼𝐼𝐼𝐼𝐼𝐼𝐼𝐼𝐼
𝑃𝑃𝑃𝑃𝑃𝑃𝑃𝑃𝑃𝑃𝑃𝑃 𝑀𝑀𝑀𝑀𝑀𝑀𝑔𝑔𝑖𝑖𝑖𝑖 =
𝑆𝑆𝑆𝑆𝑆𝑆𝑆𝑆𝑆𝑆

$16,798,000,000
𝑃𝑃𝑃𝑃𝑃𝑃𝑃𝑃𝑃𝑃𝑃𝑃 𝑀𝑀𝑀𝑀𝑀𝑀𝑀𝑀𝑀𝑀𝑀𝑀 =
$85,320,000,000

𝑃𝑃𝑃𝑃𝑃𝑃𝑃𝑃𝑃𝑃𝑃𝑃 𝑀𝑀𝑀𝑀𝑀𝑀𝑀𝑀𝑀𝑀𝑀𝑀 = 0.19688

So the firm produces $0.19688 of net income for each dollar of sales.

H. Debt to Assets

Debt to Assets tells which portion of the firm's assets are financed by debt.

𝑇𝑇𝑇𝑇𝑇𝑇𝑇𝑇𝑇𝑇 𝐿𝐿𝐿𝐿𝐿𝐿𝐿𝐿𝐿𝐿𝐿𝐿𝐿𝐿𝐿𝐿𝐿𝐿𝐿𝐿𝐿𝐿
𝐷𝐷𝐷𝐷𝐷𝐷𝐷𝐷 𝑡𝑡𝑡𝑡 𝐴𝐴𝐴𝐴𝐴𝐴𝐴𝐴𝐴𝐴𝐴𝐴 =
𝑇𝑇𝑇𝑇𝑇𝑇𝑇𝑇𝑇𝑇 𝐴𝐴𝐴𝐴𝐴𝐴𝐴𝐴𝐴𝐴𝐴𝐴
121,697,000,000
𝐷𝐷𝐷𝐷𝐷𝐷𝐷𝐷 𝑡𝑡𝑡𝑡 𝐴𝐴𝐴𝐴𝐴𝐴𝐴𝐴𝐴𝐴𝐴𝐴 =
193,694,000,000

𝐷𝐷𝐷𝐷𝐷𝐷𝐷𝐷 𝑡𝑡𝑡𝑡 𝐴𝐴𝐴𝐴𝐴𝐴𝐴𝐴𝐴𝐴𝐴𝐴 = 0.62830

So, 62.830 percent of the firm's assets have been financed with debt and the remainder is financed
with equity.

133
I. Payout Ratio

Can be calculated with either basic or diluted earnings per share.

𝐷𝐷𝐷𝐷𝐷𝐷𝐷𝐷𝐷𝐷𝐷𝐷𝐷𝐷𝐷𝐷𝐷𝐷 𝑃𝑃𝑃𝑃𝑃𝑃𝑃𝑃 𝐷𝐷𝐷𝐷𝐷𝐷𝐷𝐷𝐷𝐷𝐷𝐷𝐷𝐷𝐷𝐷𝐷𝐷 𝑝𝑝𝑝𝑝𝑝𝑝 𝑆𝑆ℎ𝑎𝑎𝑎𝑎𝑎𝑎


𝑃𝑃𝑃𝑃𝑃𝑃𝑃𝑃𝑃𝑃𝑃𝑃 𝑅𝑅𝑎𝑎𝑎𝑎𝑎𝑎𝑎𝑎 = =
𝑁𝑁𝑁𝑁𝑁𝑁 𝐼𝐼𝐼𝐼𝐼𝐼𝐼𝐼𝐼𝐼𝐼𝐼 𝑁𝑁𝑁𝑁𝑁𝑁 𝐼𝐼𝐼𝐼𝐼𝐼𝐼𝐼𝐼𝐼𝐼𝐼 𝑝𝑝𝑝𝑝𝑝𝑝 𝑆𝑆ℎ𝑎𝑎𝑎𝑎𝑎𝑎 (𝐸𝐸𝐸𝐸𝐸𝐸𝐸𝐸𝐸𝐸𝐸𝐸𝐸𝐸𝐸𝐸 𝑃𝑃𝑃𝑃𝑃𝑃 𝑆𝑆ℎ𝑎𝑎𝑎𝑎𝑎𝑎)

$1.44
𝑃𝑃𝑃𝑃𝑃𝑃𝑃𝑃𝑃𝑃𝑃𝑃 𝑅𝑅𝑅𝑅𝑅𝑅𝑅𝑅𝑅𝑅 = = 0.6725
$2.12

This indicates that we paid 67.25 percent of our net income to the stockholders as a dividend.

J. Current Ratio

𝐶𝐶𝐶𝐶𝐶𝐶𝐶𝐶𝐶𝐶𝐶𝐶𝐶𝐶 𝐴𝐴𝐴𝐴𝐴𝐴𝐴𝐴𝐴𝐴𝐴𝐴
𝐶𝐶𝐶𝐶𝐶𝐶𝐶𝐶𝐶𝐶𝐶𝐶𝐶𝐶 𝑅𝑅𝑅𝑅𝑅𝑅𝑅𝑅𝑅𝑅 =
𝐶𝐶𝐶𝐶𝐶𝐶𝐶𝐶𝐶𝐶𝐶𝐶𝐶𝐶 𝐿𝐿𝐿𝐿𝐿𝐿𝐿𝐿𝐿𝐿𝐿𝐿𝐿𝐿𝐿𝐿𝐿𝐿𝐿𝐿𝐿𝐿

139,660,000,000
𝐶𝐶𝐶𝐶𝐶𝐶𝐶𝐶𝐶𝐶𝐶𝐶𝐶𝐶 𝑅𝑅𝑅𝑅𝑅𝑅𝑅𝑅𝑅𝑅 =
59,357,000,000

= 2.3529
So, the company has $2.3529 dollars of current assets for each dollar of current liabilities.

K. Equity Multiplier

𝑇𝑇𝑇𝑇𝑇𝑇𝑇𝑇𝑇𝑇 𝐴𝐴𝐴𝐴𝐴𝐴𝐴𝐴𝐴𝐴𝐴𝐴 $193,694,000,000


𝐸𝐸𝐸𝐸𝐸𝐸𝐸𝐸𝐸𝐸𝐸𝐸 𝑀𝑀𝑀𝑀𝑀𝑀𝑀𝑀𝑀𝑀𝑀𝑀𝑀𝑀𝑀𝑀𝑀𝑀𝑀𝑀 = = = 2.6903
𝐶𝐶𝐶𝐶𝐶𝐶𝐶𝐶𝐶𝐶𝐶𝐶 𝐸𝐸𝐸𝐸𝐸𝐸𝐸𝐸𝐸𝐸𝐸𝐸 $71,997,000,000

The company has $2.6903 of total assets for each dollar of common equity invested in the firm.

L. Dupont Analysis

𝑁𝑁𝑁𝑁𝑁𝑁 𝐼𝐼𝐼𝐼𝐼𝐼𝐼𝐼𝐼𝐼𝐼𝐼 𝑆𝑆𝑆𝑆𝑆𝑆𝑆𝑆𝑆𝑆 𝑇𝑇𝑇𝑇𝑇𝑇𝑇𝑇𝑇𝑇 𝐴𝐴𝐴𝐴𝐴𝐴𝐴𝐴𝐴𝐴𝐴𝐴


𝑅𝑅𝑅𝑅𝑅𝑅 = 𝑋𝑋 𝑋𝑋
𝑆𝑆𝑆𝑆𝑆𝑆𝑆𝑆𝑆𝑆 𝑇𝑇𝑇𝑇𝑇𝑇𝑇𝑇𝑇𝑇 𝐴𝐴𝐴𝐴𝐴𝐴𝐴𝐴𝐴𝐴𝐴𝐴 𝐶𝐶𝐶𝐶𝐶𝐶𝐶𝐶𝐶𝐶𝐶𝐶 𝐸𝐸𝐸𝐸𝐸𝐸𝐸𝐸𝐸𝐸𝐸𝐸

16,798,000,000 85,320,000,000 193,694,000,000


𝑅𝑅𝑅𝑅𝑅𝑅 = 𝑋𝑋 𝑋𝑋
85,320,000,000 193,694,000,000 71,997,000,000

𝑅𝑅𝑅𝑅𝑅𝑅 = 0.196882325 𝑋𝑋 0.440488606 𝑋𝑋 2.690306541

𝑅𝑅𝑅𝑅𝑅𝑅 = 0.2333

134
Solution to Problem 7. To solve this problem, you should use the information you have available to
derive the missing data.
500,000
a. We know Return on Assets (ROA) = 0.50 and we know: 𝑅𝑅𝑅𝑅𝑅𝑅 = 0.5 = ,
𝑇𝑇𝑇𝑇𝑇𝑇𝑇𝑇𝑇𝑇 𝐴𝐴𝐴𝐴𝐴𝐴𝐴𝐴𝐴𝐴𝐴𝐴

Solving this equation for the missing variable we learn that: Total Assets = $1,000,000

b. We know that Total Asset Turnover (TAT) = 0.25, and we know that:

𝑆𝑆𝑆𝑆𝑆𝑆𝑆𝑆𝑆𝑆 𝑆𝑆𝑆𝑆𝑆𝑆𝑆𝑆𝑆𝑆
𝑇𝑇𝑇𝑇𝑇𝑇 = 2.5 = =
𝑇𝑇𝑇𝑇𝑇𝑇𝑇𝑇𝑇𝑇 𝐴𝐴𝐴𝐴𝐴𝐴𝐴𝐴𝐴𝐴𝐴𝐴 $1,000,000

Solving for the missing variable we learn that Sales = $2,500,000


𝑁𝑁𝑁𝑁𝑁𝑁 𝐼𝐼𝐼𝐼𝐼𝐼𝐼𝐼𝐼𝐼𝐼𝐼 500000
c. 𝑃𝑃𝑃𝑃𝑃𝑃𝑃𝑃𝑃𝑃𝑃𝑃 𝑀𝑀𝑀𝑀𝑀𝑀𝑀𝑀𝑀𝑀𝑀𝑀 = = = 0.20
𝑆𝑆𝑆𝑆𝑆𝑆𝑆𝑆𝑆𝑆 2500000

d. We know that Return on Equity (ROE) = 0.25, and we know Net Income equals $500,000.

$500,000
𝑅𝑅𝑅𝑅𝑅𝑅 = 0.25 = . Solving for the missing variable tells us Common Equity =
𝐶𝐶𝐶𝐶𝐶𝐶𝐶𝐶𝐶𝐶𝐶𝐶 𝐸𝐸𝐸𝐸𝐸𝐸𝐸𝐸𝐸𝐸𝐸𝐸
$2,000,000

e. We know the Current Ratio (CR) = 4. We know that Current Assets = $25,000.
𝐶𝐶𝐶𝐶𝐶𝐶𝐶𝐶𝐶𝐶𝐶𝐶𝐶𝐶 𝐴𝐴𝐴𝐴𝐴𝐴𝐴𝐴𝐴𝐴𝐴𝐴 𝐶𝐶𝐶𝐶𝐶𝐶𝐶𝐶𝐶𝐶𝐶𝐶𝐶𝐶 𝐴𝐴𝐴𝐴𝐴𝐴𝐴𝐴𝐴𝐴𝐴𝐴
We know that 𝐶𝐶𝐶𝐶 = 4 = = .
𝐶𝐶𝐶𝐶𝐶𝐶𝐶𝐶𝐶𝐶𝐶𝐶𝐶𝐶 𝐿𝐿𝐿𝐿𝐿𝐿𝐿𝐿𝐿𝐿𝐿𝐿𝐿𝐿𝐿𝐿𝐿𝐿𝐿𝐿𝐿𝐿 $25,000

Solving for the missing variable we learn that Current Assets = $100,000

f. We know that Sales = $2,500,000. We know that Inventory Turnover = 3.

𝑆𝑆𝑆𝑆𝑆𝑆𝑆𝑆𝑆𝑆 $2,500,000
We know that 𝐼𝐼𝐼𝐼 = 3 = =
𝐵𝐵𝐵𝐵𝐵𝐵𝐵𝐵𝐵𝐵𝐵𝐵𝐵𝐵𝐵𝐵𝐵𝐵 𝑜𝑜𝑜𝑜 𝑌𝑌𝑌𝑌𝑌𝑌𝑌𝑌 𝐼𝐼𝐼𝐼𝐼𝐼𝐼𝐼𝐼𝐼𝐼𝐼𝐼𝐼𝐼𝐼𝐼𝐼 𝐵𝐵𝐵𝐵𝐵𝐵𝐵𝐵𝐵𝐵𝐵𝐵𝐵𝐵𝐵𝐵𝐵𝐵 𝑜𝑜𝑜𝑜 𝑌𝑌𝑌𝑌𝑌𝑌𝑌𝑌 𝐼𝐼𝐼𝐼𝐼𝐼𝐼𝐼𝐼𝐼𝐼𝐼𝐼𝐼𝐼𝐼𝐼𝐼

Solving for the missing variable we learn that Beginning of the Year Inventory = $833,333.33

135
Chapter 4
Forecasting Financial Statements

136
Forecasting Financial Statements

In order to estimate the future (pro-forma) financial statements of a


company, you must make estimates about the size of various accounts. In
this section, we will show you how to estimate the amounts on your future
financial statements.

Step 1: Without experience in an industry it is difficult to determine how


much each item on your financial statements will be. In order to
make accurate determinations, we can rely on the experiences of
others as a basis for our estimates.

Information about the experiences of others can be obtained from


the Risk Management Association (RMA), Annual Statement
Studies. These studies can be found in our library. There is a new
study in each year. You should obtain the most recent study.

Step 2: Identify the industry that your firm is in. Sometimes there is not a
report on the exact industry that you wish to be in . In these cases,
you should get the report for the industry you believe is the closest
to that in which your firm will operate. Suppose our company is a
Retail Jewelry store.

Step 3: Obtain the reports for the industry that you will operate in and use
this report to develop your financial statements.

137
Sample Industry Data

Jewelry Industry

Historical Average Current Data Sorted by Sales Current Data Sorted by Total Assets
Sales 0-
Data 1M 1M-5M T.A. 0-500 500-2M

% Assets % % % %
10.1 Cash and Equivalents 9.5 11.4 10.4 9.5
8.9 Trade Receivables 10.5 9.5 9.1 10.1
25.5 Inventory 23.5 25.9 31.1 24.1
0.1 All Other Current 0.2 0.4 0.1 0.2
44.6 Total Current 43.7 47.2 50.7 43.9
35.4 Fixed Assets (Net) 41.1 38.8 35.5 41.5
12.5 Intangibles (Net) 2.2 1.5 1 0.5
7.5 All Other Non-Current 13.0 12.5 12.8 14.1
100 Total 100 100 100 100

Liabilities
12.5 Notes Payable-Short Term 10.5 9.5 11.5 8.5
10.7 Current Mat. LTD 4.5 3.5 2.5 5.5
15.3 Trade Payables 17.7 14.4 15.3 15.1
0.5 Income Taxes payable 0.9 0.7 0.2 1.3
1.2 All Other current 14.5 11.4 8.5 7.2
40.2 Total Current 48.1 39.5 38 37.6
35 Long Term Debt 28.5 45.3 35.5 38.4
0.9 Deferred Taxes 0.5 0.8 0.2 1.1
1.5 All Other Non-Current 4.1 2.4 3.1 3.7
22.4 Net Worth 18.8 12 23.2 19.2
100 Total Liab. & Net Worth 100 100 100 100

Income Data
100 Net Sales 100 100 100 100
35.5 Gross Profit 36.2 37.3 35.5 40.1
28.2 Operating Expenses 30.1 27.5 31.2 28.5
7.3 Operating Profit 6.1 9.8 4.3 11.6
4.4 All Other Expenses 3.3 4.1 2.4 2.1
2.9 Profit Before Taxes 2.8 5.7 1.9 9.5

Ratios
4.3 Sales/Total Assets 3.8 4.6 4.0 4.1

138
Step 4: Create your Financial Statements

Part A: Decide on a size for your firm. You may do this based on
Sales or Total Assets.

Suppose that you plan to have a company with $400,000 of


sales per year.

Part B: Select the column that corresponds with the size of your
firm. In this case, our company will have sales between $0
and $1,000,000, so we should select the first column of
current data sorted by sales for our analysis.

Part C: Estimate the Income Statement of the Company

So in this case, the following income statement figures are


relevant from the RMA figures

Gross Profit 36.2%


Operating Expenses 30.1%
Operating Profit 6.1%
All Other Expenses 3.3%
Profit Before Taxes 2.8%

Using these figures, we can estimate our income statement as


follows:

Sales $400,000
Cost of Goods Sold (Sales – Gross Profit) $255,200
Gross Profit (36.2%) $144,800
Operating Expenses (30.1%) $120,400
Earnings Before Interest and Taxes (6.1%)
(Operating Profit) $24,400
Interest Expense (3.3%)
(Other Expenses) $13,200
Earnings Before Tax (2.8%) $11,200

Unfortunately, the RMA ratios do not provide sufficient


information to estimate the taxes that the firm will have to pay.

139
Part D: Estimate the Assets of the Company. Because you have
estimated the sales of the company, you can also estimate the
Total Assets of the Company. To do this, observe the
Sales/Total Assets ratio reported in RMA. In the case of the
Jewelry industry the ratio is 3.8. (Note that when using
actual RMA data there will be three numbers for each ratio.
You will generally want to select the middle number, which is
the ratio for the middle 1/3 of companies)

Next, you use this information to compute the assets of the


company.

3.8 = Sales/Total Assets

Because we have already estimated our sales, we can fill in


part of the equation.

3.8 = $400,000/Total Assets

Now if we solve for total assets:

Total Assets = $400,000/3.8

Total Assets = $105,263.16

140
Part E: Estimate the Balance Sheet of the Company

From the RMA we note that:

Assets
Cash and Equivalents 9.5% $
Trade Receivables 10.5% $
Inventory 23.5% $
Other Current Assets 0.2% $
Total Current Assets 43.7% $
Fixed Assets (Net) 41.1% $
Intangibles 2.2% $
Other Non-Current Assets 13.0% $
Total Assets 100% $

Liabilities

Notes Payable 10.5% $


Current Maturity LT Debt 4.5% $
Accounts Payable 17.7% $
Income Taxes Payable 0.9% $
All Other Current 14.5% $
Total Current Liabilities 48.1% $
Long Term Debt 28.5% $
Deferred Taxes 0.5% $
All other Non-Current Liabilities 4.1% $
*Common Stock & Retained Earnings 18.8% $
Total Liabilities and Equity 100%

*. Note that RMA refers to Net Worth. This is exactly the same thing as
Common Stock and Retained Earnings in the language we have
learned in class.

141
With this information, and knowing that our total assets will equal
$105,263.16. We can estimate our balance sheet in this way:

Assets
Cash and Equivalents 9.5% $10,000.00
Trade Receivables 10.5% $11,052.63
Inventory 23.5% $24,736.84
Other Current Assets 0.2% $210.53
Total Current Assets 43.7% $46,000.00
Fixed Assets (Net) 41.1% $43,263.16
Intangibles 2.2% $2,315.79
Other Non-Current Assets 13.0% $13,684.21
Total Assets 100% $105,263.16

Liabilities

Notes Payable 10.5% $11,052.63


Current Maturity LT Debt 4.5% $4,736.84
Accounts Payable 17.7% $18,631.58
Income Taxes Payable 0.9% $947.37
All Other Current 14.5% $15,263.16
Total Current Liabilities 48.1% $50,631.58
Long Term Debt 28.5% $30,000.00
Deferred Taxes 0.5% $526.32
All other Non-Current Liabilities 4.1% $4,315.79
Common Stock and Retained Earnings 18.8% $19,789.47
Total Liabilities and Equity 100% $105,263.16

Note that you sometimes need to do a little rounding to make your


statements work out as the RMA’s have been rounded.

If you are starting a business and your estimates differ substantially from
what you obtain from this approach, you should explain the difference.

142
Now suppose that instead of having an estimate of sales, you know only that
you have $40,000 to invest in the business. Suppose further that you want to
retain full ownership of the business. How can you estimate the financial
statements in this case?

In this case we begin with the balance sheet and work our way back to the
income statement/

Starting again from the RMA information. However, in this case, we will
use the current data sorted by assets in RMA to make our estimates. To do
this, we must first make an estimate of the total assets of the firm based on
our plan to invest $40,000 into the company. In this case, our firm will
certainly have below $500,000 of total assets, so we should use the first
column of the data.

Assets
Cash and Equivalents 10.4% $
Trade Receivables 9.1% $
Inventory 31.1% $
Other Current Assets 0.1% $
Total Current Assets 50.7% $
Fixed Assets (Net) 35.5% $
Intangibles 1.0% $
Other Non-Current Assets 12.8% $
Total Assets 100% $

Liabilities

Notes Payable 11.5% $


Current Maturity LT Debt 2.5% $
Accounts Payable 15.3% $
Income Taxes Payable 0.2% $
All Other Current 8.5% $
Total Current Liabilities 38.0% $
Long Term Debt 35.5% $
Deferred Taxes 0.2% $
All other Non-Current Liabilities 3.1% $
Common Stock & Retained Earnings 23.2% $40,000.00
Total Liabilities and Equity 100.00%

143
First, estimate the total assets:

Total Assets = Common Stock and Ret. Earn./percent of funds from


this source

= $40,000/.232= $172,413.79

You can then compute the remaining values as a percentage of the total.

Assets
Cash and Equivalents 10.4% $17,931.0
Trade Receivables 9.1% $15,689.65
Inventory 31.1% $53,620.69
Other Current Assets .1% $172.41
Total Current Assets 50.7% $87413.79
Fixed Assets (Net) 35.5% $61,206.90
Intangibles 1.0% $1,724.14
Other Non-Current Assets 12.8% $22,068.97
Total Assets 100% $172,413.79

Liabilities

Notes Payable 11.5% $19,827.59


Current Maturity LT Debt 2.5% $4,310.34
Accounts Payable 15.3% $26,379.31
Income Taxes Payable 0.2% $344.83
All Other Current 8.5% $14,655.17
Total Current Liabilities 38.0% $65,517.24
Long Term Debt 35.5% $61,206.90
Deferred Taxes 0.2% $344.83
All other Non-Current Liabilities 3.1% $5,344.83
Common Stock and Retained Earnings 23.2% $40,000.00
Total Liabilities and Equity 100% $172,413.79

144
Next, you need to estimate the sales of the firm. We can do this based
on the Sales/Total Assets Ratio, just as we did before. In this case, we
know the total assets ($172,413.79) and must estimate the sales of the
firm. We also know that other firms have a Sales/total asset ratio of
4.0. Thus:

4.0 = Sales/$172,413.79

Sales = $172,413.79*4.0 = $689,655.16

Using this sales figure and our industry information, we can estimate
our income statement as follows:

Part C: Estimate the Income Statement of the Company

So in this case, the following income statement figures are


Relevant from the RMA figures

Gross Profit 35.5%


Operating Expenses 31.2%
Operating Profit 4.3%
All Other Expenses 2.4%
Profit Before Taxes 1.9%

Using these figures, we can estimate our income statement as


follows:

Sales $689,655.16
Cost of Goods Sold (Sales – Gross Profit) $444,827.58
Gross Profit (35.5%) $244,827.58
Operating Expenses (31.2%) $215,172.41
Earnings Before Interest and Taxes (4.3%)
(Operating Profit) $29,655.17
Interest Expense (2.4%)
(Other Expenses) $16,551.72
Earnings Before Tax (1.9%) $13,103.45

Unfortunately, the RMA ratios do not provide sufficient


information to estimate the taxes that the firm will have to pay.

145
Chapter 4
Forecasting Financial Statements
Homework Assignment

146
1. You wish to start a firm that will manufacture Lawn Mowers. You have
classified your firm as being in the Lawn Mower Manufacturing Business. You
have obtained the Risk Management Associates (RMA) Industry Financial
Statement Information on the following pages.

Using the RMA industry financial information and your sales estimate of
$2,000,000 per year, as we did in class, estimate the financial statements of your
firm.

2. You wish to start a firm that will manufacture Lawn Mowers. You have
classified your firm as being in the Lawn Mower Manufacturing Business. You
have obtained the Robert Morris and Associates (RMA) Industry Financial
Statement Information on the following pages.

Using the RMA industry financial information and your plan to invest $150,000
from your own pocket into the business, as we did in class, estimate the financial
statements of your firm.

147
Sample Industry Data

Lawn Mower Manufacturing

Historical Average Current Data Sorted by Sales Current Data Sorted by Total Assets
Sales 0-
Data 1M 1M-5M T.A. 0-500 500-2M

% Assets % % % %
5.5 Cash and Equivalents 6.5 4.5 7.3 5.4
35.1 Trade Receivables 28.7 25.8 30.1 28.4
25 Inventory 33.3 29.4 30.1 36.5
0.3 All Other Current 0.1 0.3 0 0.1
65.9 Total Current 68.6 60 67.5 70.4
30.1 Fixed Assets (Net) 28.7 33.2 26.3 27.1
0.2 Intangibles (Net) 0.1 0.6 0.3 0.2
3.8 All Other Non-Current 2.6 6.2 5.9 2.3
100 Total 100 100 100 100

Liabilities
5.4 Notes Payable-Short Term 6.1 8.2 7.2 6.9
4.5 Current Mat. LTD 3.6 5.5 4.2 5.6
25.5 Trade Payables 27.6 24.6 22.1 28.6
0.3 Income Taxes payable 0.2 0.1 0.4 0.3
0.1 All Other current 0.3 0.5 0.7 0.2
35.8 Total Current 37.8 38.9 34.6 41.6
21.5 Long Term Debt 33.1 24.2 25.1 31.6
4.6 Deferred Taxes 5.1 3.3 4.9 5.7
5.7 All Other Non-Current 7.9 4.3 5.1 6.3
32.4 Net Worth 16.1 29.3 30.3 14.8
100 Total Liab. & Net Worth 100 100 100 100

Income Data
100 Net Sales 100 100 100 100
17.1 Gross Profit 20.4 22.3 17.5 23.5
15.1 Operating Expenses 11.5 14.5 14.5 18.3
2 Operating Profit 8.9 7.8 3 5.2
0.2 All Other Expenses 0.1 0.1 0.1 0.1
1.8 Profit Before Taxes 8.8 7.7 2.9 5.1

Ratios
2.4 Sales/Total Assets 2.1 3.2 2.7 2.9

148
Chapter 4 Homework Solutions

Question 1

In this case, the following income statement figures are relevant from the RMA figures

Gross Profit 22.3%


Operating Expenses 14.5%
Operating Profit 7.8%
All Other Expenses 0.1%
Profit Before Taxes 7.7%

Using these figures, we can estimate our income statement as follows:

Sales $2,000,000
Cost of Goods Sold (Sales – Gross Profit) $1,554,000
Gross Profit (22.3%) $446,000
Operating Expenses (14.5%) $290,000
Earnings Before Interest and Taxes (7.8%)
(Operating Profit) $156,000
Interest Expense (0.1%)
(Other Expenses) $2,000
Earnings Before Tax (7.7%) $154,000

Estimate the Assets of the Company.


The Sales/Total Assets ratio reported in RMA for this industry is 3.2.

3.2 = Sales/Total Assets

3.2 = $2,000,000/Total Assets

Now if we solve for total assets:

Total Assets = $2,000,000/3.2

Total Assets = $625,000.00

149
Estimate the Balance Sheet of the Company

From the RMA we note that:

Assets
Cash and Equivalents 4.5% $28,125
Trade Receivables 25.8% $161,250
Inventory 29.4% $183,750
Other Current Assets 0.3% $1,875
Total Current Assets 60.0% $375,000
Fixed Assets (Net) 33.2% $207,500
Intangibles 0.6% $3,750.00
Other Non-Current Assets 6.2% $38,750
Total Assets 100% $625,000

Liabilities

Notes Payable 8.2% $51,250


Current Maturity LT Debt 5.5% $34,375
Accounts Payable 24.6% $153,750
Income Taxes Payable 0.1% $625
All Other Current 0.5% $3.125
Total Current Liabilities 38.9% $243,125
Long Term Debt 24.2% $151,250
Deferred Taxes 3.3% $20,625
All other Non-Current Liabilities 4.3% $26,875
Common Stock and Retained Earnings 29.3% $183,125
Total Liabilities and Equity 100% $625,000

150
Question 2:

In this case, we only know that we will invest $150,000 of our own money into the
business.

Starting from the RMA information for the balance sheet with total assets between zero
and $500,000

First, estimate the total assets:

Total Assets = Common Stock and Ret. Earn./percent of funds from


this source

= $150,000/.303= $495,049.51

You can then compute the remaining values as a percentage of the total.

Assets
Cash and Equivalents 7.3% $36,138.61
Trade Receivables 30.1% $149,009.90
Inventory 30.1% $149,009.90
Other Current Assets .0% $0
Total Current Assets 67.5% $334,158.42
Fixed Assets (Net) 26.3% $130,198.02
Intangibles 0.3% $1,485.15
Other Non-Current Assets 5.9% $29,207.92
Total Assets 100% $495,049.51

Liabilities

Notes Payable 7.2% $35,643.56


Current Maturity LT Debt 4.2% $20,792.08
Accounts Payable 22.1% $109,405.94
Income Taxes Payable 0.4% $1,980.20
All Other Current 0.7% $3,465.35
Total Current Liabilities 34.6% $171,287.13
Long Term Debt 25.1% $124,257.43
Deferred Taxes 4.9% $24,257.43
All other Non-Current Liabilities 5.1% $25,247.53
Common Stock and Retained Earnings 30.3% $150,000.00
Total Liabilities and Equity 100% $495,049.51

151
To estimate the sales of the firm. We do this based on the Sales/Total Assets
Ratio, just as we did before. In this case, we know the total assets ($495,049.51)
and must estimate the sales of the firm. We also know that other firms have a
Sales/total asset ratio of 2.7. Thus:

2.7 = Sales/$495,049.51

Sales = $495,049.51*2.7 = $1,336,633.68

Using this sales figure and our industry information, we can estimate our income
statement as follows:

Part C: Estimate the Income Statement of the Company

So in this case, the following income statement figures are


Relevant from the RMA figures

We can estimate our income statement as follows:

Gross Profit 17.5%


Operating Expenses 14.5%
Operating Profit 3.0%
All Other Expenses 0.1%
Profit Before Taxes 2.9%

Using these figures, we can estimate our income statement as follows:

Sales $1,336,633.68
Cost of Goods Sold (Sales – Gross Profit) $1,102,722.79
Gross Profit (17.5%) $233,910.89
Operating Expenses (14.5%) $193,811.88
Earnings Before Interest and Taxes (3.0%)
(Operating Profit) $40,099.01
Interest Expense (0.1%)
(Other Expenses) $1,336.63
Earnings Before Tax (2.9%) $38,762.38

152
CHAPTER 5

The Financial Environment:


Markets, Institutions and
Interest Rates

153
Chapter 5: The Financial
Environment: Markets, Institutions,
and Interest Rates
In this chapter you will learn about financial markets, how and
where firms can obtain money, Interest Rates, and Theories of
short term versus long-term borrowing.

Financial Markets
Definition of a Market

A place or system by which buyers and sellers come together


to exchange goods and or services.

Classification of Markets

I Physical Asset Markets versus Financial Markets

Physical Assets: Something that you can lay your hands on.
a. Art
b. Real Estate
c. Cars

Financial Assets: Paper Assets that represent a claim on


some economic entity

a. Currency
b. Shares of Stocks
c. Deposits in a Commercial Bank

The Role of Financial Markets is to facilitate the efficient


exchange of funds between borrowers and lenders.

154
Classification of Financial Markets

I. Money Markets versus Capital Markets

In Money Markets, short term debt instruments are


traded.

In Capital Markets, long term debt instruments and


equity instruments are traded.

II. Spot versus Futures, Forward, and Options Markets

In a spot market, cash is exchanged for the


immediate delivery of goods or services.

In a Futures, Forward, or Option Market, an


agreement is entered into for the future delivery of
some good or service at a preset price.

III. Primary Versus Secondary markets

In a primary market securities are traded for the first


time.

An Initial Public Offering (IPO) is a company


publicly issuing securities for the first time.

A seasoned offering is a company that is already


publicly traded seeking additional funding. A
company seeking new capital to finance an
expansion project.

155
Methods for Transferring Capital From Savers to Businesses’

1. Direct Transfers

- The firm Deals Directly with Savers

Stocks or Bonds
-------------------------------------
Business Savers
----------------------------------
Money

2. Indirect Direct Transfers

-- A Middle Person Operates between the Firm and Savers

a. Using an Investment Banker

Stocks or Bonds Stocks or Bonds


------------------- --  -- -- --------------
Business Investment Savers
Banker
-------------- ----------------
Money Money

b. Using a Financial Intermediary

Stocks or Bonds Intermed. Securities


----------------- ----------------
Business Financial Savers
Intermediary
-------------- ----------------
Money Money

156
Primary Markets
Three Primary Ways Capital is Transferred Between Savers and
Borrowers

Direct Transfer

Firms deal directly with Savers

Indirect Transfers

A. Using An Investment Banker

In this case, firms use an investment banker as an


intermediary.

Savers receive the securities of the firm

Investment Bank Examples:


Goldman Sachs
Morgan Stanley, Dean Witter
Smith Barney
Salomon

B. Using a Financial Intermediary

In this case, firms obtain money directly from the


financial intermediary.

The financial intermediary holds the securities of the


firm.

The financial intermediary issues its own securities


to Savers.

157
Financial Intermediaries

1. Commercial Banks

Traditionally Accept Deposits and Make Commercial


Loans.

2. Savings and Loans

Traditionally serve small savers and make housing


loans

3. Mutual Savings Banks

Similar to a Savings and Loan

4. Credit Unions

Cooperative Association whose members are supposed


to have some common bond.

5. Life Insurance Companies

Accept Premiums from individuals, invest those premiums,


and ultimately pay a benefit to the policy beneficiary. Also
do tax-deferred savings plans today.

6. Mutual Funds

Pool the funds of savers to make investments.

7. Pension Funds

Retirement plans funded by Corporations and Government


agencies.

158
Changing Role of Financial Institutions

1. Historically Highly specialized as to the products they


offer and the geographic areas served.

Today, they operate world-wide and offer a variety of


products.

2. Financial Institutions have historically been heavily


regulated.

Today, they are regulated to a much lesser degree

3. Technological Changes have altered the operating


environment.

a. ATM machines allowed banks to offer services at a


variety of locations.

b. Internet allows banks to offer services anywhere


there is a computer and an internet connection.

The result is the creation of very large integrated financial


institutions that offer a large variety of products in a large
geographic area.

Secondary Markets
Once a security has initially been issued, it frequently trades
on a secondary market.

Examples of Secondary Markets

1. New York Stock Exchange


2. Nasdaq-Amex
3. London Stock Exchange
4. Tokyo Stock Exchange

159
The Cost of Money
Borrowed Money

Interest – Rent on Borrowed Money

Equity Investments

The rent is the return that investors expect to receive in the form of
dividends and capital gains.

Factors that Affect Interest rate levels

1. Production Opportunities

When firms and individuals have better production


opportunities, they are willing to pay more to obtain the
necessary money to undertake the opportunity.

2. Time Preference For Consumption

The more people like to consume today, the more firms that
wish to borrow money will have to pay to get those people
not to consume but rather to save.

3. Risk

Investors will require a higher return to invest in projects that


have higher levels of risk

160
4. Expected Inflation

Investors will require enough return to offset the reduced


purchasing power caused by inflation.

5. Federal Reserve Policy

The Federal Reserve can exert influence over interest rates in


several ways.

The goal is to maintain a healthy economy by changing


interest rates.

6. Federal Deficits

The Federal Government frequently spends more than it


takes in.

To compensate for the difference, the Federal Government


must borrow money.

By increasing the demand for borrowed funds, this borrowing


increases interest rates.

161
Components of the Interest Rate
A. The Real Risk Free Rate -- K*
The reward to investors for postponing consumption

B. Inflation Premium -- IP
The additional interest required to compensate investors for
the loss of purchasing power due to increasing prices.

Note: Nominal Risk Free Rate -- Krf


So: Krf = K* + IP
Usually measured as the return on Treasury Bonds

C. Default Risk Premium -- DRP

The additional interest required to compensate investors for


the chance that the borrower will not pay the interest or
principal as required.

D. Liquidity Premium -- LP
The additional interest required to hold investments that can
not be quickly and easily converted to cash.

E. Maturity Risk Premium -- MRP


The additional interest required to hold investments that have
longer terms to maturity.

Based on these components, we can write an equation for the


observed interest rate in the market, K as:

K = K* + IP + DRP + LP + MRP
Or
K = Krf + DRP + LP + MRP

162
The Yield Curve or the Term Structure of Interest
Rates

The yield curve shows the relationship between interest rates and
time to maturity.

We want to see how the interest rate changes with the amount of
time that we borrow money for.

When preparing a yield curve care must be taken to analyze


securities that differ only with respect to time to maturity.

The inflation premium that is used to compute the Yield


Curve is the average annual inflation for the time period of
interest.

If we are interested in a two-year security

2012 inflation 2%
2013 inflation 4%

IP1 + IP 2 + ...IPn
IP =
N

IP 2012 + IP 2013
IP =
2

2+4
IP = =3
2

163
Three Candidate Shapes for the Yield Curve

Normal Yield Curve


5
4
3
2
1
0
1 year 5 year 10 year 20 year 30 year

Interst Rates

Flat Yield Curve


2

1.5

0.5

0
1 year 5 year 10 year 20 year 30 year

Interst Rates

Inverted Yield Curve


6
5
4
3
2
1
0
1 year 5 year 10 year 20 year 30 year

Interst Rates

An inverted yield curve is a predictor of an immanent recession.

164
Theories of the shape of the Yield Curve and implications for firm
behavior

Two ways to make a 6 year loan.


Method 1: Borrow Money for 6 years.

Method 2: Borrow Money for 3 years and pay off the loan.
Then Borrow Money a second time for 3 years.

Either way, we have borrowed money for 6 years. The issue


is if one method is less expensive than the other.

165
Theories

1. Expectations Theory

The Expectations Theory of the Yield Curve says that


expectations about future inflation determines the shape of
the yield curve. Investors and borrowers do not have
preferences for long or short term borrowing.

Implies that it doesn’t matter if you borrow money long term


or short term, either way you will end up paying the same
amount.

2. Liquidity Preference Theory

Says that lenders wish to make short-term loans. Borrows


wish to make long-term loans. In order to entice lenders to
make long term loans, borrowers must pay an interest
premium.

Implies that firms can benefit by borrowing short term rather


than long term.

3. Market Segmentation

There are groups of borrowers and lenders who wish to


borrow and lend short term.

There are groups of borrowers and lenders who wish to


borrow and lend long term.

The long term market and short-term market do not interact.

Implies that it pays to look for bargains for long term or short
term borrowing.

166
Predicting Interest Rates

If you believe in the expectations theory of the term structure


of interest rates, you can state the relationship of interest rates
as follows:

(1 + S 0 − L) L = (1 + S 0 − N ) n1 (1 + X n− L ) n 2

Where S0-L = a long term rate


S0-n = a short term rate
Xn-L = a short term rate that we wish to predict
n1 = the maturity of a short term rate
n2 = the maturity of a second short term rate
L = the maturity of a long term rate

With regard to maturities n1+n2=L

By rearranging the above formula, we can solve for Xn-L as


follows:

(1 + S 0 − L ) L
n1
= (1 + X n− L ) n 2
(1 + S 0 − N )

and

( (1 + S 0 − L) L
(1 + S 0 − N ) n1
) 1
n2
− 1 = X n− L

167
Suppose we wish to predict the 16 year interest rate that will
prevail 14 years from today.

We must observe two figures in the market to make this


prediction. We must observe the current thirty year rate
(suppose it is 6.4 percent). We must also observe the current
fourteen year rate (suppose it is 5.1 percent).

|---------------------------------------------------------------|
0 30 year
Rate = 6.4 percent

|-------------------------------|--------------------------------|
0 14 Years 30 year

Rate = 5.1 percent ??????????????

Rate we wish to predict the sixteen year rate of interest that


will be present in the market 14 years from today.

168
We solve for Xn-L as follows

( (1 + S 0 − L ) L
(1 + S 0 − N ) n1
) 1
n2
− 1 = X n− L

( (1 + 0.064) 30
(1 + 0.051)14
) 1
16
− 1 = X n− L

( 6.4306
2.0065
) 1
16
−1 = X n− L

1
X n−L = (3.205) − 1 = 3.2050.0625 − 1
16

0.0755 = X n− L

Thus, we predict that the 16 year rate of interest, 14 years


from today will be 7.55 percent.

169
Chapter 5 Homework and Solutions

170
Chapter 5 Homework

1. Define each of the following terms:

a. Money market; Capital Market


b. Primary market; Secondary Market
c. Investment Banker
d. Financial Intermediary
e. Mutual Fund
f. Production opportunities; time preference for consumption
g. The real risk free rate of interest; nominal risk-free rate of interest
h. The inflation premium
i. Default risk premium
j. Liquidity; the liquidity premium
k. The term structure of interest rates; yield curve
l. Shapes of the Yield Curve
m. Expectations theory
n. Liquidity preference theory
o. Market segmentation theory

2. Today is January 1, 2014. Suppose that investors expect annual future inflation rates
to be as follows:

Year Annual Inflation


2014 7%
2015 6%
2016 3%
each year after 2016 3%

Assume that the real risk-free rate of interest will remain at a constant 2 percent
per year for the foreseeable future. Also assume that maturity risk premiums on
Treasury Bonds and Bills are 0.10 percent for 1-year securities. For securities
having maturities in more than one year, maturity risk premiums increase 0.1
percent for each year to maturity, up to a limit of 1.0 percent on 10-year or
longer term T-bonds. Calculate the interest rate on 1,2,3,4,5, 10 and 20-year
Treasury Securities, and plot the yield curve.

171
3. You are given the following data. Plot a yield curve based on this data.

Term Rate
6 month 5.4%
1 year 5.6%
2 year 5.8%
3 year 5.9%
4 year 6.0%
5 year 6.1%
10 year 6.3%
20 year 6.5%
30 year 6.6%

4. Suppose that at the beginning of 2014, the expected inflation rate for 2014 was 3
percent, for 2015, 4 percent, for 2016, 2 percent and for 2017 and thereafter, 1
percent. Assume that there is no default risk premium, liquidity premium or
maturity risk premium.

a. What is the average expected inflation rate over the 5-year period 2014-2018?
b. What average nominal risk-free interest rate would, over the 5-year period
from 2014 through 2018, be expected to produce a 3 percent real risk-free rate
of return on 5-year Treasury Securities?
c. Now assume a real rate of 2 percent and a maturity risk premium which starts
at 0.1 percent for one-year securities and increases by 0.1 percent each year.
Furthermore, assume that inflation will occur as indicated earlier in the
problem. Estimate the interest rate in January 2014 on bonds that mature in
1,2,3,4,5, 10 and 20 years, and draw a yield curve based on these data.

5. Suppose that the current rate of interest on 30 year fixed rate mortgages is 6.5 percent.
Suppose that the current rate on 15 year fixed rate mortgages is 6 percent. Using
the techniques that we learned in class, predict what the 15 year mortgage rate
will be 15 years from today.

172
Solutions to Homework for Chapter 5:
Question 1

a. A money market is a financial market for debt securities with a maturity of less
than one year.
Capital markets are markets for long-term debt securities (with a maturity of more
than one year) and corporate stocks.

b. Primary markets are markets in which newly issued securities are traded for the
first time. That is the company is selling securities to investors. Secondary
markets are markets where already existing securities are traded between two
different investors.

c. An investment banker is a middleman between businesses seeking capital for their


companies and savers who wish to invest in those companies.

d. A financial intermediary makes loans to persons businesses and governments


using funds that it obtains by issuing its own securities (that is by accepting
deposits).

e. A mutual fund is a company that pools the funds of many investors together to
make investments.

f. Production opportunities are the returns that you will receive from investing in
productive assets, such as an oven for your restaurant. The time preference for
consumption says that people would rather consume today than at some future
point in time.

g. The real risk-free rate of interest is the reward that you receive for investing today
and consuming later, rather than consuming today. The nominal interest risk-free
rate of interest is the real risk-free rate of interest plus an additional interest
component called an inflation premium which
compensate for inflation.

h. The inflation premium is the additional interest added to the real risk free rate to
compensate the investor for the loss of purchasing power caused by inflation.

i. Default risk is the risk that a borrower will not pay the interest on the loan, or will
not pay the principal on the loan as it comes due.

j. Liquidity refers to the ability to convert an asset to cash quickly and at a fair
price. The liquidity premium is the additional interest rate to compensate
investors for investing in something that can not be quickly converted to cash.

173
k. The term structure of interest rates is the relationship between the return that you
will receive if you invest in a bond (if you lend the US government money for
example), and the length of time that you make the loan for. A yield curve is a
graph that shows the relationship between the interest rate and the amount of time
that the money is committed to an investment for.

l. A normal yield curve is a yield curve that slopes upward. It is called normal
because the yield curve normally takes this shape. An abnormal (or inverted)
yield curve is a yield curve that slopes downward. A third shape of the yield
curve is a flat yield curve. It occurs when the interest rate you receive on the
investment does not depend on the amount of time you make the loan for.

m. The expectations theory says that expectations about future inflation determines
the relationship between short term interest rates and long term interest rates.

n. The liquidity preference theory says that borrowers would rather borrow for the
long term. Lenders would rather lend for the short term. In order to get lenders to
make long term loans, they must be given an interest incentive called a liquidity
premium.

o. The market segmentation theory says that there are groups of people who like to
lend and borrow long term. They do not consider investing in the short-term
market even if there is a great interest rate advantage to doing so. There are
different groups of people who like to lend and borrow short term. They do not
consider investing in the long-term market even if there is a great interest rate
advantage to doing so. Thus there is no particular relationship between interest
rates for long term investments and interest rates for short-term investments.

Question 2

First, we must calculate the average rate of inflation that we should expect to exist over
each of the time periods under consideration.

Period Inflation for this period Average inflation over the past n
years.
1 year 7 =7/1 = 7
2 years 6 = (7 + 6)/2 = 6.5
3 years 3 = (7 + 6 + 3)/3 = 5.33
4 years 3 = (7 + 6 + 3 + 3)/4 = 4.75
5 years 3 = (7 + 6 + 3 +3 + 3)/5 = 4.4
10 years 3 = (7 + 6 + 3 + 3 + 3 + 3 + 3 + 3 + 3 + 3)/10 = 3.7
20 years 3 = (7 + 6 + 3*18)/20 = 3.35

174
Now we can calculate the interest rate. We can use the formula k = k* + IP + DRP + LP
+ MRP

Year k* + IP + DRP + LP + MRP = k


1 2 7 0 0 .1 9.1
2 2 6.5 0 0 .2 8.7
3 2 5.33 0 0 .3 7.63
4 2 4.75 0 0 .4 7.15
5 2 4.4 0 0 .5 6.9
10 2 3.7 0 0 1 6.7
20 2 3.35 0 0 1 6.35

Now you are ready to graph the yield curve. In order to do this you will make a graph
with the years on the bottom axis and k on the upper axis.

Question 2: Yield Curve

10

6
Rate

5 Seri…

0
1 2 3 4 5 10 20
Term

175
Question 3

Question 3: Yield Curve

6.8

6.6

6.4

6.2

6
Rate

Series2
5.8

5.6

5.4

5.2

5
0.5 1 2 3 4 5 10 20 30
Term

Question 4

Part a: to do this part we must simply calculate the average level of inflation for the next
5 years as follows:

We are given the following information


2014 inflation = 3
2015 inflation = 4
2016 inflation = 2
2017 inflation = 1
2018 inflation = 1

So, the average is (3 + 4 + 2 + 1 + 1)/5 = 2.2

Part b: In order to calculate this part, we can use the formula k = k* + IP + DRP + LP +
MRP. Because there is no DRP, LP or MRP, in this case, the formula is also equal to Krf
= K* + IP. We want to solve for k in the formula (or equivalently Krf). For this part we
will assume that there is no default risk premium, liquidity premium or maturity risk
premium.

176
Year k* + IP + DRP + LP + MRP = k

5 3 2.2 0 0 0 = 5.2

Part C:

To do this part we must first calculate the average inflation for each of the time periods
under consideration.

Period Inflation for this period Average inflation over the past n
years.

1 3 3.0
2 4 3.5
3 2 3.0
4 1 2.5
5 1 2.2
10 1 1.6
20 1 1.3

Now we can calculate the interest rate. We can use the formula k = k* + IP + DRP + LP
+ MRP

Year k* + IP + DRP + LP + MRP = k


1 2 3 0 0 .1 5.1
2 2 3.5 0 0 .2 5.7
3 2 3 0 0 .3 5.3
4 2 2.5 0 0 .4 4.9
5 2 2.2 0 0 .5 4.7
10 2 1.6 0 0 1 4.6
20 2 1.3 0 0 2 5.3

177
Question 4: Yield Curve

4
Rate

3 Serie…

0
1 2 3 4 5 10 20
Term

5.
( (1 + S 0 − L) L
(1 + S 0 − N ) n1
) 1
n2
− 1 = X n− L

((1(1++00.065
.06)
)
) 30

15
1
15
− 1 = X n− L

6.6144 1
( )15 − 1 = 𝑋𝑋𝑛𝑛−𝐿𝐿
2.3966

0.0700 = X n − L

178
1

CHAPTER 6

Risk and Return:


The Basics

179
2

Risk and Return: The Basics

In this chapter we will learn about the relationship between risk


and return.

Golden Rule of Finance: In order to earn a higher return you must


be willing to accept a higher level of risk.

We need to assess the return and riskiness of projects.

Failure to do so properly can result in bankruptcy or losses of


substantial amounts of money

Failure to do so properly can result in lower profits than what


we might otherwise achieve.

Goals of this chapter

1. To show you how to measure return


2. To show you how to measure risk
3. To show you the relationship between risk and return

180
3

How to Measure Return

HOW TO MEASURE RETURN

1. Expected Return: The return that we expect to receive


from an investment.

Looking into our crystal ball and estimating what will


happen in the future

If I buy a rental property in Costa Rica, I estimate that I


will get a 15 percent return on my investment.

2. Required Return: The minimum acceptable return from


an investment.

I will only invest in a rental property in Costa Rica if


I can get at least a 10 percent return on my investment.

3. Realized Return: The return we actually end up getting


after the fact.

So I have purchased a rental property in Costa Rica.


One year later I wish to know the return this investment
actually produced for me.

The investment actually produced a 10 percent rate of


return.

181
4

HOW TO USE THE MEASURES OF RETURN

1. To determine if you should make an investment you


should compare the expected return to the required return.

If the expected return is higher than the required return


you should make the investment.

If the expected return is lower than the required return,


you should not make the investment

2. To determine if an investment has performed well or not


you should compare the realized return to the required
return.

If the realized return is higher than the required return,


the project has performed well.

If the realized return is lower than the required return,


the project has performed poorly.

3. To determine how good your capital budgeting skills are


you should compare the expected return to the realized
return.

If the realized return is near the expected return, you


have done a good job of capital budgeting.

If the realized return is a great deal different than the


expected return, you have done a poor job of capital
budgeting.

182
5

CALCULATING RETURNS

CALCULATING RETURNS FOR SINGLE STOCKS

A. Expected Returns

I. Subjective Method:

This method involves looking forward into the future and


making an estimate about what might happen in the future.

Step 1: Estimate the following

a. the possible future states of nature


b. the probability that each state of nature will
occur
c. the return on the asset if each state occurs.

State Prob Return


Pi Ki
Ki-a Ki-b
Boom .2 60% 25%
Normal .6 25% 20%
Recession .2 -35% 15%
1

Pi = the probability that this state of nature will occur


Ki = the return on the investment if this state of nature
occurs.

183
6

Step 2: Compute the Expected Return



K   PiKi

For Stock A

State Prob Return


Pi Ki-a PiKi
Boom .2 60% 12%
Normal .6 25% 15%
Recession .2  -35% -7%
1 K   PiKi = 20%

For Stock B

State Prob Return


Pi Ki-b PiKi
Boom .2 25% 5%
Normal .6 20% 12%
Recession .2 15% 3%
1 K   PiKi = 20%

The two stocks have the same return, however; they


have different risks.

Weaknesses of this method

It is difficult to estimate all possible future states of


nature and the probability of their occurrence.

It is difficult to estimate the return in each state of


nature.

184
7

II. Historical Method

Sometimes it is more convenient to examine historical


returns and assume that they are a fair guess of what we
should expect in the future.

Step 1: Collect historical information about the returns of the


stock in question

Return
Year Kh
2007 30%
2008 20%
2009 13%
2010 -10%
2011 -20%
2012 39%

Kh = the historical return on the investment in each year

185
8

Step 2: Compute the historical average return

  Kh
K  K avg 
N

K avg  The historical average return

Return
Year Kh
2007 30%
2008 20%
2009 13%
2010 -10%
2011 -20%
2012 39%
  Kh 72
K  K avg  =  12.0%
N 6

We assume that the historical average return is a good


estimation of what will happen in the future.

186
9

B. REALIZED RETURNS

The realized return is the return that we actually received


after the fact.

Amt Money End Yr  Amt Money Beg Yr  Cash Re c


Kr 
Amt Money Beg Yr

Suppose we wish to calculate the realized return on an


investment in the Yellow Snow Company. At the beginning
of the year, the stock price was $80. At the end of the year
the stock price was $95. During the year we received a $5
dividend from the company. What is the realized rate of
return.

95  80  5
Kr   .25 or 25%
80

187
10

C. REQUIRED RATE OF RETURN

The minimum acceptable rate of return.

Unfortunately, there is no “one method” for computing the


required rate of return on an investment. Rather, many
different methods have been suggested for doing it.

1. One commonly used method is called the CAPM


(SML)

CAPM = Capital Asset Pricing Model


SML = Security Market Line

The CAPM says that the required return on any


stock can be computed based on the risk free rate
of interest and a premium that it based on the
expected return on the market and the riskiness of
the stock. The CAPM is a theory that gives us the
Security Market Line (SML).

The SML is given by

Kreq  Krf  i ( Km  Krf )

Kreq = the required rate of return


Krf = the risk free rate of interest
Km = the expected return on the market
Bi = the Beta for stock i. Beta is a measure of
Risk that we will learn more about later.

Beta can be obtained from the Value Line


Investment Service or various online
services.

188
11

Suppose that you expect that the return on


the stock market will be 14 percent next
year. The risk free rate of interest is 5
percent. You look in Value Line and find
that the Beta for Microsoft is 1.2. What is
the required rate of return on Microsoft
stock?

Kreq  Krf  i ( Km  Krf )

Kreq  5  1.2(14  5)

= 15.8%

Alternate Methods for estimating the required rate of return:

A. Capital Market Line


Says we should rely the standard deviation as a
measure of risk rather than Beta.
B. Arbitrage Pricing Theory
Says that we should use other measures of risk in
addition to beta, to determine the appropriate
required rate of return.

Suggested Measures:
a. Price to Book Ratio
b. Industry
c. Size of the Firm

189
12

CALCULATING RETURNS FOR A PORTFOLIO

A. Expected Returns

i. To calculate the expected return on a portfolio using


subjective data

The expected return on a portfolio is equal to a weighted


average of the expected returns on the stocks in the portfolio.
   
Kp  W 1K 1  W 2 K 2  ...WnKn

Kp  The expected return on a portfolio
W1 = The proportion of our funds invested in stock 1
W2
 = The proportion of our funds invested in stock 2
K1 = The expected return on stock 1
K 2 = The expected return on stock 2

Step 1: Determine how much we will invest in each stock.

Suppose we are considering investing 20 percent of our


money in stock 1, 15 percent of our money in stock 2,
15 percent of our money in stock 3 and 50 percent of
our money in stock 4.

Step 2: Compute the expected return on each individual


Stock as we learned in the previous section. (p. 6-8)

Ki Wi
Stock 1 40% 0.20
Stock 2 35% 0.15
Stock 3 15% 0.15
Stock 4 20% 0.50

190
13

Step 3: Compute the expected return



Kp  ( 40 X .20)  (35 X .15)  (15 X .15)  ( 20 X .50)

= 8 + 5.25 + 2.25 + 10 = 25.5%

ii. To Calculate the Expected Return on a Portfolio Using


Historical Data

Step 1: Determine the historical realized rate of return on


each stock for each historical year.

Kh
Kh1 Kh2
2007 30% 29%
2008 20% 31%
2009 13% 30.58%
2010 -10% -17%
2011 -20% -16%
2012 39% 14.42%

Kh = the historical returns on the stock.

Step 2: Determine how much of our funds we will invest in


each stock.

Suppose we will invest ½ of our money in each stock.

191
14

Step 3: Compute the return on the portfolio in each historical


year.

Khp  W 1Kh1  W 2 Kh2  ...WnKhn

Khp = the historical return on the portfolio


Kh1 = the historical return on stock 1
Kh2 = the historical return on stock 2

Kh
Kh1 Kh2 Khp
2007 30% 29% .5(30) + .5(29) = 29.5
2008 20% 31% .5(20) + .5(31) = 25.5
2009 13% 30.58% .5(13) + .5(30.58) = 21.79
2010 -10% -17% .5(-10) + .5(-17) = -13.5
2011 -20% -16% .5(-20) +.5(-16) = -18
2012 39% 14.42 .5(39) + .5(14.42) = 26.71

Step 4: Compute the average historical return on the


portfolio. We assume this is our expected future
return.

  Khp
Kp  K avgp 
N

K avgp = the historical average return on a portfolio


N = the number of observations

 29.5  25.5  21.79  (13.5)  (18)  26.71


Kp  K avgp   12%
6

192
15

Realized Rate of Return on a Portfolio

Amt Money End Yr  Amt Money Beg Yr  Cash Re c


Krp 
Amt Money Beg Yr

This is the same procedure as we followed for a single stock.


The only difference is we must compute the amount of
money we have at each time point.

Suppose we buy 100 shares of IBM stock for $50 per share
and 200 shares of Microsoft stock for $70 per share. During
the year IBM pays a $1.00 per share dividend and Microsoft
pays a $1.50 per share dividend. At the end of the year the
price of IBM stock is $51 per share and the price of
Microsoft stock is $73 per share. What is the realized rate of
return on this portfolio.

Step 1: Compute Amount of money at the beginning of the


Year.

IBM 100 Sh X $50 = $5,000


Microsoft 200 Sh X $70 = $14,000
Total $19,000

Step 2: Compute the total amount of the cash received

IBM 100 Sh X $1 = $100


Microsoft 200 Sh X $1.5 = $300
Total $400

193
16

Step 3: Compute the amount of money at the end of the year

IBM 100 Sh X $51 = $5,100


Microsoft 200 Sh X $73 = $14,600
Total $19,700

Step 4: Using the above information compute the realized


return.

$19,700  $19,000  $400


Krp 
$19,000

= 0.0579 or 5.79%

194
17

Required Rate of Return on a Portfolio

Kreqp  W 1Kreq1  W 2 Kreq 2  ...WnKreqn

Kreq1 = the required rate of return on stock 1


Kreq2 = the required rate of return on stock 2

Step 1: Compute the required rate of return on each individual


stock using the methods we previously learned.

Suppose that the expected return on the market is 14


percent and the risk free rate of interest is 5 percent.
The beta for stock 1 is 1.2 and the beta for stock 2 is 1.4

From P. 10-11: Kreq = Krf + B(Km-Krf)

Kreq1  5  1.2(14  5) = 15.8%

Kreq2  5  1.4(14  5) = 17.6%

Step 2: Determine how much of your money you will invest in


each stock.

Suppose that we will invest 70 percent of our funds in


stock 1 and 30 percent of our funds in stock 2.

Step 3: Compute the required rate of return on the portfolio:

Kreqp  W 1Kreq1  W 2 Kreq 2  ...WnKreqn

Kreqp  .7(15.8)  .3(17.6) = 16.34%

Also Acceptable
Kreqp  .7(0.158)  .3(0.176)  0.1634

195
18

Measures of Risk

Stand Alone Risk

1. Standard Deviation
2. Variance
3. Coefficient of Variation

Portfolio

1. Standard Deviation
2. Variance
3. Coefficient of Variation
4. Beta

For each measure of risk, larger numbers indicate more


risk.

196
19

Calculating Risk Measures

STAND ALONE RISK

1. Standard Deviation and Variance

Method 1: Using Subjective Data (Population Standard


Deviation)

   ( ki  k ) 2 pi = Standard Deviation

 2   ( ki  k ) 2 pi = Variance

Note:   ( )
2 2


Three inputs are required to solve the formula ki, k and pi

Step 1: Obtain the probabilities, return in each state, and the


expected return. Recall that we calculated the expected
return earlier for stock A (see page 5-6).

State Prob Return


Pi Ki-a PiKi
Boom .2 60% 12%
Normal .6 25% 15%
Recession .2  -35% -7%
1 K   PiKi = 20%

Step 2: Compute the standard deviation as follows

197
20

Computation of Standard Deviation for Stock A

State Prob Return


  
Pi Ki-a PiKi (ki  k ) ( ki  k ) 2 ( ki  k ) 2 pi
Boom .2 60% 12% 60-20 = 40 40 X 40 =1,600 1600 X .2 = 320
Normal .6 25% 15% 25-20 = 5 5 X 5 = 25 25 X .6 = 15
Recession .2 -35% -7% -35-20 = -55 -55 X -55 =3,025 3025 X .2 = 605
 2  2
1 K = 20% Variance =    ( ki  k ) pi = 940
 2
Standard Deviation =    ( ki  k ) pi = 940 =30.66%

198
21

Method 2: Using Historical Data (Sample Standard Deviation)

 (kh  K avg ) 2
S = standard deviation
N 1
2
2  (kh  K avg )
S  = variance
N 1

Recall that

Kh = the historical return on the investment in each year


K avg  The historical average return

Three pieces of information are needed to compute the


standard deviation and variance, kh, K avg and N .

Suppose we have gathered the following historical


information about a stock:
Return
Year Kh
2007 30%
2008 20%
2009 13%
2010 -10%
2011 -20%
2012 39%

199
22

Step 1: Compute the historical average return as we


learned earlier (see page 7-8)

Return
Year Kh
2007 30%
2008 20%
2009 13%
2010 -10%
2011 -20%
2012 39%
  Kh 72
K  K avg  =  12.0%
N 6

200
23

Compute the Variance and Standard Deviation

(kh  K avg ) 2 2
S
 = standard deviation, S 2 
 (kh  K avg )
= variance
N 1 N 1

Year Kh Kh  K avg ( Kh  K avg ) 2


2007 30 30 - 12 = 18 18 X 18 = 324
2008 20 20 - 12 = 8 8 X 8 = 64
2009 13 13 - 12 = 1 1 X 1 = 1
2010 -10 -10 - 12 = -22 -22 X –22 = 484
2011 -20 -20 – 12 = -32 -32 X -32 = 1,024
2012 39 39 - 12 = 27 27 X 27 = 729
2
K avg  12%  (kh  K avg ) = 2,626

2 2,626
2  (kh  K avg )
Variance = S    525.2
N 1 6 1

 (kh  K avg ) 2
Standard Deviation = S   525.2  22.92
N 1

201
24

Use of Standard Deviation:

The standard deviation has multiple uses including to


compute Confidence Intervals

Confidence Intervals tell us how confident we can be that


realized return on an investment will fall within some range.

For a single stock



CI 68  K  1 , orK avg  1S

CI 95  K 2 , orK avg  2 S

CI 99.75  K 3 , orK avg  3S

For a portfolio

CIp 68  K avgp  1Sp


CIp 95  K avgp  2 Sp
CIp 99.75  K avgp  3Sp

202
25

Suppose that we are examining a stock that has a 12 percent


expected return and a 22.92 percent standard deviation

We can compute and interpret confidence intervals as


follows:

CI 68  K  1 , orK avg  1S

= 12+22.92 = 34.92
= 12-22.92 = -10.92

We can be 68 percent confident that our realized return


will be between –10.92 percent and 34.92 percent.

CI 95  K 2 , orK avg  2 S

= 12+2(22.92) = 57.84
= 12-2(22.92) = -33.84

We can be 95 percent confident that our realized return


will be between –33.84 percent and 57.84 percent.

CI 99.75  K 3 , orK avg  3S

= 12+3(22.92) = 80.76
= 12-3(22.92) = -56.76

We can be 99.75 percent confident that our realized


return will be between –56.76 percent and 80.76
percent.

203
26

WEAKNESS OF THE STANDARD DEVIATION

The standard deviation is an absolute measure of risk. As such it


can be deceptive when applied to two securities that have very
different expected returns.

Suppose that we are examining the following two


investments.

Stock K 
A 2% 10
B 100% 10

According to the standard deviation, both of these stocks


have the same amount of risk.

However, it would seem logical that the risk of Stock A is


higher than the risk of Stock B

204
27

Coefficient of Variation a relative measure of risk that tells us the


amount of risk per percent of expected return.

For a Single Stock

 S
CV   or
K K avg

For a Portfolio
Sp
CVp 
K avgp

For the two stocks we are examining:



Stock K  CV
10
A 2% 10 5
2

10
B 100% 10  0 .1
100

This coefficient of variation indicates that stock A is


riskier than stock B.

205
28

The Standard Deviation and Variance on a Portfolio

Recall that the expected return on a portfolio equals a


weighted average of the expected returns on the stock in the
portfolio. The standard deviation on a portfolio does not
equal a weighted average of the standard deviations on the
stocks in the portfolio. Rather, the standard deviation on a
portfolio will be less than or equal to a weighted average of
the standard deviations on the stock in the portfolio.

We will examine the standard deviation on a portfolio using


historical data. The standard deviation on a portfolio using
subjective data can be computed, however; we will not cover
the topic in this course.

 (khp  K avgp) 2
Sp  = Standard Deviation
N 1

 (khp  K avgp) 2
S 2p  = Variance
N 1

We need three inputs to solve the formula:


khp, K avgp and N .

In addition, in order to compute Khp, we must know the


proportion of our money we will invest in each stock.

Recall from our discussion of expected returns that we know


how to compute khp and K avgp . (See Page 14)

206
29

Suppose we have the following historical information about Stock


1 and Stock 2 and wish to compute the standard deviation on a
portfolio. We will invest ½ of our money in each Stock 1 and
Stock 2. The historical returns on Stock 1 and Stock 2 are as
follows.

Year Kh1 Kh2


2007 30 29
2008 20 31
2009 13 30.58
2010 -10 -17
2011 -20 -16
2012 39 14.42

207
30

Computation of the Standard Deviation on a Portfolio

Year Kh1 Kh2 Khp ( Khp  K avgp) ( Khp  K avgp) 2


2007 30 29 29.5 17.5 306.25
2008 20 31 25.5 13.5 182.25
2009 13 30.58 21.79 9.79 95.84
2010 -10 -17 -13.5 -25.5 650.25
2011 -20 -16 -18 -30 900
2012 39 14.42 26.71 14.71 216.38
K avg  12 12 K avgp  12%  (khp  K avgp ) 2 = 2350.97
S 22.92 22.92

 (khp  K avgp) 2 2350.97


Variance = S 2p    470.19
N 1 6 1

 (khp  K avgp) 2
Standard Deviation = Sp   470.19  21.69
N 1
Note: See page 8 for computation details of K avg See page 23 for computations details of S. See
page 14 for computation details of K avgp Note that the expected return on this portfolio is equal to
the expected returns on the two stocks in the portfolio. The standard deviation on the portfolio,
however; is lower than the standard deviation on either stock in the portfolio.

208
31

Portfolio 1: 50% Stock 1 and 50% Stock 2

Stock 1 Stock 2 Portfolio

2007 30% 29% 29.5%


2008 20 31 25.5
2009 13 30.58 21.79
2010 -10 -17 -13.5
2011 -20 -16 -18
2012 39 14.42 26.71
Avg. Ret 12% 12% 12%
Std Dev 22.92% 22.92% 21.69%

Portfolio 2: 50% Stock 1 and 50% Stock 3

Stock 1 Stock 3 Portfolio

2007 30% 30% 30%


2008 20 20 20
2009 13 13 13
2010 -10 -10 -10
2011 -20 -20 -20
2012 39 39 39
Avg. Ret 12% 12% 12%
Std Dev 22.92% 22.92% 22.92%

Portfolio 3: 50% Stock 1 and 50% Stock 4

Stock 1 Stock 4 Portfolio

2007 30% -6% 12%


2008 20 4 12
2009 13 11 12
2010 -10 34 12
2011 -20 44 12
2012 39 -15 12
Avg. Ret 12% 12% 12%
Std Dev 22.92% 22.92% 0%

209
32

So we can see that the expected return on a portfolio is equal to a


weighted average of the expected returns on the stocks in the
portfolio. However, the standard deviation on a portfolio will be
less than or equal to a weighted average of the standard deviations
on the on the stocks in the portfolio.

The amount of risk that we can get rid of by combining stocks


together into a portfolio depends upon how much the returns on the
two stocks tend to move together or opposite. The more the
returns move opposite each other, the more risk we can get rid of
by combining the stocks into a portfolio.

Two measures of how much risk we can get rid of by combining


stocks together into a portfolio. Measures of how much the returns
on two stocks move together or opposite each other.

1. Covariance

^ ^

ij  
[( Ki  Ki ) X ( Kj  Kj )]
N 1

The difficulty associated with the Covariance is that it


is not easily interpretable.

You are not responsible for being able to compute this


formula.

210
33

2. Correlation Coefficient:

ij
ij 
iXj

ij  CorrelationCoefficient between stocks i and j


ij  Covariance between stocks i and j
i = Standard Deviation on Stock i
i = Standard Deviation on Stock j

The correlation coefficient will have a value ranging


from + 1 to –1

A correlation coefficient of +1 is called perfect positive


correlation. If two stocks are perfectly positively
correlated the returns on the stocks move exactly
together. In this case, we can not get rid of any risk by
combining the two stocks into a portfolio.

A correlation coefficient of –1 is called perfect negative


correlation. If two stocks are perfectly negatively
correlated the returns on the two stocks move exactly
opposite each other. In this case we can get rid of all
risk by combining the two stocks together into a
portfolio.

A correlation coefficient between +1 and –1 indicates


that the returns on two stocks move somewhat together,
or somewhat opposite each other, but not exactly. In
this case we can get rid of some risk, but not all risk by
combining the two stocks together into a portfolio.

211
34

How Does Portfolio Size Affect Risk

If we can reduce risk by combining 2 stocks together into a


portfolio, can we get rid of addition risk by adding more
stocks?

Two Parts of Total Risk

Unsystematic Risk (Firm Specific, Diversifiable).

This is the risk associated with a specific stock.


The risk that Bill Gates Dies for Microsoft Stock

If we purchase enough different stocks, we can get rid


of all unsystematic risk.

Systematic Risk (Market Risk, Non-Diversifiable)

This is the risk associated with the stock market as a


whole. The risk that there will be a major terrorist
attack.

No matter how many stocks we purchase, we can not


get rid of systematic risk.

How many stocks do you need to hold to get rid of most


unsystematic risk?

The evidence on this issue is mixed, ranging from 10 stocks


to 30 stocks in a portfolio to get rid of most unsystematic
risk.

212
35

Beta

Since we can get rid of most or all unsystematic risk by combining


stocks together into a portfolio, the only thing that is left is
systematic risk.

It would be desirable to have a measure of systematic risk – that


risk that we cannot get rid of. Such a measure exists and is called
Beta.

Beta tells us how the price of a security changes in relation to


market changes.

The stock market as a whole has a Beta of 1.

213
36

Interpretation of Beta for individual firms

Beta of 1:

When the stock market increases by 1%, the price of


this stock will increase by 1%.

When the stock market decreases by 1%, the price of


this stock will decrease by 1%

Beta > 1:

When the stock market increases by 1%, the price of


this stock will increase by more than 1%.

When the stock market decreases by 1%, the price of


this stock will decrease by more than 1%

Beta < 1:

When the stock market increases by 1%, the price of


this stock will increase by less than 1%.

When the stock market decreases by 1%, the price of


this stock will decrease by less than 1%

How to use Beta:

Recall that Beta was an input into the Capital Asset Pricing
Model. We used the Capital Asset Pricing model to compute
the required rate of return on a stock.

214
37

Chapter 6 Homework and Solution

215
38

Chapter 6 Homework

1. Given the following subjective information, compute the expected return, the standard
deviation and the coefficient of variation.

State Probability of this state Return on Miller Co. Stock


Boom 0.3 25%
Normal 0.6 15%
Recession 0.1 11%

2. Given the following historical information and assuming that the past is a good
estimate of what will happen in the future, compute the expected return, the standard
deviation and the coefficient of variation.

Year Return on Heiniken Stock


2008 -18
2009 33
2010 15
2011 -.5
2012 33

3. You bought a stock for 35 dollars at the beginning of the year. During the year the
stock paid a dividend of $2.00. You sold the stock at the end of the year for $36.
What is your realized rate of return on this stock?

4. You will invest in a portfolio. You will invest 60 percent of your funds in stock 1 and
40 percent of your funds in stock 2. You have the following historical information
about the returns on the two stocks. What is the expected return on the portfolio
assuming that what has happened in the past is a good estimate of what will happen in
the future. Calculate the standard deviation and coefficient of variation of the
portfolio.

Year Return Stock 1 Return Stock 2

2008 10 % 15 %
2009 12 % 11 %
2010 22 % 40 %
2011 10 % 20 %
2012 15 % 15 %

5. You have checked in Value Line and discovered the Beta for Wall Mart stock is 1.1.
You expect the return on the market next year will be 15 percent. You have checked
with the bank and they will give you 6 percent interest if you put your money in a risk
free account with them. Using the SML, what is the required rate of return on this
stock?

216
39

6. You are interested in making an investment that has a 20 percent expected return and a
5 percent standard deviation. Compute a 68, 95, and 99.75 percent confidence
interval for the realized return on this stock.

217
40

Chapter 6 Homework Solutions:

1.

State Pi Ki PiKi − ( − ) ( − )
Boom .3 25 % 7.5 7.4 54.76 16.428
Normal .6 15 % 9 -2.6 6.76 4.056
Recession .1 11 % 1.1 -6.6 43.56 4.356
∑ = = 17.6 ∑( − ) = 24.84

= ( − )

= √24.84 = 4.98

4.98
= = 0.2829
17.6

2. Kavg = (-18+33+15-.5+33)/5 = 12.5 this is the expected return

Year Kh Kh-Kavg (Kh-Kavg)2


2008 -18 -30.5 930.25
2009 33 20.5 420.25
2010 15 2.5 6.25
2011 -.5 -13 169
2012 33 20.5 420.25

∑( ℎ − ) = 1946

∑( ℎ − )
= = √486.5 = 22.06
−1

22.06
= = = 1.765
12.5

3. Kr = (36-35 +2)/35 = .0857 or 8.57 %

218
41

4.
Year Kh1 Kh2 Khp

2008 10 % 15 % .6(10) + .4(15) = 12


2009 12 % 11 % .6(12) + .4(11) = 11.6
2010 22 % 40 % .6(22) + .4(40) = 29.2
2011 10 % 20 % .6(10) + .4(20) = 14
2012 15 % 15 % .6(15) + .4(15) = 15

Kavgp = (12+11.6+29.2+14+15)/5 = 16.36

Standard Deviation.

Year Khp Khp – Kavgp (Khp – Kavgp)2

2008 12 -4.36 19
2009 11.6 -4.76 22.66
2010 29.2 12.84 164.87
2011 14 -2.36 5.57
2012 15 -1.36 1.85

∑( ℎ − ) = 213.95

∑( ℎ − )
= 53.49
−1

∑( )
= = 7.31

7.31
= = = 0.4468
16.36

Coefficient of Variation = S/ Kavgp = 7.31/16.36 = .4468

5. Kreq = Krf + bi(Km – Krf)


= 6 + 1.1(15-6)
= 15.9

6.

219
42


CI 68  K  1

= 20+5 = 25
= 20-5 = 15

We can be 68 percent confident that our realized return will be between


25% and 15%.

CI 95  K 2

= 20+2(5) = 30
= 20-2(5) = 10

We can be 95 percent confident that our realized return will be between


30% and 10%.

CI 99.75  K 3

= 20+3(5) = 35
= 20-3(5) = 5

We can be 99.75 percent confident that our realized return will be between
35% and 5%.

220
Chapter 7
Business and Financial Risk

221
Chapter 7: Business and Financial Risk

In this section we will discuss four concepts

A. Break Even Analysis

B. Operating Leverage

C. Financial Leverage

D. Combined Leverage

Leverage involves using fixed costs to magnify the potential return to a firm.

Consider a company that sells “Meatloaf” impersonator MP3 songs for $2.00 each.
The company is faced with the decision to rent or purchase the servers that will
deliver the music to its customers. Renting the delivery will cost $1.50 per song
sold in variable costs. In addition, if we rent the equipment our fixed operating
costs will be $25,000 per year. If we purchase the equipment, our variable costs
per song will decline to $1.00 per song. However, our fixed operating costs will
increase to $175,000 per year. The company is in a 30 percent tax bracket. The
company currently borrows $300,000 at a 10 percent interest rate. We wish to
compare these two alternatives:

Note: fixed costs remain constant regardless of how many units you sell. Variable
costs depend upon the number of units that you sell. If I open a store to sell t-shirts
in the mall, variable costs would include what we paid to acquire the t-shirts from
our supplier. Fixed costs would include the price of the Yellow Pages Add that I
take out in the newspaper.

222
To begin the analysis, compute the income statements under each scenario. To
begin with, assume that we will sell 500,000 songs per year.

Rent Buy
Sales $1,000,000 $1,000,000
Variable Costs $750,000 $500,000
Fixed Costs $25,000 $175,000
EBIT $225,000 $325,000
I $30,000 $30,000
EBT $195,000 $295,000
Tax $58,500 $88,500
NI $136,500 $206,500

Clearly, in this analysis, the company would be better off by purchasing the
equipment.

However, what happens if you have overestimated the number of people who want
to download the song “Paradise by the Dashboard Light” Instead of selling
500,000 songs, you only sell 250,000 songs. How does this fact change the above
analysis?

Sales $500,000 $500,000


Variable Costs $375,000 $250,000
Fixed Costs $25,000 $175,000
EBIT $100,000 $75,000
I $30,000 $30,000
EBT $70,000 $45,000
Tax $21,000 $13,500
NI $49,000 $31,500

In this case, the company would be better off by renting the equipment.

223
Break Even Analysis:

Operating Break Even Analysis tells us the number of units we must sell in order
to have an Earnings Before Interest and Taxes of $0. Alternatively, it is the
number of units we must sell to cover our fixed and variable operating costs.

Notation:

Q = Quantity Sold
P = Price per Unit
V = Variable Cost per Unit
F = Total Fixed Costs
QBE = Break Even Quantity

𝑭𝑭
𝑸𝑸𝑸𝑸𝑸𝑸 =
𝑷𝑷 − 𝑽𝑽

224
If we rent the equipment:
25,000
𝑄𝑄𝑄𝑄𝑄𝑄 = = 50,000 𝑈𝑈𝑈𝑈𝑈𝑈𝑈𝑈𝑈𝑈
2.00 − 1.50
That is: If we rent the equipment, we will have a zero EBIT if our sales are
50,000 units. At sales levels above 50,000 units, we will earn a positive
EBIT. At sales levels below 50,000 units, we will earn a negative EBIT.

If we purchase the equipment:


175,000
𝑄𝑄𝑄𝑄𝑄𝑄 = = 175,000 𝑈𝑈𝑈𝑈𝑈𝑈𝑈𝑈𝑈𝑈
2.00 − 1.00
That is: If we purchase the equipment, we will have a zero EBIT if our sales
are 175,000 units. At sales levels above 175,000 units, we will earn a
positive EBIT. At sales levels below 175,000 units, we will earn a negative
EBIT.

The final result is that if we purchase the machine we make a larger profit if our
sales are good. However, if we rent the machine, we make a larger profit if our
sales are poor.

225
Business Risk and Operating Leverage

Business risk is the variability in EBIT that occurs within a company.

The Degree of Operating Leverage (DOL) measures business risk by telling us


how much EBIT changes when sales change by a small amount.

How to measure operating leverage at sales level Q:

𝑄𝑄(𝑃𝑃 − 𝑉𝑉)
𝐷𝐷𝐷𝐷𝐷𝐷 =
𝑄𝑄(𝑃𝑃 − 𝑉𝑉) − 𝐹𝐹

Consider the example above with sales of 500,000 songs.

Renting Equipment

𝑄𝑄(𝑃𝑃 − 𝑉𝑉) 500,000(2 − 1.5)


𝐷𝐷𝐷𝐷𝐷𝐷 = = = 1.11
𝑄𝑄(𝑃𝑃 − 𝑉𝑉) − 𝐹𝐹 500,000(2 − 1.5) − 25,000

We can interpret this number as follows: If sales go up by 1 percent, our EBIT


will go up by 1.11 percent.

Purchasing Equipment

𝑄𝑄(𝑃𝑃 − 𝑉𝑉) 500,000(2 − 1)


𝐷𝐷𝐷𝐷𝐷𝐷 = = = 1.53846
𝑄𝑄(𝑃𝑃 − 𝑉𝑉) − 𝐹𝐹 500,000(2 − 1) − 175,000

We can interpret this number as follows: If sales go up by 1 percent, our EBIT


will go up by 1.53846 percent. If sales go down by 1 percent, EBIT will go down
by 1.53846 percent.

226
Financial Risk and Degree of Financial Leverage

Financial risk is the risk associated with the way that the firm is financed. The
Degree of Financial Leverage (DFL) tells us how much our Earnings Before Taxes
(EBT) changes when EBIT change by a small amount.

Consider the example above with 500,000 units of sales and purchasing the
equipment. In one instance we finance the company with $300,000 of debt that
costs 10 percent interest. Alternatively, we are considering financing the firm with
$100,000 of debt at 10 percent interest and obtaining the remainder of the needed
funds from stock issues.

$300,000 $100,000
Debt Debt
Sales $1,000,000 $1,000,000
Variable Costs $500,000 $500,000
Fixed Costs $175,000 $175,000
EBIT $325,000 $325,000
I $30,000 $10,000
EBT $295,000 $315,000
Tax $88,500 $94,500
NI $206,500 $220,500

The Degree of Financial Leverage is measured as follows

𝐸𝐸𝐸𝐸𝐸𝐸𝐸𝐸
𝐷𝐷𝐷𝐷𝐷𝐷 =
𝐸𝐸𝐸𝐸𝐸𝐸𝐸𝐸 − 𝐼𝐼

227
With $300,000 of debt

325,000
𝐷𝐷𝐷𝐷𝐷𝐷 =
325,000 − 30,000
DFL =1.1017

This DFL can be interpreted as follows: If EBIT goes up by 1 percent, EBT will
go up by 1.1017 percent. If EBIT goes down by 1 percent, EBT will go down by
1.1017 percent.

With $100,000 of debt


325,000
𝐷𝐷𝐷𝐷𝐷𝐷 =
325000 − 10,000

DFL = 1.0317

Notes:

Higher DFL indicates more risk.

228
Combined Leverage

Combined Leverage tells us how much our Earnings Before Taxes (EBT) changes
when sales change by a small amount. It measures the combined effects of all
fixed costs.

Recall the following scenario

$300,000
Debt
Sales $1,000,000
Variable Costs $500,000
Fixed Costs $175,000
EBIT $325,000
I $30,000
EBT $295,000
Tax $88,500
NI $206,500

Recall further that we have previously calculated the DOL for this scenario to be:
Recall also that we have previously calculated the DFL for this
scenario to be: 1.1017.

The DCL can be computed as follows”

𝐷𝐷𝐷𝐷𝐷𝐷 = 𝐷𝐷𝐷𝐷𝐷𝐷 ∗ 𝐷𝐷𝐷𝐷𝐷𝐷

DCL = 1.53846*1.1017 = 1.6949

This figure can be interpreted as: If sales increase by 1 percent, EBT will increase
by 1.6949 percent. If sales decrease by 1 percent, EBT will decrease by 1.6949
percent.

229
Chapter 7:
Business and Financial Risk
Homework Assignment and Solution

230
Chapter 7: Business and Financial Risk Homework

Questions:

1. Consider a baseball bat manufacturing firm. The firm has total fixed costs of $10,000 per
month. The firm sells the bats it manufactures for $50 each. Variable costs to manufacture each
bat are $25 each. What is the breakeven quantity?
2. You are assigned the job of analyzing the breakeven quantity for General Motors
Corporation. General Motors has fixed costs of $40,000,000,000 per year. The average sales
price for its cars are $20,000. The average variable costs per car are $15,000. How many cars
must General Motors sell to break even?
3: You have calculated the breakeven quantity of sales for RipYouOff Used Cars Dealership to
be 110. The company has total fixed costs of $800,000. The company sells its cars for $25,000
each. Calculate the variable cost per unit for RipYouOff?
4. The “Damn Big Pizza Company”, has estimated that it will sell 200,000 pizza’s this
year at an average price of $11.95 each. The company has fixed costs of $500,000. In addition,
variable costs of producing each Pizza are $5.95.

a. Compute the operating break even point and the degree of operating leverage for
TDBPC.
b. What will happen to the firm’s Degree of Operating Leverage if a machine is purchased
that increases fixed costs by $150,000 per year, but reduce our cost of producing each
pizza by $0.95?

5. The “Delivery in a Bikini Pizza Company” (there once was a company that provided this
service) has estimated the following income statements before and after it makes certain changes
to the company. The company sells 1,000,000 meals per year currently and expects to sell
1,500,000 meals per year after the change.

Before After
Change Change
Sales $10,000,000 $15,000,000
Variable Costs $9,000,000 $13,000,000
Fixed Costs $750,000 $1,250,000
EBIT $250,000 $750,000
I $150,000 $500,000
EBT $100,000 $250,000
Tax $20,000 $62,500
NI $80,000 $187,500

Compute the Degree of Operating Leverage, the Degree of Financial Leverage and the Degree of
Combined Leverage before and after the change.

231
Business and Financial Risk Homework Solutions
$10,000
1. 𝑄𝑄𝑄𝑄𝑄𝑄 = = 400 𝑏𝑏𝑏𝑏𝑏𝑏𝑏𝑏
$50−$25

𝐹𝐹 40,000,000,000
2. 𝑄𝑄𝑄𝑄𝑄𝑄 = = = 8,000,000 𝑐𝑐𝑐𝑐𝑐𝑐𝑐𝑐 𝑝𝑝𝑝𝑝𝑝𝑝 𝑦𝑦𝑦𝑦𝑦𝑦𝑦𝑦
𝑃𝑃−𝑉𝑉 20,000−15,000

𝐹𝐹
3. 𝑄𝑄𝑄𝑄𝑄𝑄 =
𝑃𝑃−𝑉𝑉

$800,000
110 =
$25,000 − 𝑉𝑉𝑉𝑉𝑉𝑉𝑉𝑉𝑉𝑉𝑉𝑉𝑉𝑉𝑉𝑉 𝐶𝐶𝐶𝐶𝐶𝐶𝐶𝐶𝐶𝐶 𝑈𝑈𝑈𝑈𝑈𝑈𝑈𝑈

110 ∗ $25,000 − 110 ∗ 𝑉𝑉𝑉𝑉𝑉𝑉𝑉𝑉𝑉𝑉𝑉𝑉𝑉𝑉𝑉𝑉 𝐶𝐶𝐶𝐶𝐶𝐶𝐶𝐶 𝑈𝑈𝑈𝑈𝑈𝑈𝑈𝑈 = $800,000

$2,750,000 − 110 ∗ 𝑉𝑉𝑉𝑉𝑉𝑉𝑉𝑉𝑉𝑉𝑉𝑉𝑉𝑉𝑉𝑉 𝐶𝐶𝐶𝐶𝐶𝐶𝐶𝐶 𝑈𝑈𝑈𝑈𝑈𝑈𝑈𝑈 = $800,000

−110 ∗ 𝑉𝑉𝑉𝑉𝑉𝑉𝑉𝑉𝑉𝑉𝑉𝑉𝑉𝑉𝑉𝑉 𝐶𝐶𝐶𝐶𝐶𝐶𝐶𝐶 𝑈𝑈𝑈𝑈𝑈𝑈𝑈𝑈 = −$1,950,000

𝑉𝑉𝑉𝑉𝑉𝑉𝑉𝑉𝑉𝑉𝑉𝑉𝑉𝑉𝑉𝑉 𝐶𝐶𝐶𝐶𝐶𝐶𝐶𝐶 𝑈𝑈𝑈𝑈𝑈𝑈𝑈𝑈 = $17,727.27


𝐹𝐹
4. a. 𝑄𝑄𝑄𝑄𝑄𝑄 = 𝑃𝑃−𝑉𝑉

500,000
𝑄𝑄𝑄𝑄𝑄𝑄 = = 83,333
11.95 − 5.95

𝑄𝑄(𝑃𝑃 − 𝑉𝑉) 200,000(11.95 − 5.95)


𝐷𝐷𝐷𝐷𝐷𝐷 = = = 1.714
𝑄𝑄(𝑃𝑃 − 𝑉𝑉) − 𝐹𝐹 200,000(11.95 − 5.95) − 500,000

b.
𝑄𝑄(𝑃𝑃 − 𝑉𝑉) 200,000(11.95 − 5.00)
𝐷𝐷𝐷𝐷𝐷𝐷 = = = 1.878
𝑄𝑄(𝑃𝑃 − 𝑉𝑉) − 𝐹𝐹 200,000(11.95 − 5.00) − 650,000

5. Before Change
𝑄𝑄(𝑃𝑃 − 𝑉𝑉) 1,000,000(10.00 − 9.00)
𝐷𝐷𝐷𝐷𝐷𝐷 = = =4
𝑄𝑄(𝑃𝑃 − 𝑉𝑉) − 𝐹𝐹 1,000,000(10.00 − 9.00) − 750,000

𝐸𝐸𝐸𝐸𝐸𝐸𝐸𝐸 250,000
𝐷𝐷𝐷𝐷𝐷𝐷 = = = 2.5
𝐸𝐸𝐸𝐸𝐸𝐸𝐸𝐸 − 𝐼𝐼 250,000 − 150,000

DCL = 4 X 2.5 = 10

232
After Change
𝑄𝑄(𝑃𝑃 − 𝑉𝑉) 1,500,000(10.00 − 8.67)
𝐷𝐷𝐷𝐷𝐷𝐷 = = = 2.68
𝑄𝑄(𝑃𝑃 − 𝑉𝑉) − 𝐹𝐹 1,500,000(10.00 − 8.67) − 1,250,000

𝐸𝐸𝐸𝐸𝐸𝐸𝐸𝐸 750,000
𝐷𝐷𝐷𝐷𝐷𝐷 = = = 3.0
𝐸𝐸𝐸𝐸𝐸𝐸𝐸𝐸 − 𝐼𝐼 750,000 − 500,000

DCL = 2.68 X 3 = 8.04

233
Chapter 8: Time Value of Money

234
Chapter 8 Readings

235
THIS MANUSCRIPT APPEARS IN THE JOURNAL OF THE AMERICAN ACADEMY OF BUSINESS
CAMBRIDGE, Vol. 2(1), September 2002, p. 72-79
A New Method for Teaching the Time Value of Money
Terrance Jalbert, University of Hawaii at Hilo

ABSTRACT

Students frequently experience difficulty in identifying the appropriate time value of money (TVM) technique to
apply to a problem. This paper surveys the TVM presentation in seven popular introductory finance textbooks. A
new presentation technique is then developed. The presentation technique is based on a simple method for
identifying the appropriate TVM technique to apply to any problem. TVM techniques conducive to applying the
calculations in a generalized setting are then presented. Visual aids are provided to assist students in selecting
correct techniques. By using these techniques students are able to more easily identify appropriate TVM techniques.

INTRODUCTION

Many techniques have been developed for presenting the time value of money (TVM). Despite this considerable
effort on the part of instructors, students frequently experience difficulty identifying the appropriate technique to
apply to a specific problem (Eddy and Swanson, 1996). However, it is well known that a pedagogy, which works
well with one audience, does not necessarily work well with another (Bloom, 1956). Thus, the development of new
and different techniques that appeal to various audiences is beneficial. This paper develops a new technique for
teaching the TVM. The technique is specifically intended to appeal to students that benefit from precise definitions
and visual aids. The technique affords instructors a new tool in their arsenal to teach students TVM concepts. The
paper begins by surveying how seven popular introductory finance textbooks address the TVM issue. Next, a new
approach for teaching the TVM is presented. The approach begins with a simple method for distinguishing between
a single sum of money, annuity, perpetuity, growing perpetuity and uneven cash flow stream. Cash flows are
distinguished by examining conditions that must be met in order for a series of cash flows to qualify for each
classification. TVM techniques conducive to applying the calculations in a generalized setting are then presented.
Finally visual aids are provided to walk students through selecting the appropriate TVM technique for a problem.

A SURVEY OF INTRODUCTORY FINANCE TEXTS

Students nearly unanimously experience difficulty in identifying the appropriate technique to apply to TVM
problems. While the TVM issue is complex, some of the difficulty can be attributed to the approach that finance
texts take to the issue. This contention is confirmed by Eddy and Swanson who argue that instructors do not
sufficiently develop a frame of reference which begins with simple learning objectives focused on individual topics
and progresses to higher levels of understanding (Eddy and Swanson, 1996). This section contains a survey of the
approaches used in seven popular finance texts to present TVM concepts. The seven books examined in this study
are: Stanley Block and Geoffrey Hirt (BH) eighth edition of Foundations of Financial Management, 2) Charles
Moyer, James McGuigan and William Kretlow (MMK) eighth edition of Contemporary Financial Management, 3)
Zvi Bodie and Robert Merton (BM), first edition of Finance 4) Gary Emery (EM), first edition of Corporate Finance
Principles and Practice 5) Arthur Keown, David Scott, John Martin, and William Petty (KSMP), seventh edition of
Basic Financial Management, 6) William R. Lasher (LA), second edition of Practical Financial Management, and 7)
Eugene Brigham, Louis Gapenski and Michael Ehrhardt (BGE), ninth edition of Financial Management Theory and
Practice. These books are believed to be a representative cross section of texts used in introductory finance courses.

The survey revealed three common areas of concern regarding TVM presentations. The first concern is the balance
authors must make between overly detailed explanations and explanations that are overly simplistic. Regardless of
the level of detail provided, basic elements of the concept must be incorporated to ensure a complete understanding.
Failure to include the basic elements leads to a presentation that is imprecise and confusing. In each of the finance
texts surveyed, a definition for an annuity is provided. EM (p. 100) defines an annuity to be a multipayment
problem with equal periodic cash flows. BH (p. 235) define an annuity to be a series of consecutive payments or
receipts of equal amount. MMK (p. 137) define an annuity to be the receipt or payment of equal cash flows per
period for a specified amount of time. KSMP (p. 178) define an annuity to be a series of equal dollar payments for a

236
specified number of years. LA (p.142) defines an annuity to be a stream of equal payments, made or received,
separated by equal intervals of time. BM (p. 100) define an annuity to be a level stream of cash flows. BGE (p.
250) define an annuity to be a series of equal payments made at fixed intervals for a specified number of periods.
These definitions vary widely in their precision. Definition variations are also found with regard to the discussion of
uneven cash flow streams. EM, BH, KSMP and BM, do not address the issue of uneven cash flow streams in their
TVM chapters. MMK (p. 147), LA (p. 168), and BGE (258) introduce uneven cash flow streams as a means of
resolving the TVM problem that occurs because of unequal payment amounts. These definitions are generally
imprecise and leave open questions that the students must infer from example problems. More precise definitions
should help students more easily identify situations where the technique is appropriate.

The second concern that the survey revealed is incomplete explanations of how TVM techniques can be utilized.
Specifically, the surveyed texts do not sufficiently address the issue of the present and future values of annuities due
and ordinary annuities. Each of the books surveyed addresses the issue by noting that an ordinary annuity involved
receiving the payments at the end of each year and an annuity due involves receiving the payments at the beginning
of each year. While the statements made are technically correct, they lack generality. Students are frequently
unable to generalize the techniques to advanced applications such as deferred annuities.

The third concern is the introduction of TVM techniques in multiple chapters throughout the book. Spreading the
presentation of TVM techniques across multiple chapters increases the difficulty of integrating the material into a
broad understanding. This problem is common in the presentation of growing perpetuities. The technique is
frequently included in the stock valuation chapter rather than the TVM chapter. BM, BH, MMK, KSMP, LA and
BGE do not discuss growing perpetuities as part of the TVM chapter. EM (p. 105) is the only author to discuss
growing perpetuities as part of his TVM chapter.

In the remainder of this paper, a new approach for presenting TVM techniques is provided. The methodology
developed in this paper addresses each of the three issues identified in the survey. The method incorporates
definitions that allow students to more easily identify appropriate TVM techniques to apply to a problem. It
compactly discusses TVM techniques in one location and clearly distinguishes between each of the techniques. It is
developed so that students can generalize the techniques to advanced problems. The method begins by identifying
characteristics of cash flows that must be examined to determine which TVM technique is appropriate for the
problem. Visual aids are presented in Figure 1 and Table 1. These visual aids are often helpful for students to
internally package the materials in their minds. Readers should consult these visual aids throughout the following
discussion.

CASH FLOW CHARACTERISTICS

To determine the appropriate TVM technique to apply to a problem, four characteristics of the cash flows must be
analyzed. By examining these four characteristics, the appropriate TVM technique for any basic problem can be
identified. Moreover, the examination reveals situations where advanced techniques are required. The four
characteristics to be examined are: 1) Is there a series of cash flows? 2) Is the number of cash flows limited? 3) Is
there equal time spacing between each of the cash flows? and 4) Is the dollar amount of each cash flow equal? The
first characteristic to examine is if there is a single cash flow or a series of cash flows. A single cash flow implies
that there is only one cash flow, whereas a series of cash flows implies that there are more than one cash flow. An
investment that promises to pay $100 per year at the end of each of the following three years is a series of cash
flows. The investment provides three cash flows of $100 each. Alternatively, an investment that promises to pay a
single amount of $1,000, five years from today is not a series of cash flows. If a problem involves a single sum, it is
not necessary to examine the remaining characteristics. The correct technique is to compute the present or future
value of a single sum. Techniques for computing the present and future values of single sums are well known and
are not discussed here. If a problem involves a series of cash flows, the remaining three characteristics must be
examined to determine the appropriate TVM technique to apply to the problem. The second characteristic that must
be examined is if there are a limited number of cash flows or if the cash flows continue into infinity. An investment
that promises to pay a $100 cash flow on January 1st of each of the next three years has a limited number of cash
flows. An investment that pays a $100 cash flow on January 1st each year into infinity has an infinite number of
cash flows. The third characteristic to be examined is if there is an equal amount of time between each of the cash
flows. This means that the cash flows must occur at some constant time interval such as one cash flow per month,
or one cash flow per year. An investment that promises to pay us $100 on January 1st of each of the next three years

237
has equal time spacing. There is a one-year interval between each of the cash flows. On the other hand, an
investment that promises to pay $100 on January 1st of the next year, nothing on January 1st of the second year, $100
on January 1st of the third year and $100 on January 1st of the fourth year does not have equal time spacing. In this
case, the interval between the first two cash flows is two years while the interval between the third and fourth is one
year. Finally, The fourth characteristic to be examined is if each cash flow is for an equal dollar amount or for
varying dollar amounts. An investment that promises to pay $100 per year at the end of each of the next three years
has equal dollar amounts as each cash flow is for $100. An investment that promises to pay $100 the first year and
$200 the second year does not have equal dollar amounts. While there are a series of cash flows, the cash flows are
not for the same dollar amount. By examining these four characteristics, the appropriate technique to apply to any
TVM problem can be identified.

ANNUITIES

If the answer to each of the four questions in the previous section is yes, a series of cash flows qualifies as an
annuity. An annuity thus has the following characteristics: 1) there is series of cash flows, 2) the number of cash
flows limited, 3) there is equal time spacing between each of the cash flows and 4) the dollar amount of each cash
flow equal. The formal definition of an annuity that follows is a series of cash flows where the number of cash
flows is limited, there is an equal time spacing between each of the cash flows, and the dollar amount of each cash
flow is for an equal amount. Defining an annuity in this way explicitly invites the student to examine each of the
four characteristics. BGE is the only book surveyed that addresses each of the four characteristics in their definition.

Once it is established that a series of cash flows qualifies as an annuity, the present value or the future value of the
cash flows can be computed using techniques designed specifically for an annuity. Two options are available for
calculating the present value of an annuity using most financial calculators and TVM tables. The first option is to
compute the value of the cash flows at the time of the first cash flow by computing the present value of an annuity
due. The second option is to compute the value of the cash flows one time period before the first cash flow by
computing the present value of an ordinary annuity.

Suppose a loan obligates the borrower to deposit three $100 cash flows into an account. The first cash flow will be
made three years from today. The second cash flow will be made four years from today and the third cash flow will
be made five years from today. The appropriate discount rate for these cash flows is assumed to be ten percent. The
following time line depicts this annuity. The time line also shows the point in time at which we calculate the value
of the annuity if we apply the present value of an ordinary annuity technique (PVOA) and if we apply the present
value of an annuity due technique (PVAD) to the cash flows:

0 1 2 3 4 5 6 7
|----------|----------|----------|----------|----------|----------|----------| Fig. 2.1
100 100 100
PV PV
OA AD

The computations for the present and future value of an annuity can be made in several ways including by formula,
TVM tables and financial calculators. By applying the present value of an annuity due technique to this stream of
cash flows, the value of the cash flows at time three is computed to be $273.55. Applying the present value of an
ordinary annuity calculation to this stream of cash flows, the value of the cash flows at time two on the time line is
computed to be $248.69.

Two alternatives are also available for calculating the future value of an annuity. The first alternative is to compute
the value of the cash flows at the time of the last cash flow by computing the future value of an ordinary annuity.
The second alternative is to compute the value of the cash flows one time period after the last cash flow by
computing the future value of an annuity due. The annuity consisting of three $100 cash flows is again shown on
the following time line. The time line also shows the point in time at which the value of the annuity is calculated if
we apply the future value of an ordinary annuity technique (FVOA) and if we apply the present value of an annuity
due technique (FVAD) to the annuity.

238
0 1 2 3 4 5 6 7
|----------|----------|----------|----------|----------|----------|----------| Fig. 2.2
100 100 100
FV FV
OA AD

The appropriate interest rate is again ten percent. Applying the future value of an annuity due calculations to this
stream of cash flows, the value of the cash flows at time six is computed to be $364.10. Applying the future value
of an ordinary annuity calculation to this stream of cash flows, the value of the cash flows at time five on the time
line is computed to be $331.00.

By combining Figures 2.1 and 2.2, a generalized illustration for selecting the appropriate annuity calculation
technique is created. The figure is augmented by showing the mode financial calculators must be set in to make the
calculations. The following figure facilitates easy identification of the appropriate computational technique for any
annuity problem. Students are required to commit this illustration to memory.

A B C D E F G H I
|----------|----------|----------|----------|----------|----------|----------|----------| Fig. 2.3
First Middle Last
Cash Flow Cash Flows Cash Flow

PV PV FV FV
OA AD OA AD
Mode END BEG END BEG

Figure 2.3 shows that the value of any annuity can be computed at four different points on the time line: 1) one time
period before the first cash flow (PVOA), 2) at the time of the first cash flow (PVAD), 3) at the time of the last cash
flow (FVOA), and 4) one time period following the last cash flow (FVAD). Letters are intentionally used in Figure
2.3 to help students generalize the illustration to any time period.

To illustrate the use of Figure 2.3, consider two annuities. Both annuities will provide cash flows of $500 per year
for the next five years. The first annuity (Fig. 2.4) will provide the first cash flow today. The second annuity (Fig.
2.5) will provide the first cash flow one year from today. The objective is to compute the value of either annuity
today, at time 0 on the time line.

0 1 2 3 4 5
|----------|----------|----------|----------|----------| Fig. 2.4
500 500 500 500 500

0 1 2 3 4 5 6
|----------|----------|----------|----------|----------|-----------| Fig. 2.5
500 500 500 500 500

For the first annuity (Fig. 2.4), the first cash flow occurs at time zero. Time zero is also the point in time at which
the value of the cash flows will be computed. As such, the present value will be computed at the time of the first
cash flow. Referring to Figure 2.3, it is seen that in order to compute the value of the cash flows at the time of the
first cash flow, the present value of an annuity due should be computed. For the second annuity (Fig. 2.4), the first
cash flow occurs at time one. Time zero is the point in time at which the value of the cash flows will be computed.
As such, the present value will be computed one time period before the first cash flow. Referring to Figure 2.3, it is
seen that in order to compute the value of the cash flows one time period before the first cash flow, the present value
of an ordinary annuity should be computed. The value of the second annuity could also be computed at time one on
the time line by applying the present value of an annuity due computations to the annuity.

A similar analysis can be made for computing future values. The value of the first annuity (Fig. 2.4) can be
computed at either time four or time five by using the annuity techniques. Referring to Figure 2.3, it is seen that in

239
order to calculate the value of the annuity at time four, the future value of an ordinary annuity is computed. In order
to compute the value of the annuity at time five, the future value of an annuity due is computed. The value of the
second annuity (Fig. 2.5) can be computed at either time five or time six using annuity techniques. Again referring
to figure 2.3, it is seen that in order to calculate the value of the annuity at time five, the future value of an ordinary
annuity is computed. In order to compute the value of the annuity at time six, the future value of an annuity due is
computed.

PERPETUITIES

The conditions for a series of cash flows to qualify as an annuity are not always met. Fortunately, other TVM
techniques are available when the conditions are not met. The correct technique depends upon which of the
conditions are not met. When conditions 1,3, and 4 of an annuity are met, but the second condition of an annuity is
not, the series of cash flows is referred to as a perpetuity. A perpetuity is a series of cash flows that continue
forever, where there is an equal time spacing between each of the cash flows and the dollar amount of each cash
flow is for an equal amount. As such, three of the annuity conditions are met. However, the second condition of an
annuity is violated because the cash flows go on forever. The investment will give the investor some equal cash
flow each year forever. An example of a perpetuity is an investment that will give the investor $100 each year
forever.

Once it is established that a series of cash flows qualifies as a perpetuity, the present value of the cash flows can be
computed using techniques designed specifically for a perpetuity. Two alternatives are available for calculating the
present value of a perpetuity. The correct technique depends upon when the next cash flow will be received. The
present value one time period before the receipt of a cash flow is due can be calculated as the present value of an
ordinary perpetuity. The present value immediately prior to the receipt of a cash flow can be calculated as the
present value of a perpetuity due.

Suppose that an investor wishes to calculate the present value a perpetuity that will pay her $100 per year each year
forever. The first cash flow will be received at the end of the third year. The appropriate discount rate is assumed to
be ten percent. The following time line illustrates this perpetuity. The time line also shows the point in time at
which the value of the perpetuity is calculated if we apply the present value of an ordinary perpetuity technique
(PVOP) and if we apply the present value of a perpetuity due technique (PVPD) to the cash flows.

0 1 2 3 4 5 6 7
|----------|----------|----------|----------|----------|----------|----------|………… Fig. 3.1
100 100 100 100 100 ………
PV PV
OP PD

CF
The formula for computing the present value of a perpetuity due is: PVPD = + CF . The formula for
i
CF
computing the present value of an ordinary perpetuity is PVOP = . Where CF denotes the amount of the cash
i
flows and i denotes the appropriate discount rate for the cash flows. Computing the present value of an ordinary
perpetuity for these cash flows, the value of the cash flows at time two is computed to be $1,000. Computing the
present value of a perpetuity due for these cash flows, the value of the cash flows at time three is computed to be
$1,100.

GROWING PERPETUITIES

Another combination of the conditions of an annuity that may not be met occurs when the first and third conditions
are met but the second and fourth conditions are not met. In this case, there are a series of cash flows which have an
equal amount of time between each of the cash flows. Thus the first and third conditions of an annuity are met.
However, the second assumption of an annuity is not met because the cash flows continue forever. In addition, the
fourth condition of an annuity is not met because the cash flows are not equal. While the cash flows are not equal, a
different condition does apply with regard to the magnitude of the cash flows. That condition is that the cash flows

240
must become larger by some constant percentage amount in each successive time period. When this combination of
conditions are present, the series of cash flows is referred to as a growing perpetuity. Specifically, a growing
perpetuity is a series of cash flows that continue forever, where there is a equal time spacing between each of the
cash flows, and the dollar amount of each cash flow increases by some constant percentage amount in each
successive time period. In a manner similar to a perpetuity, the value of the cash flows can be computed at two
different points on the time line. The present value of a growing perpetuity due technique (PVGPD) computes the
value of the cash flows immediately prior to receiving a cash flow. The present value of an ordinary growing
perpetuity technique (PVGOP) computes the value of the cash flows one time period prior to receiving a cash flow.
An example of a growing perpetuity is an investment that promises to pay $100 at time 3. The investment will
continue to provide annual cash flows forever, however, the cash flows will become five percent larger in each
successive year. The appropriate discount rate for this growing perpetuity is assumed to be ten percent. The
following time line illustrates this growing perpetuity. The time line also shows the points on the time line that the
value of the growing perpetuity is calculated if we apply the present value of an ordinary growing perpetuity and
present value of a growing perpetuity due calculations to the cash flows.

0 1 2 3 4 5 6 7
|----------|----------|----------|----------|----------|----------|----------|………. Fig. 4.1
100 105 110.25 115.76 121.55…..
PV PV
GOP GPD

The formula for calculating the present value of a growing ordinary perpetuity is PVGOPN = CFN + 1 . The formula
(i − g )
for calculating the present value of a growing perpetuity due is PVGPDN = CF N +1
+ CFN . Where N denotes the
(i − g )
time period and g denotes the percentage growth rate. If the present value of a growing perpetuity due technique is
applied to these cash flows, the value of the cash flows at time three on the time line is computed to be $2,200. If the
present value of a growing ordinary perpetuity technique is applied to these cash flows, the value of the cash flows
at time two on the time line is computed to be $2,000.

UNEVEN CASH FLOWS

Another combination of the conditions for an annuity that may not be met occurs when the first and second
conditions of an annuity are met, but the third and fourth conditions of an annuity are not met. In this case, we have
a series of cash flows and the number of cash flows is limited. Thus, the first and second conditions of an annuity
are met. However, the time spacing between the cash flows may or may not be equal and the dollar amount of the
cash flows may or may not be equal in each year. As such, the third and fourth conditions of an annuity are not met.
When this combination of conditions occurs, the series of cash flows is called an uneven cash flow stream. Thus an
uneven cash flow stream is formally defined as a series of cash flows, where the number of cash flows is limited,
there may or may not be an equal time spacings between the cash flows and the cash flows may or may not be for
equal amounts. An example of an uneven cash flow stream is an investment that will give us $100 three years from
today, nothing at the end of the fourth year, $100 at the end of the fifth year and $200 at the end of the sixth year.
These cash flows are illustrated on the following time line:

0 1 2 3 4 5 6 7
|----------|----------|----------|----------|----------|----------|----------| Fig. 5.1
100 0 100 200

The present value and the future value of an uneven cash flow stream can be calculated. To calculate present value
or the future value of an uneven cash flow stream, each cash flow must be treated as a single sum and the present
value or future value of each single sum must be calculated individually. The time at which the value of the cash
flows is computed can be customized by adjusting the number of time periods used in the calculations. Many
calculators are equipped with a feature that allows the user to input all of the cash flows into the calculator at one
time. The calculator then simultaneously computes the present value of each of the individual cash flows and totals

241
them. Using these techniques, the present value of the cash flows at time three is computed to be $332.91. The
future value of this uneven cash flow stream at time seven on the time line is computed to be $487.41.

VISUAL AIDS

Table 1 and Figure 1 provide two visial aids for seleting the appropriate TVM technique for a problem. These visual
aids are often helpful for students to internally package the materials in their minds. Figure 1 is a flowchart that
walks the user through each of the cash flow characteristics. By answering yes or no to several questions, the user
can determine the appropriate TVM technique for any problem. In addition to pointing the student to the correct
technique, the table identifies two specific situations in which a combination of techniques must be utilized to solve
the problem. The precise combination of techniques that must be used is case specific. Table 1 shows the
conditions necessary for a series of cash flows to be classified as a single sum, an annuity, a perpetuity, a growing
perpetuity or an uneven cash flow stream. The rows indicate each of the four cash flow characteristics. The
columns indicate the four possible classifications of a series of cash flows. The cells indicate if the condition must
be present or absent in order for the cash flows to qualify for the classification.

CONCLUDING COMMENTS

Students frequently experience difficulty identifying the appropriate time value of money (TVM) technique to apply
to a problem. In this paper it is suggested that this difficulty might be due in part to imprecise definitions. Seven
textbooks are surveyed to determine common teaching techniques. Next, a new method for presenting TVM
techinques is presented. The method is based on precisely distingushing between single sums, annuities,
perpetuities, growing perpetuities and uneven cash flow streams. The distinguishment is based on the examination
of four characteristics of a series of cash flows. Visual aids are developed to provide students an easy means of
navigating the maze of TVM techniques. The technique developed here afford instructors a new tool in their arsenal
to teach students TVM skills. The technique is specifically intended to appeal to students that benefit from precise
definitions and visual aids.

242
FIGURE 1: FLOWCHART FOR SELECTING APPRPRIATE TIME VALUE OF MONEY TECHNIQUES

Stream of Cash Yes Limited Number Yes Equal Time Yes Equal Dollar Yes
Annuity
Flows? of Cash Flows? Spacing? Amount?

No No No No

Uneven Cash Uneven Cash


Single Sum
Flow Stream Flow Stream

Equal Time Yes Equal Dollar Yes


Perpetuity
Spacing? Amount?

No No

Constant Yes Growing


Percentage
Perpetuity
Growth?

No

Combination Combination
Problem Problem

TABLE 1: CLASSIFICATION OF CASH FLOWS

Single Annuity Perpetuity Growing Uneven Cash


Sum Perpetuity Flow Stream
Series of Cash Flows Absent Present Present Present Present
Limited Number of Cash Flows Present Present Absent Absent Present
Each Cash Flow is for an Equal $ Amount N/A Present Present Absent Absent
Equal Time Interval Between Cash Flows N/A Present Present Present Absent
REFERENCES

Block, Stanley and Geoffrey Hirt (1997). Foundations of Financial Management. 8th ed. Boston: Irwin.
Bloom, B. (1956). Taxonomy of Educational Objectives, Handbook I: Cognitive Domain, New York: McKay.
Bodie, Zvi and Robert Merton (2000). Finance. Upper Saddle River, NJ: Prentice Hall.
Brigham, Eugene, Louis Gapenski, Michael Ehrhardt (1999). Financial Management Theory and Practice, 9th ed.
Fort Worth: Dryden.
Eddy, Albert and Gene Swanson (1996), “A Hierarchy of Skills Approach to Teaching Accounting Present Value,”
Journal of Accounting Education 14(1) p. 123-131.
Emery, Gary (1998). Corporate Finance, Reading, Massachusetts: Addison-Wesley.
Keown, Arthur, et al. (1996). Basic Financial Management, 7th ed. Upper Saddle River, NJ: Prentice Hall 1996.
Lasher, William (2000). Practical Financial Management. 2nd ed. Cincinnati: South-Western.
Moyer, Charles, James McGuigan, William Kretlow (2001). Contemporary Financial Management. 8th ed.
Cincinnati: South-Western.

243
Chapter 8 Lecture Notes

244
Chapter 8: Time Value of Money

In this chapter you will learn Time Value of Money Techniques.


Time Value of Money Techniques are the Foundation of much of
what we do in finance.

It is critical that you have a thorough understanding of this material


for the remainder of the course.

Usefulness
Time Value of Money Techniques are useful both for business
decisions and in your personal life. Time Value of Money
Techniques allow you to answer questions like:

If I deposit $100 in the bank today into an account that earns 10


percent interest how much money will I have in the bank 5 years
from today?

If I deposit $100 at the end of each of the next 5 years into an


account that pays 10 percent interest. How much money will I
have in the bank 5 years from today?

I borrowed $100,000 to buy a house. The interest rate is 6 percent


nominal compounded monthly. You will pay off the loan in
monthly payments over 30 years. The monthly payment is
$599.55. If instead of making $599.55 payments you make
$699.55 payments, how much sooner will you pay off the loan?

245
Time Lines
A graphical technique to display the passage of time

0 1 2 3 4 5 6 7

|----------|----------|----------|----------|----------|----------|----------|

Key Language

Today point 0
One year from today point 1
Two years from today point 2
The beginning of the first year point 0
The end of the first year point 1
The beginning of the second year point 1

The beginning of each of the next three years

0 1 2 3 4 5 6 7

|----------|----------|----------|----------|----------|----------|----------|
* * *
The end of each of the next three years

0 1 2 3 4 5 6 7

|----------|----------|----------|----------|----------|----------|----------|
* * *

246
Nominal versus Periodic Interest

Nominal Interest – The Interest Rate quoted by the bank

-- The nominal interest rate is always given as an


annualized number

Periodic Interest Rate – The interest rate for some fraction of a full
year.

Periodic Interest = Nominal Interest X F

F = Fraction of a Year

Suppose that I go to the bank to borrow money. The bank tells me


that the interest rate on the loan will be 12 percent nominal. I will
borrow the money for one month. What is the periodic interest
rate for one month?

1
Periodic Interest = 12 X = 1%
12

247
Simple Interest versus Compound Interest
Simple Interest: Interest on the principal amount that you deposit
in the bank only.

Compound Interest: Interest on the principal amount that you


deposit in the bank, plus interest on any interest that you have
previously earned.

Simple Interest Illustration

Suppose that I deposit $100 into the bank today into an


account that will earn 10 percent simple interest per year.
How much money will I have in the bank 2 years from
today?

0 1 2

|--------------------------|------------------------------|

-100 10 10
Interest on $100 Interest on $100
$100 X .10 = $10 $100 X .10 = $10

So we have a total at the end of the second year of:

Initial Deposit $100


1st year Interest $10
2nd year Interest $10
Total $120

248
Compound Interest Illustration

Suppose that I deposit $100 into the bank today into an


account that will earn 10 percent nominal interest
compounded annually. How much money will I have in the
bank 2 years from today?

0 1 2

|--------------------------|------------------------------|

-100 10 11
Interest on $100 Interest on $100
$100 X .10 = $10 $100 X .10 = $10

Plus interest on $10


$10 X .10 = $1

So we can calculate our total as:


Initial Deposit $100
1st year Interest on initial deposit $10
2nd year Interest on initial deposit $10
2nd year Interest on Interest earned 1st year $1
Total $121

Compounding refers to the frequency that we stop and add the


interest on to the amount earning interest.

** Except where specifically noted, we will always be discussing


compound interest in the course

249
Technique Selection
Depending upon the type of problem under consideration, different
Time Value of Money (TVM) Techniques must be used to solve
the problem. The first key to solving a TVM problem is to select
the correct technique. To do that, you must answer four questions
regarding the problem:

1. Is there a Series of Cash Flows?

2. Is the number of Cash flows Limited?

3. Is there an equal time spacing between each of the cash


flows?

4. Is each cash flow for an equal dollar amount?

If you answer each of these questions correctly, it will direct you to


the correct TVM technique to use to solve any time value of
money problem

250
Flowchart for Selecting the Appropriate Time
Value of Money Technique

Stream of Cash Yes Limited Number Yes Equal Time Yes Equal Dollar Yes
Annuity
Flows? of Cash Flows? Spacing? Amount?

No No No No

Uneven Cash Uneven Cash


Single Sum
Flow Stream Flow Stream

Equal Time Yes Equal Dollar Yes


Perpetuity
Spacing? Amount?

No No

Constant Yes Growing


Percentage
Perpetuity
Growth?

No

Combination Combination
Problem Problem

251
Techniques
a. Single Sum

* One deposit or receipt of money only

i.e. I will deposit $100 in the bank today.

b. Annuity

* A series of cash flows


* There is a limited number of cash flows
* There is an equal time spacing between each cash flow
* Each cash flow is for an equal dollar amount

i.e. I will deposit $100 in the bank at the end of each of


the next 5 years.

c. Perpetuity

* A series of cash flows


* The cash flows continue into perpetuity
* There is an equal time spacing between each cash flow
* Each Cash Flow is for an equal dollar amount

i.e. I will receive $100 per year from an investment


each year forever.

252
d. Growing Perpetuity

* Series of Cash Flows


* The Cash Flows Continue forever
* There is an equal time spacing between each cash flow
* The cash flows are not for an equal amount.
Rather, the cash flows increase by some constant
percentage amount each year.

i.e. I will receive $100 from an investment at the end of


the current year. I will continue to receive payments
from the investment each year forever. However, the
payments will become 10 percent larger in each
successive year.

e. Uneven Cash Flow Streams

* Series of Cash Flows


* Limited Number of Cash Flows
* May or may not be an equal time spacing
between each cash flow
* The cash flows may or may not be for an equal amount.

i.e. I will deposit $100 in the bank this year $200 one year
from today, and $400 two years from today.

253
f. Combination Problems (Multiple types of combination
problems

* Series of Cash Flows


* Unlimited Number of Cash Flows
* Equal time spacing between each cash flow
* Each cash flows is not for an equal dollar amount. The
cash flows do not grow at a constant percentage rate.

i.e. You will receive $100 from an investment at the end of


each of the next three years. You will continue to receive
payments each year forever. After three years, each
subsequent cash flow will be 5 % larger.

254
Methods for Solving Time Value of Money
Problems

1. Formula Method – you enter the necessary information


directly into a formula and solve the formula.

Advantages
a. you only need a simple calculator
b. Information in Books is generally presented using the
formula method.

Disadvantages
a. you must memorize the formulas
b. some problems are trial and error type of problems, so
it takes a great deal of time to solve the problem

2. Tables Method: Involves using Time value of money tables


to solve problems.

Advantages
a. you do not need to memorize the formulas
b. you only need a simple calculator

Disadvantages
a. You must have access to the tables
b. Some problems are trial and error type of problems, so
it takes a great deal of time to solve the problem.

255
3. Calculator

Advantages
a. you do not need to memorize the formulas
b. you can solve any problem quickly
c. may be used when taking the CFA examination

Disadvantage
a. you must spend $30 on a calculator
b. your can not store information for multiple problems
in memory.

4. Spreadsheet

Advantages
a. you do not need to memorize the formulas
b. You can solve any problem quickly
c. You can store problems in memory for later use

Disadvantages
a. You must purchase a computer and spreadsheet
Program
b. It is not a particularly intuitive way to learn TVM

In this class, we will primarily use the calculator method.


However, you will be required to know how to use some formulas.

256
To Set your Calculator for Use in This Class

To Set the Number of Payments Per Year.

2nd P/Y 1 Enter CLR CLR

To Set the Number of Decimal Places Displayed

2nd Format 9 Enter CLR CLR

257
To Clear Your Calculator
a. To clear what is showing on your screen

Clr

b. To clear the time value of money memory

2nd ClR TVM

c. To clear a work area in your calculator

i. enter the work area you wish to clear


ii. 2nd CLR WRK

For example, to clear the cash flow area of your


calculator

i. CF
ii. 2nd CLR WRK

* Make Sure to Clear Your Calculator After Each Problem

* If you are entering all of the information for a problem correctly


and you are still receiving the wrong answer, the most likely
reason is that your calculator is either not set correctly, or not
cleared properly.

258
Single Sum
A single amount of money:

Stream of Cash Yes Limited Number Yes Equal Time Yes Equal Dollar Yes
Annuity
Flows? of Cash Flows? Spacing? Amount?

No No No No

Uneven Cash Uneven Cash


Single Sum
Flow Stream Flow Stream

Equal Time Yes Equal Dollar Yes


Perpetuity
Spacing? Amount?

No No

Constant Yes Growing


Percentage
Perpetuity
Growth?

No

Combination Combination
Problem Problem

259
The future value or the present value of a single sum can be
computed:

a. Future Value of a Single Sum

Suppose I deposit $100 today into an account that pays 10


percent nominal interest compounded annually. How
much money will I have in the account 5 years from
today? I will not take any money out of the account.

0 1 2 3 4 5

|----------|----------|----------|----------|-----------|
-100 ?

PV = $100
N=5
I = 10
CPT FV = ?

FV = $161.05

Formula Method

FV=PV(1+i)N * you must memorize this formula

FV=$100(1+0.10)5

FV= $161.05

In your calculator: 1.10 "yx" 5 = X $100 = $161.05

Note that your calculator requires the interest rate in percentage


form, but the formula requires the interest rate in decimal form.

260
b. Present Value of a Single Sum

I wish to have $100 available to me 3 years from today. I


will deposit just enough money into an account today for
that purpose. The account will pay 10 percent nominal
interest compounded annually. How much do I need to
deposit today?

0 1 2 3 4 5

|----------|----------|----------|----------|-----------|
? 100

FV = $100
N=3
I = 10
CPT PV = ?

PV = $75.13

Formula Method
𝐹𝐹𝐹𝐹
𝑃𝑃𝑃𝑃 = * you must memorize this formula
(1+𝑖𝑖)𝑁𝑁

100
𝑃𝑃𝑃𝑃 =
(1 + 0.10)3

100 100
𝑃𝑃𝑃𝑃 = =
(1 + 0.10)3 1.331

PV = $75.13

261
c. Solving for other variables in a single sum problem

Depending upon which information we know, we can


solve for other variables in TVM Problems

i. Solving for the Interest Rate

Suppose my friend wants to make a loan from me. He


tells me that if I give him $100 today, he will give me
$140 three years from today. What interest rate am I
earning?

0 1 2 3 4 5

|----------|----------|----------|----------|-----------|
-100 140

FV = $140.00
PV = -100
N=3

CPT I = ?

I = 11.87%

262
ii. Solving for N

Suppose that I will deposit some money into the bank


today. How long will it take me to double my money if I
earn 20% nominal interest compounded annually?

0 ?

|----------|----------|----------|----------|-----------|
-$100 $200

PV = -100
FV = $200
I = 20
CPT N = ?

N = 3.8 years

Note: When you input both the present value and a future
value into your calculator in the same problem,
you must enter one as a negative number. I
recommend that you always enter the present
value as a negative number.

263
Effective Annual Interest Rates
Sometimes interest rates are quoted in non-annually compounded
terms. We would like to convert these quotations into an annually
compounded equivalent for direct comparison.

Which would you prefer?

10 percent nominal interest compounded annually


10 percent nominal interest compounded semi-annually

10 percent nominal interest compounded annually


9.5 percent nominal interest compounded monthly

To compare these rates, we compute the effective annual rates.


The effective annual rate is the annually compounded equivalent
interest rate.

The effective annual rate allows us to answer the


questions

9.5 percent nominal interest compounded monthly


equals _____ nominal interest compounded annually

Your calculator has a function that allows you to make the


computations.

2nd Iconv
Nom = 9.5 ↵ ↓ ↓
C/Y = 12 ↵ ↓ ↓
EFF = ?, CPT, EFF = 9.925

So: 9.5 percent nominal interest compounded monthly


equals 9.925% nominal interest compounded annually

264
Other Compounding Periods
Suppose that you will deposit $100 into the bank in an account that
pays 10 percent nominal interest compounded semi-annually.
How much money will you have in the bank three years from
today?

0 1 2 3 4 5

|----------|----------|----------|----------|-----------|
-100 ?

There are two methods you can use to solve this problem

Method 1: Interest Rate Conversion Method

Step 1: Compute the Effective Annual Interest Rate

2nd Iconv
Nom = 10 ↵ ↓ ↓
C/Y = 2 ↵ ↓↓
EFF = ?, CPT, EFF = 10.25

Step 2: Compute the Future Value using the EFF interest


Rate

PV = -100
N=3
I = 10.25
CPT FV = ?

FV = 134.01

265
Method 2: Adjust the Calculation Method

Step 1: Compute the Number of interest earning periods:

Ncalc = N X M = 3 X 2 = 6

N = the number of years


M = the number of compounding periods per year

Step 2: Compute the Interest Earned Each Period

I 10
Icalc = = =5
M 2

I = the nominal rate


M = the number of compounding periods per year

Step 3: Solve for the Future Value using the Interest Rate
and Number of Periods Computed in Steps 1 and 2.

PV = 100
N=6
I=5
CPT FV = ?

FV = $134.01

Note: It is recommended that you use the Adjust the


Calculation Method. It will work in all situations. The
EFF method will not work in certain circumstances.

266
Continuous Compounding

Some bank accounts compound interest continuously. In this case


we must use a formula to compute the present or future values.

Continuously Compounded Future Value

Suppose that you will deposit $100 into the bank in an account that
pays 10 percent nominal interest compounded continuously. How
much money will you have in the bank three years from today?

𝐹𝐹𝐹𝐹 = 𝑃𝑃𝑃𝑃𝑒𝑒 𝑖𝑖𝑖𝑖 * you must memorize this formula

𝐹𝐹𝐹𝐹 = 100𝑒𝑒 .10(3)

𝐹𝐹𝐹𝐹 = $134.99

In your calculator this is entered as 0.10 X 3 = 2nd ex X 100 =


$134.99

Continuously Compounded Present Value

Suppose that you wish to have $100 in the bank three years from
today. The account pays 10 percent nominal interest compounded
continuously. How much do you need to deposit today?

𝑃𝑃𝑃𝑃 = 𝐹𝐹𝐹𝐹𝑒𝑒 −𝑖𝑖𝑡𝑡 * you must memorize this formula

𝑃𝑃𝑃𝑃 = 100𝑒𝑒 −.10(3)

𝑃𝑃𝑃𝑃 = $74.08

In your calculator this is entered as 0.10 X 3 =, +- 2nd ex X 100 =


$74.08

267
Annuities

Recall the characteristics of an annuity

1. Series of Cash Flows


2. Limited Number of Cash Flows
3. Equal Time Spacing Between Each Cash Flow
4. Each Cash Flow is for an Equal Amount

Stream of Cash Yes Limited Number Yes Equal Time Yes Equal Dollar Yes
Annuity
Flows? of Cash Flows? Spacing? Amount?

No No No No

Uneven Cash Uneven Cash


Single Sum
Flow Stream Flow Stream

Equal Time Yes Equal Dollar Yes


Perpetuity
Spacing? Amount?

No No

Constant Yes Growing


Percentage
Perpetuity
Growth?

No

Combination Combination
Problem Problem

Example Time Line for an Annuity

0 1 2 3 4 5 6

|----------|----------|----------|----------|-----------|------------|
100 100 100 100

268
Once we have confirmed that a series of cash flows is an annuity,
we can compute the value of the annuity at four different time
points:

Technique Mode

1. At the time of the first cash flow PVAD BEG

2. One time period before the first cash flow PVOA END

3. At the time of the last cash flow FVOA END

4. One Time Period after the last cash flow FVAD BEG

PVAD = Present Value of an Annuity Due


PVOA = Present Value of an Ordinary Annuity
FVOA = Future Value of an Ordinary Annuity
FVAD = Future Value of an Annuity Due

269
1. Annuity Technique Identifier for PV of an Ordinary Annuity

A B C D E F G H

|----------|----------|----------|----------|----------|----------|----------|

100 100 100 100 100

1st Middle Middle Middle Last


CF CF CF CF CF
?

PVOA
Mode END

2. Annuity Technique Identifier for PV of an Annuity Due

A B C D E F G H

|----------|----------|----------|----------|----------|----------|----------|

100 100 100 100 100

1st Middle Middle Middle Last


CF CF CF CF CF

PVAD
Mode BEG

270
3. Annuity Technique Identifier for FV of an Ordinary Annuity

A B C D E F G H

|----------|----------|----------|----------|----------|----------|----------|

100 100 100 100 100

1st Middle Middle Middle Last


CF CF CF CF CF

FVOA
Mode END

4. Annuity Technique Identifier for FV of an Annuity Due

A B C D E F G H

|----------|----------|----------|----------|----------|----------|----------|

100 100 100 100 100

1st Middle Middle Middle Last


CF CF CF CF CF

FVAD
Mode BEG

271
Combined Annuity Computation Technique Identifier

A B C D E F G H

|----------|----------|----------|----------|----------|----------|----------|

100 100 100 100 100

First Middle Middle Middle Last


CF CF CF CF CF
? ? ? ?
PVOA PVAD FVOA FVAD
Mode END BEG END BEG

272
Consider the following annuity

0 1 2 3 4 5 6 7

|----------|----------|----------|----------|----------|----------|----------|

200 200 200 200 200

If I want to compute the value of these cash flows at time 0, I use


the present value of an ordinary annuity (PVOA, END).

If I want to compute the value of these cash flows at time 1, I use


the present value of an annuity due (PVAD, BEG)

If I want to compute the value of these cash flows at time 5, I use


the future value of an ordinary annuity (FVOA, END)

If I want to compute the value of these cash flows at time 6, I use


the future value of an annuity due (FVAD, BEG)
To Switch your calculator from the beginning mode to the end
modes or from the end mode to the beginning mode:

2nd BGN 2nd Set CLR

Note that when your calculator is set in the beginning mode, you
will see BGN in the display

273
Examples of Annuities

Suppose that I will deposit $100 into the bank at the end of each of
the next 5 years. The account will pay 7 percent nominal interest
compounded annually. How much money will I have in the bank 5
years from today?

0 1 2 3 4 5 6 7

|----------|----------|----------|----------|----------|----------|----------|

100 100 100 100 100


?
I want to know the value at the time of the last cash flow. Thus I
use the future value of an ordinary annuity (FVOA, END)
technique.

PMT = 100
N=5
I=7
CPT FV = ?

FV = $575.07

274
Suppose that I want to have $10,000 available at the end of each of
the next 5 years for graduate school. I want to deposit just enough
money into an account today for that purpose. The account will
pay 6 percent nominal interest compounded annually. How much
money do I need to deposit today?

0 1 2 3 4 5 6 7

|----------|----------|----------|----------|----------|----------|----------|

? 10,000 10,000 10,000 10,000 10,000

I want to know the value one time period before the first cash flow.
Thus I use the present value of an ordinary annuity (PVOA, END)
technique.

PMT = 10,000
N=5
I=6
CPT PV = ?

PV = $42,123.64

275
Suppose that I will deposit $1,000 into the bank at the end of each
of the next 4 years. The account will pay 6 percent nominal
interest compounded annually. How much money will I have in
the bank 5 years from today?

0 1 2 3 4 5 6 7

|----------|----------|----------|----------|----------|----------|----------|

1,000 1,000 1,000 1,000


?
I want to know the value one time period after the last cash flow.
Thus I use the future value of an annuity due (FVAD, BEG)
technique.

PMT = 1,000
N=4
I=6
CPT FV = ?

FV = $4637.09

276
Suppose that I want to have $10,000 available at the beginning of
each of the next 5 years for graduate school. I want to deposit just
enough money into an account today for that purpose. The account
will pay 6 percent nominal interest compounded annually. How
much money do I need to deposit today?

0 1 2 3 4 5 6 7

|----------|----------|----------|----------|----------|----------|----------|

10,000 10,000 10,000 10,000 10,000


?
I want to know the value at the time of the first cash flow. Thus I
use the present value of an annuity due (PVAD, BEG) technique.

PMT = 10,000
N=5
I=6
CPT PV = ?

PV = $44,651.06

277
Solving for the Payment in an Annuity

Suppose I have $100,000 in the bank today. I wish to go to


graduate school for the next four years. I wish to take equal
amounts out of the account at the beginning of each of the next
four years to pay for school. The last withdrawal will exactly
empty the account. I will earn 8 percent nominal interest
compounded annually on any money remaining in the account.
How much can I withdraw each year?

0 1 2 3 4 5 6 7

|----------|----------|----------|----------|----------|----------|----------|

? ? ? ?
100,000

Beginning Mode
PV = 100,000
N=4
I=8
PMT = ? = $27,955.63

278
Suppose I want to have $50,000 in the bank 5 years from today. I
will make equal deposits into the bank at the end of each of the
next 5 years to fund the account. I will earn 8 percent nominal
interest compounded annually on any money in the account. How
much do I need to deposit each year?

0 1 2 3 4 5 6 7

|----------|----------|----------|----------|----------|----------|----------|

? ? ? ? ?
50,000

End Mode
FV = 50,000
N=5
I=8
PMT = ? = $8,522.82

279
Solving for the Interest Rate in an Annuity

Suppose that I deposit $200,000 in the bank today. I wish to take


$25,000 out of the account at the end of each of the next 20 years.
The last withdrawal will exactly empty the account. What interest
rate will I need to earn on the account to be able to do this?

0 1 2 3 4 5 ….. 20

|----------|----------|----------|----------|----------|----------|----------|

25,000 25,000 25,000 25,000 25,000 25,000 25,000


-200,000

End Mode
PV = -200,000
N = 20
PMT = $25,000
I = ? = 10.93%

280
Example of Annuity
You have just won the Texas Lottery. You will receive 20 equal
annual payments of $500,000 each. You will receive the first
payment today. You have decided that you want to get as much
money as you can get today. So you go to the bank to exchange
these payments for a lump sum of money today. The bank tells
you they will pay you based on an 8% nominal compounded
annually discount rate.

How much money will the bank give you today?

0 1 2 3 4…..….19
|----------|----------|----------|----------|----------|
500k 500k 500k 500k 500k 500k
?

Solution

BEG MODE

PMT = 500,000
N = 20
I=8
CPT PV = ?

PV = $5,301,799.60

281
Loan Amortization
Amortizing a loan is the process of designing a loan so that each of
your payments will be for the same amount.

This is done by varying the amount of interest that you pay and the
amount of principal that you pay on the loan each month.

While your payments will be for the same amount each month:

In the early months, you pay a large amount of interest and


very little principal.

In the later months of the loan, you pay a large amount of


principal and very little interest.

Example: Suppose you want to borrow $100,000 to buy a house


today. You will make monthly payments, (starting at the end of
the current month) to pay off the loan. The bank tells you that the
interest rate on the loan is 6 percent nominal, compounded
monthly. The loan will be for 30 years. Create an amortization
table for the loan.

Step 1: Compute the Payment Amount

In order to compute the payment amount, we must use the


“adjust the calculation method” that we learned previously

Ncalc = N X M = 30 X 12 = 360

I 6
Icalc = = = 0.5 %
M 12

282
PV = 100000
I = 0.5
N = 360
PMT = ? = 599.55

Step 2: Create an Amortization Table

(1) (2) (3) (4) (5) (6)


Month Beg Bal Payment Interest Principal End Bal

1 $100000 $599.55 $500.00 $99.55 $99,900.45


2 $99,900.45 $599.55 $499.50 $100.05 $99,800.40
3 $99,800.40 $599.55 $499.00 $100.55 $99,699.85
.
.
.
360 $597.09 599.55 $2.99 $596.56 *0.52

*The final difference is due to rounding. Typically the last


payment is adjusted slightly to correct for any rounding errors.

The Interest (4) is computed as the simple interest on the beginning


balance

Month 1: Beg Bal $100,000


Periodic Interest Rate 0.005
Interest $500

Month 2: Beg Bal $99,900.45


Periodic Interest Rate 0.005
Interest $499.50

283
The Principal (5) is computed as the payment minus the interest

Month 1: $599.55 - $500 = $99.55


Month 2: $599.55 - $499.50 = $100.05

The ending balance (6) is computed as the Beginning Balance –


Principal

Month 1: $100,000 - $99.55 = $99,900.45


Month 2: $99,900.45 - $100.05 = $99,800.40

284
Amortization Schedule
Month Beg Bal Payment Interest Prin End Bal
1 100000.00 599.55 500.00 99.55 99900.45
2 99900.45 599.55 499.50 100.05 99800.40
3 99800.40 599.55 499.00 100.55 99699.85
4 99699.85 599.55 498.50 101.05 99598.80
5 99598.80 599.55 497.99 101.56 99497.25
6 99497.25 599.55 497.49 102.06 99395.18
7 99395.18 599.55 496.98 102.57 99292.61
8 99292.61 599.55 496.46 103.09 99189.52
9 99189.52 599.55 495.95 103.60 99085.92
10 99085.92 599.55 495.43 104.12 98981.80
11 98981.80 599.55 494.91 104.64 98877.16
12 98877.16 599.55 494.39 105.16 98771.99
13 98771.99 599.55 493.86 105.69 98666.30
14 98666.30 599.55 493.33 106.22 98560.09
15 98560.09 599.55 492.80 106.75 98453.34
16 98453.34 599.55 492.27 107.28 98346.05
17 98346.05 599.55 491.73 107.82 98238.23
18 98238.23 599.55 491.19 108.36 98129.87
19 98129.87 599.55 490.65 108.90 98020.97
20 98020.97 599.55 490.10 109.45 97911.53
21 97911.53 599.55 489.56 109.99 97801.54
22 97801.54 599.55 489.01 110.54 97690.99
23 97690.99 599.55 488.45 111.10 97579.90
24 97579.90 599.55 487.90 111.65 97468.25
25 97468.25 599.55 487.34 112.21 97356.04
26 97356.04 599.55 486.78 112.77 97243.27
27 97243.27 599.55 486.22 113.33 97129.94
28 97129.94 599.55 485.65 113.90 97016.04
29 97016.04 599.55 485.08 114.47 96901.57
30 96901.57 599.55 484.51 115.04 96786.52
31 96786.52 599.55 483.93 115.62 96670.91
32 96670.91 599.55 483.35 116.20 96554.71
33 96554.71 599.55 482.77 116.78 96437.94
34 96437.94 599.55 482.19 117.36 96320.57
35 96320.57 599.55 481.60 117.95 96202.63
36 96202.63 599.55 481.01 118.54 96084.09
37 96084.09 599.55 480.42 119.13 95964.96
38 95964.96 599.55 479.82 119.73 95845.24
39 95845.24 599.55 479.23 120.32 95724.91
40 95724.91 599.55 478.62 120.93 95603.99
41 95603.99 599.55 478.02 121.53 95482.46
42 95482.46 599.55 477.41 122.14 95360.32
43 95360.32 599.55 476.80 122.75 95237.57
44 95237.57 599.55 476.19 123.36 95114.21

285
45 95114.21 599.55 475.57 123.98 94990.23
46 94990.23 599.55 474.95 124.60 94865.63
47 94865.63 599.55 474.33 125.22 94740.41
48 94740.41 599.55 473.70 125.85 94614.56
49 94614.56 599.55 473.07 126.48 94488.08
50 94488.08 599.55 472.44 127.11 94360.97
51 94360.97 599.55 471.80 127.75 94233.23
52 94233.23 599.55 471.17 128.38 94104.85
53 94104.85 599.55 470.52 129.03 93975.82
54 93975.82 599.55 469.88 129.67 93846.15
55 93846.15 599.55 469.23 130.32 93715.83
56 93715.83 599.55 468.58 130.97 93584.86
57 93584.86 599.55 467.92 131.63 93453.23
58 93453.23 599.55 467.27 132.28 93320.95
59 93320.95 599.55 466.60 132.95 93188.00
60 93188.00 599.55 465.94 133.61 93054.39
61 93054.39 599.55 465.27 134.28 92920.12
62 92920.12 599.55 464.60 134.95 92785.17
63 92785.17 599.55 463.93 135.62 92649.54
64 92649.54 599.55 463.25 136.30 92513.24
65 92513.24 599.55 462.57 136.98 92376.26
66 92376.26 599.55 461.88 137.67 92238.59
67 92238.59 599.55 461.19 138.36 92100.23
68 92100.23 599.55 460.50 139.05 91961.18
69 91961.18 599.55 459.81 139.74 91821.44
70 91821.44 599.55 459.11 140.44 91680.99
71 91680.99 599.55 458.40 141.15 91539.85
72 91539.85 599.55 457.70 141.85 91398.00
73 91398.00 599.55 456.99 142.56 91255.44
74 91255.44 599.55 456.28 143.27 91112.17
75 91112.17 599.55 455.56 143.99 90968.18
76 90968.18 599.55 454.84 144.71 90823.47
77 90823.47 599.55 454.12 145.43 90678.03
78 90678.03 599.55 453.39 146.16 90531.87
79 90531.87 599.55 452.66 146.89 90384.98
80 90384.98 599.55 451.92 147.63 90237.36
81 90237.36 599.55 451.19 148.36 90089.00
82 90089.00 599.55 450.44 149.11 89939.89
83 89939.89 599.55 449.70 149.85 89790.04
84 89790.04 599.55 448.95 150.60 89639.44
85 89639.44 599.55 448.20 151.35 89488.09
86 89488.09 599.55 447.44 152.11 89335.98
87 89335.98 599.55 446.68 152.87 89183.11
88 89183.11 599.55 445.92 153.63 89029.47
89 89029.47 599.55 445.15 154.40 88875.07
90 88875.07 599.55 444.38 155.17 88719.90
91 88719.90 599.55 443.60 155.95 88563.95
92 88563.95 599.55 442.82 156.73 88407.22

286
93 88407.22 599.55 442.04 157.51 88249.70
94 88249.70 599.55 441.25 158.30 88091.40
95 88091.40 599.55 440.46 159.09 87932.31
96 87932.31 599.55 439.66 159.89 87772.42
97 87772.42 599.55 438.86 160.69 87611.73
98 87611.73 599.55 438.06 161.49 87450.24
99 87450.24 599.55 437.25 162.30 87287.94
100 87287.94 599.55 436.44 163.11 87124.83
101 87124.83 599.55 435.62 163.93 86960.90
102 86960.90 599.55 434.80 164.75 86796.16
103 86796.16 599.55 433.98 165.57 86630.59
104 86630.59 599.55 433.15 166.40 86464.19
105 86464.19 599.55 432.32 167.23 86296.96
106 86296.96 599.55 431.48 168.07 86128.90
107 86128.90 599.55 430.64 168.91 85959.99
108 85959.99 599.55 429.80 169.75 85790.24
109 85790.24 599.55 428.95 170.60 85619.64
110 85619.64 599.55 428.10 171.45 85448.19
111 85448.19 599.55 427.24 172.31 85275.88
112 85275.88 599.55 426.38 173.17 85102.71
113 85102.71 599.55 425.51 174.04 84928.68
114 84928.68 599.55 424.64 174.91 84753.77
115 84753.77 599.55 423.77 175.78 84577.99
116 84577.99 599.55 422.89 176.66 84401.33
117 84401.33 599.55 422.01 177.54 84223.79
118 84223.79 599.55 421.12 178.43 84045.35
119 84045.35 599.55 420.23 179.32 83866.03
120 83866.03 599.55 419.33 180.22 83685.81
121 83685.81 599.55 418.43 181.12 83504.69
122 83504.69 599.55 417.52 182.03 83322.66
123 83322.66 599.55 416.61 182.94 83139.73
124 83139.73 599.55 415.70 183.85 82955.88
125 82955.88 599.55 414.78 184.77 82771.10
126 82771.10 599.55 413.86 185.69 82585.41
127 82585.41 599.55 412.93 186.62 82398.79
128 82398.79 599.55 411.99 187.56 82211.23
129 82211.23 599.55 411.06 188.49 82022.74
130 82022.74 599.55 410.11 189.44 81833.30
131 81833.30 599.55 409.17 190.38 81642.92
132 81642.92 599.55 408.21 191.34 81451.58
133 81451.58 599.55 407.26 192.29 81259.29
134 81259.29 599.55 406.30 193.25 81066.04
135 81066.04 599.55 405.33 194.22 80871.82
136 80871.82 599.55 404.36 195.19 80676.63
137 80676.63 599.55 403.38 196.17 80480.46
138 80480.46 599.55 402.40 197.15 80283.31
139 80283.31 599.55 401.42 198.13 80085.18
140 80085.18 599.55 400.43 199.12 79886.05

287
141 79886.05 599.55 399.43 200.12 79685.93
142 79685.93 599.55 398.43 201.12 79484.81
143 79484.81 599.55 397.42 202.13 79282.69
144 79282.69 599.55 396.41 203.14 79079.55
145 79079.55 599.55 395.40 204.15 78875.40
146 78875.40 599.55 394.38 205.17 78670.23
147 78670.23 599.55 393.35 206.20 78464.03
148 78464.03 599.55 392.32 207.23 78256.80
149 78256.80 599.55 391.28 208.27 78048.53
150 78048.53 599.55 390.24 209.31 77839.22
151 77839.22 599.55 389.20 210.35 77628.87
152 77628.87 599.55 388.14 211.41 77417.46
153 77417.46 599.55 387.09 212.46 77205.00
154 77205.00 599.55 386.03 213.52 76991.48
155 76991.48 599.55 384.96 214.59 76776.88
156 76776.88 599.55 383.88 215.67 76561.22
157 76561.22 599.55 382.81 216.74 76344.47
158 76344.47 599.55 381.72 217.83 76126.65
159 76126.65 599.55 380.63 218.92 75907.73
160 75907.73 599.55 379.54 220.01 75687.72
161 75687.72 599.55 378.44 221.11 75466.61
162 75466.61 599.55 377.33 222.22 75244.39
163 75244.39 599.55 376.22 223.33 75021.06
164 75021.06 599.55 375.11 224.44 74796.62
165 74796.62 599.55 373.98 225.57 74571.05
166 74571.05 599.55 372.86 226.69 74344.36
167 74344.36 599.55 371.72 227.83 74116.53
168 74116.53 599.55 370.58 228.97 73887.56
169 73887.56 599.55 369.44 230.11 73657.45
170 73657.45 599.55 368.29 231.26 73426.19
171 73426.19 599.55 367.13 232.42 73193.77
172 73193.77 599.55 365.97 233.58 72960.19
173 72960.19 599.55 364.80 234.75 72725.44
174 72725.44 599.55 363.63 235.92 72489.51
175 72489.51 599.55 362.45 237.10 72252.41
176 72252.41 599.55 361.26 238.29 72014.12
177 72014.12 599.55 360.07 239.48 71774.64
178 71774.64 599.55 358.87 240.68 71533.97
179 71533.97 599.55 357.67 241.88 71292.09
180 71292.09 599.55 356.46 243.09 71049.00
181 71049.00 599.55 355.24 244.31 70804.69
182 70804.69 599.55 354.02 245.53 70559.17
183 70559.17 599.55 352.80 246.75 70312.41
184 70312.41 599.55 351.56 247.99 70064.42
185 70064.42 599.55 350.32 249.23 69815.20
186 69815.20 599.55 349.08 250.47 69564.72
187 69564.72 599.55 347.82 251.73 69313.00
188 69313.00 599.55 346.56 252.99 69060.01

288
189 69060.01 599.55 345.30 254.25 68805.76
190 68805.76 599.55 344.03 255.52 68550.24
191 68550.24 599.55 342.75 256.80 68293.44
192 68293.44 599.55 341.47 258.08 68035.36
193 68035.36 599.55 340.18 259.37 67775.98
194 67775.98 599.55 338.88 260.67 67515.31
195 67515.31 599.55 337.58 261.97 67253.34
196 67253.34 599.55 336.27 263.28 66990.06
197 66990.06 599.55 334.95 264.60 66725.46
198 66725.46 599.55 333.63 265.92 66459.53
199 66459.53 599.55 332.30 267.25 66192.28
200 66192.28 599.55 330.96 268.59 65923.69
201 65923.69 599.55 329.62 269.93 65653.76
202 65653.76 599.55 328.27 271.28 65382.48
203 65382.48 599.55 326.91 272.64 65109.84
204 65109.84 599.55 325.55 274.00 64835.84
205 64835.84 599.55 324.18 275.37 64560.47
206 64560.47 599.55 322.80 276.75 64283.72
207 64283.72 599.55 321.42 278.13 64005.59
208 64005.59 599.55 320.03 279.52 63726.07
209 63726.07 599.55 318.63 280.92 63445.15
210 63445.15 599.55 317.23 282.32 63162.83
211 63162.83 599.55 315.81 283.74 62879.09
212 62879.09 599.55 314.40 285.15 62593.94
213 62593.94 599.55 312.97 286.58 62307.36
214 62307.36 599.55 311.54 288.01 62019.34
215 62019.34 599.55 310.10 289.45 61729.89
216 61729.89 599.55 308.65 290.90 61438.99
217 61438.99 599.55 307.19 292.36 61146.63
218 61146.63 599.55 305.73 293.82 60852.82
219 60852.82 599.55 304.26 295.29 60557.53
220 60557.53 599.55 302.79 296.76 60260.77
221 60260.77 599.55 301.30 298.25 59962.52
222 59962.52 599.55 299.81 299.74 59662.79
223 59662.79 599.55 298.31 301.24 59361.55
224 59361.55 599.55 296.81 302.74 59058.81
225 59058.81 599.55 295.29 304.26 58754.55
226 58754.55 599.55 293.77 305.78 58448.77
227 58448.77 599.55 292.24 307.31 58141.47
228 58141.47 599.55 290.71 308.84 57832.63
229 57832.63 599.55 289.16 310.39 57522.24
230 57522.24 599.55 287.61 311.94 57210.30
231 57210.30 599.55 286.05 313.50 56896.80
232 56896.80 599.55 284.48 315.07 56581.74
233 56581.74 599.55 282.91 316.64 56265.09
234 56265.09 599.55 281.33 318.22 55946.87
235 55946.87 599.55 279.73 319.82 55627.05
236 55627.05 599.55 278.14 321.41 55305.64

289
237 55305.64 599.55 276.53 323.02 54982.62
238 54982.62 599.55 274.91 324.64 54657.98
239 54657.98 599.55 273.29 326.26 54331.72
240 54331.72 599.55 271.66 327.89 54003.83
241 54003.83 599.55 270.02 329.53 53674.30
242 53674.30 599.55 268.37 331.18 53343.12
243 53343.12 599.55 266.72 332.83 53010.29
244 53010.29 599.55 265.05 334.50 52675.79
245 52675.79 599.55 263.38 336.17 52339.62
246 52339.62 599.55 261.70 337.85 52001.76
247 52001.76 599.55 260.01 339.54 51662.22
248 51662.22 599.55 258.31 341.24 51320.98
249 51320.98 599.55 256.60 342.95 50978.04
250 50978.04 599.55 254.89 344.66 50633.38
251 50633.38 599.55 253.17 346.38 50287.00
252 50287.00 599.55 251.43 348.12 49938.88
253 49938.88 599.55 249.69 349.86 49589.02
254 49589.02 599.55 247.95 351.60 49237.42
255 49237.42 599.55 246.19 353.36 48884.06
256 48884.06 599.55 244.42 355.13 48528.93
257 48528.93 599.55 242.64 356.91 48172.02
258 48172.02 599.55 240.86 358.69 47813.33
259 47813.33 599.55 239.07 360.48 47452.85
260 47452.85 599.55 237.26 362.29 47090.56
261 47090.56 599.55 235.45 364.10 46726.47
262 46726.47 599.55 233.63 365.92 46360.55
263 46360.55 599.55 231.80 367.75 45992.80
264 45992.80 599.55 229.96 369.59 45623.21
265 45623.21 599.55 228.12 371.43 45251.78
266 45251.78 599.55 226.26 373.29 44878.49
267 44878.49 599.55 224.39 375.16 44503.33
268 44503.33 599.55 222.52 377.03 44126.30
269 44126.30 599.55 220.63 378.92 43747.38
270 43747.38 599.55 218.74 380.81 43366.57
271 43366.57 599.55 216.83 382.72 42983.85
272 42983.85 599.55 214.92 384.63 42599.22
273 42599.22 599.55 213.00 386.55 42212.67
274 42212.67 599.55 211.06 388.49 41824.18
275 41824.18 599.55 209.12 390.43 41433.75
276 41433.75 599.55 207.17 392.38 41041.37
277 41041.37 599.55 205.21 394.34 40647.03
278 40647.03 599.55 203.24 396.31 40250.71
279 40250.71 599.55 201.25 398.30 39852.41
280 39852.41 599.55 199.26 400.29 39452.13
281 39452.13 599.55 197.26 402.29 39049.84
282 39049.84 599.55 195.25 404.30 38645.54
283 38645.54 599.55 193.23 406.32 38239.21
284 38239.21 599.55 191.20 408.35 37830.86

290
285 37830.86 599.55 189.15 410.40 37420.46
286 37420.46 599.55 187.10 412.45 37008.02
287 37008.02 599.55 185.04 414.51 36593.51
288 36593.51 599.55 182.97 416.58 36176.92
289 36176.92 599.55 180.88 418.67 35758.26
290 35758.26 599.55 178.79 420.76 35337.50
291 35337.50 599.55 176.69 422.86 34914.64
292 34914.64 599.55 174.57 424.98 34489.66
293 34489.66 599.55 172.45 427.10 34062.56
294 34062.56 599.55 170.31 429.24 33633.32
295 33633.32 599.55 168.17 431.38 33201.94
296 33201.94 599.55 166.01 433.54 32768.40
297 32768.40 599.55 163.84 435.71 32332.69
298 32332.69 599.55 161.66 437.89 31894.80
299 31894.80 599.55 159.47 440.08 31454.73
300 31454.73 599.55 157.27 442.28 31012.45
301 31012.45 599.55 155.06 444.49 30567.96
302 30567.96 599.55 152.84 446.71 30121.25
303 30121.25 599.55 150.61 448.94 29672.31
304 29672.31 599.55 148.36 451.19 29221.12
305 29221.12 599.55 146.11 453.44 28767.68
306 28767.68 599.55 143.84 455.71 28311.96
307 28311.96 599.55 141.56 457.99 27853.97
308 27853.97 599.55 139.27 460.28 27393.69
309 27393.69 599.55 136.97 462.58 26931.11
310 26931.11 599.55 134.66 464.89 26466.22
311 26466.22 599.55 132.33 467.22 25999.00
312 25999.00 599.55 129.99 469.56 25529.44
313 25529.44 599.55 127.65 471.90 25057.54
314 25057.54 599.55 125.29 474.26 24583.28
315 24583.28 599.55 122.92 476.63 24106.65
316 24106.65 599.55 120.53 479.02 23627.63
317 23627.63 599.55 118.14 481.41 23146.22
318 23146.22 599.55 115.73 483.82 22662.40
319 22662.40 599.55 113.31 486.24 22176.16
320 22176.16 599.55 110.88 488.67 21687.49
321 21687.49 599.55 108.44 491.11 21196.38
322 21196.38 599.55 105.98 493.57 20702.81
323 20702.81 599.55 103.51 496.04 20206.77
324 20206.77 599.55 101.03 498.52 19708.26
325 19708.26 599.55 98.54 501.01 19207.25
326 19207.25 599.55 96.04 503.51 18703.74
327 18703.74 599.55 93.52 506.03 18197.70
328 18197.70 599.55 90.99 508.56 17689.14
329 17689.14 599.55 88.45 511.10 17178.04
330 17178.04 599.55 85.89 513.66 16664.38
331 16664.38 599.55 83.32 516.23 16148.15
332 16148.15 599.55 80.74 518.81 15629.34

291
333 15629.34 599.55 78.15 521.40 15107.94
334 15107.94 599.55 75.54 524.01 14583.93
335 14583.93 599.55 72.92 526.63 14057.30
336 14057.30 599.55 70.29 529.26 13528.03
337 13528.03 599.55 67.64 531.91 12996.12
338 12996.12 599.55 64.98 534.57 12461.56
339 12461.56 599.55 62.31 537.24 11924.31
340 11924.31 599.55 59.62 539.93 11384.38
341 11384.38 599.55 56.92 542.63 10841.76
342 10841.76 599.55 54.21 545.34 10296.42
343 10296.42 599.55 51.48 548.07 9748.35
344 9748.35 599.55 48.74 550.81 9197.54
345 9197.54 599.55 45.99 553.56 8643.98
346 8643.98 599.55 43.22 556.33 8087.65
347 8087.65 599.55 40.44 559.11 7528.53
348 7528.53 599.55 37.64 561.91 6966.63
349 6966.63 599.55 34.83 564.72 6401.91
350 6401.91 599.55 32.01 567.54 5834.37
351 5834.37 599.55 29.17 570.38 5263.99
352 5263.99 599.55 26.32 573.23 4690.76
353 4690.76 599.55 23.45 576.10 4114.67
354 4114.67 599.55 20.57 578.98 3535.69
355 3535.69 599.55 17.68 581.87 2953.82
356 2953.82 599.55 14.77 584.78 2369.04
357 2369.04 599.55 11.85 587.70 1781.33
358 1781.33 599.55 8.91 590.64 1190.69
359 1190.69 599.55 5.95 593.60 597.09
360 597.09 599.55 2.99 596.56 0.53

292
Now suppose that instead of making the $599.55 monthly
payments, we make a $699.55 monthly payment. How much
sooner will we pay off the loan?

PMT = 699.55
PV = -100,000
I = 0.5
CPT N = ?

N = 251.50 months or 20.96 years to pay off the loan

Thus, by increasing our payments by $100 per month, we reduce


the number of monthly payments that we must make by:

360 – 251.50 = 108.5 months or 9.04 years

293
More Time Lines
You have just entered the following information into your
calculator. The cash flows you are entering into the calculator
occur, as shown on the following time line. (Note, your calculator
is set in the beg mode)

PMT = $1,000
N=5
I = 10
FV = ?

From this calculation you get an answer of $6,715.61. You then


enter the following information into your calculator.

PV = $6,715.61
I = 10
N=2
FV = ?

You get the answer $8,125.89

$8,125.89 represents the value of these payments at which point on


the following time line?

0 1 2 3 4 5 6 7 8
|---------|---------|---------|---------|---------|---------|---------|-------|
1,000 1,000 1,000 1,000 1,000

The answer is time Point 7

294
Perpetuities
Characteristics of a Perpetuity

1. There is a Series of Cash Flows


2. The number of cash flows is unlimited
3. There is an equal time spacing between each cash flow
4. The dollar amount of each cash flow is equal

Stream of Cash Yes Limited Number Yes Equal Time Yes Equal Dollar Yes
Annuity
Flows? of Cash Flows? Spacing? Amount?

No No No No

Uneven Cash Uneven Cash


Single Sum
Flow Stream Flow Stream

Equal Time Yes Equal Dollar Yes


Perpetuity
Spacing? Amount?

No No

Constant Yes Growing


Percentage
Perpetuity
Growth?

No

Combination Combination
Problem Problem

295
Example Time Line for a Perpetuity

0 1 2 3 4 5 ∞

|----------|----------|----------|----------|-----------|……----
100 100 100 100 100

Investments designed as Perpetuities include:

1. Shares of Preferred Stock


2. British Bonds

The Value of a perpetuity can be computed at two different time


points.

1. The time of the first cash flow PVPD

2. One time period before the first cash flow PVOP

PVPD = Present Value of a Perpetuity Due


PVOP = Present Value of an Ordinary Perpetuity

296
Example 1: Suppose you are considering making an investment
that promises to pay you $1,000 per year into infinity. You
will receive the first payment one year from today. You
require a 12 percent return on this type of investment. What
should you be willing to pay for this investment today?

0 1 2 3 4 5 ∞

|----------|----------|----------|----------|-----------|……----
1000 1000 1000 1000 1000
?

To solve this problem, we can compute the present value of


an ordinary perpetuity.

PMT 1 1000
PVOP = = = $8,333.33
kreq .12

297
Example 2: Suppose you are considering making an investment
that promises to pay you $1,000 per year into infinity. You
will receive the first payment 5 minutes after you purchase
the investment. You require a 12 percent return on this type
of investment. What should you be willing to pay for this
investment today?

0 1 2 3 4 5 ∞

|----------|----------|----------|----------|-----------|……----
1000 1000 1000 1000 1000 1000
?

To solve this problem, we can compute the present value of a


perpetuity due as.

PVPD = PMT 0 + PMT 0 = 1000 + 1000 = $9,333.33


kreq .12

298
Growing Perpetuity
Characteristics of a Growing Perpetuity

1. There is a series of cash flows


2. The number of cash flows is unlimited
3. There is an equal time spacing between each cash flow
4. Each cash flow is not for an equal amount. Rather, the
cash flows become larger by some constant percentage
amount in each subsequent year.

Stream of Cash Yes Limited Number Yes Equal Time Yes Equal Dollar Yes
Annuity
Flows? of Cash Flows? Spacing? Amount?

No No No No

Uneven Cash Uneven Cash


Single Sum
Flow Stream Flow Stream

Equal Time Yes Equal Dollar Yes


Perpetuity
Spacing? Amount?

No No

Constant Yes Growing


Percentage
Perpetuity
Growth?

No

Combination Combination
Problem Problem

299
Example Time Line for a Growing Perpetuity

0 1 2 3 4 5 ∞

|----------|----------|----------|----------|-----------|……----
100 110 121 133.10 146.41

The value of a growing perpetuity can be computed at two time


points:

1. The time of the first cash flow PVGPD

2. One time period before the first cash flow PVGOP

PVGPD = Present Value of a Growing Perpetuity Due


PVGOP = Present Value of a Growing Ordinary Perpetuity

300
Example: Suppose you expect to receive a payment of $10 one
year from today from an investment. You will continue to receive
payments each year forever. The payments will become 5 percent
larger in each successive year. You require a 10 percent return on
this investment. What is the present value of this stream of cash
flows? (that is what is the value of the cash flows at time zero.

0 1 2 3 4 5 ∞

|----------|----------|----------|----------|-----------|……----
10 10.50 11.025 11.58 12.16

This problem can be solved using the growing ordinary perpetuity


technique: In this case we know PMT (1)

PMT 1 PMT − 0 (1 + g )
PVGOP = =
kreq − g kreq − g

PMT 1 = the payment we will receive one year from today.


PMT-0 = the payment received 5 minutes ago.
g = the constant percentage growth rate
Kreq = the required rate of return

Which form of the formula you use depends upon the information
you have available in the problem. PMT1 or PMT-0

PMT 1 10
PVGOP = = = $200
kreq − g .10 − .05

301
Growing Ordinary Perpetuities

Example

Suppose you just received a payment of $9.524 from an investment


5 minutes ago. You will continue to receive payments each year
forever. The payments will become 5 percent larger in each
successive year. You require a 10 percent return on this
investment. What is the present value of this stream of cash flows?

-0 0 1 2 3 4 5 ∞

|---|----------|----------|----------|----------|-----------|……----
$9.524 $10 10.50 11.025 11.58 12.16

This problem can be solved using the growing ordinary perpetuity


technique: In this case we know PMT (-0)

PMT 1 PMT − 0 (1 + g )
PVGOP = =
kreq − g kreq − g

PMT − 0 (1+ g ) 9.524(1.05)


PVGOP = = = $200
kreq − g .10 − .05

302
Example of Growing Perpetuity Due

Suppose you expect to receive a payment of $10 five minutes from


now from an investment. You will continue to receive payments
each year forever. The payments will become 5 percent larger in
each successive year. You require a 10 percent return on this
investment. What is the present value of this stream of cash flows?

This problem is a growing perpetuity due:

PMT 0 (1 + g )
PVGPD = + PMT 0
kreq − g

PMT0 = the payment you will receive 5 minutes from now.


g = the constant percentage growth rate
Kreq = the required rate of return

PMT 0 (1 + g ) 10(1.05)
PVGPD = + PMT 0 = + 10 = $220
kreq − g .10 − .05

303
Uneven Cash Flow Streams

Characteristics of an Uneven Cash Flow Stream

1. Series of Cash Flows


2. Limited Number of Cash Flows
3. There may or may not be an equal time spacing between
each of the cash flows.
4. Each Cash Flow may or may not be for an equal dollar
amount.

Stream of Cash Yes Limited Number Yes Equal Time Yes Equal Dollar Yes
Annuity
Flows? of Cash Flows? Spacing? Amount?

No No No No

Uneven Cash Uneven Cash


Single Sum
Flow Stream Flow Stream

Equal Time Yes Equal Dollar Yes


Perpetuity
Spacing? Amount?

No No

Constant Yes Growing


Percentage
Perpetuity
Growth?

No

Combination Combination
Problem Problem

304
Example of a time line for an Uneven Cash Flow Stream

0 1 2 3 4 5 6 7

|----------|----------|----------|----------|----------|----------|----------|

100 200 0 200 0 800

305
Present Value of an Uneven Cash Flow Stream

We will receive the following cash flows from an investment.

Time Cash Flow


0 100
1 200
2 0
3 500
4 500

We require a 12 percent return on our investment. What is the


present value of these cash flows (i.e. what is the value at time 0).

0 1 2 3 4

|----------|----------|----------|----------|

100 200 0 500 500

We can solve for the present value of an uneven cash flow stream
using the Cash Flow Register in our calculator as follows:

CF
CF0 = 100 ↵ ↓
C01 = 200 ↵ ↓↓
C02 = 0 ↵ ↓↓
C03 = 500 ↵ ↓↓
C04 = 500 ↵ ↓↓

NPV
I = 12 ↵ ↓
CPT NPV = ? NPV = $952.22

306
Future Value of an Uneven Cash Flow Stream

Suppose we will deposit the following cash flows in the bank into
an account that pays 12 percent nominal interest compounded
annually. How much money will we have in the bank four years
from today?

0 1 2 3 4

|----------|----------|----------|----------|

100 200 0 500 500

Unfortunately, our calculator can not calculate the future value of


an uneven cash flow stream directly, so we must follow a two step
process:

Step 1: Compute the Present Value of an uneven cash flow stream.

We completed this step in the previous problem receiving an


answer of $952.22

Step 2: Compute the Future Value of a Single Sum

PV = 952.22
I = 12
N=4
CPT FV = ? = $1,498.34

307
Example of an Uneven Cash Flow Problem

Suppose you deposit $5,000 per year into an account at the end of
each of the next 3 years, and $6,000 into the account each year for
the following 7 years. How much money will you have in the
account at the end of the 10th year? Assume that we earn 10
percent nominal interest compounded annually on our investments.

0 1 2 3 4 5 6 7 8 9 10
|-------|-------|-------|-------|-------|------|------|------|------|------|
$5K $5K $5K $6K $6K $6K $6K $6K $6K $6K
?

We are interested in the amount of money we will have at time 10.


Thus, we are interested in the future value of an uneven cash flow
stream:

Step 1: Compute the Present Value of an Uneven Cash Flow


Stream

CF
CF0 = 0 ↵ ↓
C01 = 5,000 ↵ ↓
F01 = 3 ↵ ↓
C02 = 6,000 ↵ ↓
F02 = 7 ↵

NPV
I = 10 ↵ ↓
NPV
CPT NPV = 34,380.55

This is the value of the payments at time 0

308
Step 2: Compute the Future Value of a Single Sum

PV = 34,380.55
I = 10
N = 10
CPT FV = ?

FV = $89,174.29

309
Solving for the Interest Rate in an Uneven Cash Flow Problem

Suppose we will make an investment with the following cash


flows. What interest rate are we earning on our initial $1,000
investment?

Time Cash Flow


0 -1000
1 500
2 700
3 400

In order to solve this problem, we enter the cash flows into the
calculator as before

CF0 = -1000 ↵ ↓
C01 = 500 ↵ ↓↓
C02 = 700 ↵ ↓↓
C03 = 400 ↵ ↓↓

IRR
CPT

IRR = 28.61%

Note that solving for the interest rate is only possible if there is
exactly one sign change in the cash flows. It is not possible if
there are no cash flow sign changes or if there is more than one
sign change like there is in the following example

CF0 = -1000 ↵ ↓
C01 = 500 ↵ ↓ ↓
C02 = 700 ↵ ↓ ↓
C03 = -400 ↵ ↓ ↓

310
Combination Problems
Sometimes we have an unusual problem that does not fit into any
of the above classifications. In this case we need to use more than
one technique to solve the problem.

Combination problems can come in a variety of forms with one


being:

1. Stream of Cash Flows


2. Unlimited Number of Cash Flows
3. Equal time spacing between each cash flow
4. Cash flows are not for an equal dollar amount
Moreover, the cash flows to not grow by some constant
percentage amount in each successive time period.

Stream of Cash Yes Limited Number Yes Equal Time Yes Equal Dollar Yes
Annuity
Flows? of Cash Flows? Spacing? Amount?

No No No No

Uneven Cash Uneven Cash


Single Sum
Flow Stream Flow Stream

Equal Time Yes Equal Dollar Yes


Perpetuity
Spacing? Amount?

No No

Constant Yes Growing


Percentage
Perpetuity
Growth?

No

Combination Combination
Problem Problem

311
Example of a Combination Problem

Suppose you will receive $100 from an investment at the end of


each of the next three years. After that you will continue to receive
payments each year forever, but the payments will become 10
percent larger in each successive year. You require a 12 percent
return on this type of investment. What should you be willing to
pay to buy this investment today? (that is, What is the present
value of these cash flows at time 0?)

Step 1: Draw a time line

0 1 2 3 4 5 6 7 8 ∞

|--------|--------|--------|--------|--------|--------|--------|--------|-----|-
$100 $100 $100 $110 $121 $133.1 ………
?

There is a series of cash flows


There is an unlimited number of cash flows.
There is an equal time spacing between each of the cash flows
Each cash flow is not for an equal dollar amount
There is not a constant percentage growth rate

Thus, we have a combination problem.

312
Step 2: Examine the cash flows in two parts. The first part is
after time period three. The cash flows after time period three
are a growing ordinary perpetuity.

Recall that the formula for a growing perpetuity is:

PMT 1 PMT − 0 (1 + g )
PVGOP0 = =
kreq − g kreq − g

In this case we will use the first form of the formula

PMT 1
PVGOP0 =
kreq − g

However, it is necessary to modify the formula for our needs

PMT 4
PVGOP3 =
kreq − g

So we have:

110
PVGOP3 = = $5,500
.12 − .10

313
Step 3: Redraw the Time Line to reflect our computations

0 1 2 3 4 5 6 7 8 9
|--------|--------|--------|--------|--------|--------|--------|--------|--------|
$100 $100 $100
$5,500
$5,600

0 1 2 3 4 5 6 7 8 9
|--------|--------|--------|--------|--------|--------|--------|--------|--------|
$100 $100 $5,600
?

Step 4: The above cash flows are an uneven cash flow stream. So
we can compute the present value of an uneven cash flow
stream.

C01 = 100
C02 = 100
C03 = 5600

NPV
I = 12
NPV = ? = $4,154.97

314
Notes on Using Microsoft Excel to complete time value of
money computations.

You can use the function fx on the Excel menus to access the
function list. From here, you can select, FV, PV, PMT, IRR or
NPV, which will bring up a dialog box for you. You enter the data
and the computer will output the appropriate answer.

Note: Unlike your calculator, Excel requires you to enter the


interest rate in decimal form.

You must use caution when computing NPV using Excel.

Suppose you have the following cash flows and an interest


rate of 12 percent.

Time CF
0 -100
1 50
2 150
3 200

The Intuitive way to solve the problem:

In the NPV dialog box, you will enter cash flows 0-3 for
"Value 1", and 0.12 for the rate.

Select OK for excel to complete the computations. The


answer is 184.44. This answer is not correct.

315
The correct way to solve the problem:

In the NPV dialog box, you will enter cash flows 1-3 for "Value
1", and 0.12 for the rate. DO NOT INCLUDE THE VALUE FOR
TIME 0 HERE.

Select OK for excel to complete the computations. The answer is


206.58. This answer is not correct.

Excel creates a formula that looks like this:

=NPV(D4,D7:D9)

Where "D4" = 0.12 and "D7:D9" is the series of cash flows, 50,
100, 200

You must edit the formula Excel produced (by subtracting the time
zero cash flow) as follows to get the correct answer.

=NPV(D4, D7:D9)-$100 = 206.57

which is the correct answer.

316
Chapter 8 Homework and Solutions

317
Chapter 8 Homework

1. Which buttons do you need to push on your calculator to set your calculator for use in this class?

2. Which buttons do you need to push on your calculator to clear

A. What is showing on your screen


B. The time value of money section of your calculator
C. The Cash Flow section of your calculator

3. You will deposit $1,000 today into an account. Calculate how much money you will have in the
account under each of the following conditions.

A. You earn 6 percent nominal interest compounded annually. You leave the money in the
account for one year.
B. You earn 6 percent nominal interest compounded annually. You leave the money in the
account for two years.
C. You earn 6 percent nominal interest compounded annually. You leave the money in the
account for thirty years.
D. You earn 6 percent nominal interest compounded annually. You leave the money in the
account for forty years.
E. Do the above answers tell you anything about the plans you should make for retirement?

4. Now suppose that rather than earning six percent interest, you can earn ten percent nominal
interest compounded annually. Rework the above problem using this information.

5. Now suppose that you can earn 10 percent nominal interest compounded semi-annually on your
investments. Rework parts A-D of Problem 3 using this information.

6. Your grandmother has just purchased an investment as a gift for you (The investment is a
government bond). The investment is set up so that you will not get any money today, however;
you will get a lump sum amount at some time in the future. You want to know what the
investment is worth to you today under each of the following conditions.

A. You will receive $1,000,000 from the investment 1 year from today (you have got to love
that!!!!). The appropriate interest rate is 6 percent nominal compounded annually.
B. You will receive $1,000,000 from the investment two years from today. The appropriate
interest rate is 6 percent nominal compounded annually.
C. You will receive $1,000,000 from the investment 30 years from today. The appropriate
interest rate is 6 percent nominal compounded annually.
D. You will receive $1,000,000 from the investment 40 years from today . The appropriate
interest rate is 6 percent nominal compounded annually.
E. When would you like to receive the money and why?

7. Now suppose the appropriate interest rate is 10 percent nominal compounded annually rather than
6 percent in the above problem.

I. Rework parts A-D of the above problem using the new information.

II. What do these results tell us about how the discount rate affects the present value?

318
8. Now suppose the appropriate interest rate is 10 percent nominal compounded monthly in the
above problem.

I. Rework parts A-D of the above problem using the new information.

II. What does this problem tell us about compounding periods and present values.

9. You go to the First Hawaiian Bank. They tell you that you can place your money into one of two
accounts. The accounts are identical except for the interest rate that they pay. Compare each of
the following pairs of accounts to determine which of the two accounts is best for you.

A. 10 percent nominal compounded annually or 10 percent nominal compounded monthly.


B. 9.5 percent nominal compounded monthly or 10 percent nominal compounded semi-annually.
C. 10 percent nominal compounded daily (365 day year) or 10.5 percent nominal compounded
quarterly.
D. 10 percent nominal compounded quarterly or 10.2 percent nominal compounded monthly.
E. 7 percent nominal compounded weekly or 7.5 percent nominal compounded semi-annually.

10. You go to the bank today to deposit $10,000 into an account that pays 5 percent nominal interest,
compounded continuously. How much money will you have in the bank ten years from today if
you do not take any money out of the account?

11. You wish to have $2,000,000 available for retirement 30 years from today. The bank tells you
they will pay you 9 percent nominal interest, compounded continuously on the account. How
much do you need to deposit today to achieve your goal?

12. What are the four conditions that must be met in order for a series of cash flows to be an annuity?

13. Draw a time line that illustrates the point in time when we calculate the value of an annuity if we
calculate the present value of an ordinary annuity, the present value of an annuity due, the future
value of an ordinary annuity, and the future value of an annuity due.

14. THIS HOMEWORK QUESTION HAS BEEN REMOVED. A NEW QUESTION WILL BE
ADDED TO REPLCE IT NEXT SEMESTER.

15. THIS HOMEWORK QUESTION HAS BEEN REMOVED. A NEW QUESTION WILL BE
ADDED TO REPLCE IT NEXT SEMESTER.

16. You have just won the largest Power Ball lottery of all time. You won a $150,000,000 lottery
Congratulations!!!!. Of course, you have decided that your education is important and you will
continue your studies…. Particularly your finance studies so you know how to invest all of that
money wisely. The lottery is paid in 20 equal annual installments of $7,500,000 each. You will
receive the first payment as soon as the armored car that you have rented gets you and the winning
lottery ticket to the lottery office. You are a little bit greedy though, and decide that you would
like to get your hands on as much money as you possibly can today. So you stop at the bank and
tell them that you will give them all of the payments for a single lump sum today. The bank says
they will buy the payments from you. The bank says they will use a 7.5 percent nominal annually
compounded interest to calculate the present value of the payments, which is what you will
receive. How much will the bank give you today?

17. You are not entirely satisfied with the amount that the bank has indicated they will give you in the

319
previous problem. Just before you sign the papers, you change your mind and decide not to sell
the payments to the bank. The bank, eager to get your business says that they will give you
$87,600,000 today instead of the amount you calculated in the previous problem. What interest
rate is the bank using to calculate the present value of the payments now?

18. Calculate the present value (that is the value at time 0) of the following:

A. An investment that will give you $1,000 at the end of each of the next 10 years. The
appropriate interest rate is 10 percent nominal compounded annually.

B. An investment that will give you $1,000 at the end of each of the next 10 years. The
appropriate interest rate is 20 percent nominal compounded annually.

C. An investment that will give you $1,000 at the end of each of the next 20 years. The
appropriate interest rate is 10 percent nominal compounded annually.

D. An investment that will give you $1,100 at the end of each of the next 10 years. The
appropriate interest rate is 10 percent nominal compounded annually.

19. Now suppose that you will receive the payments in the above problem at the beginning of each
year rather than at the end of each year. Re-compute the answers for the above problem using this
new information.

20. Calculate the Future Value of the following:

A. You deposit $1,000 per year at the end of each of the next 20 years into an account that
pays 10 percent nominal interest compounded annually. How much will you have in the
account at the end of the 20th year?
B. You deposit $2,000 per year at the end of each of the next 20 years into an account that
pays 10 percent nominal interest compounded annually. How much will you have in the
account the end of the 20th year?
C. You deposit $1,000 per year at the end of each of the next 10 years into an account that
pays 10 percent nominal interest compounded annually. How much will you have in the
account at the end of the 10th year?
D. You deposit $1,000 per year at the end of each of the next 20 years into an account that
pays 12 percent nominal interest compounded annually. How much will you have in the
account at the end of the 20th year?

21. Calculate the Future Value of the following:

A. You deposit $1,000 per year at the beginning of each of the next 20 years into an account
that pays 10 percent nominal interest compounded annually. How much will you have in
the account at the end of the 20th year?
B. You deposit $2,000 per year at the beginning of each of the next 20 years into an account
that pays 10 percent nominal interest compounded annually. How much will you have in
the account the end of the 20th year?
C. You deposit $1,000 per year at the beginning of each of the next 10 years into an account
that pays 10 percent nominal interest compounded annually. How much will you have in
the account at the end of the 10th year?
D. You deposit $1,000 per year at the beginning of each of the next 20 years into an account
that pays 12 percent nominal interest compounded annually. How much will you have in
the account at the end of the 20th year?

320
22. Suppose I deposit $10,000 into the bank at the end of each of the next 6 years. The account will
pay 10 percent nominal interest compounded annually. How much money will I have in the bank
7 years from today?

23. You are considering buying a car. The car dealer says the price of your dream machine is
$20,000. You will borrow all $20,000. You plan to make monthly payments on the car for the
next 5 years. The first payment will be made today. The interest rate on this loan is 6.99 percent
nominal compounded monthly.

A. How much will the payments be?

B. You are also considering a lower priced car. How much will your payments be if you
borrow $14,000 instead of $20,000. All other features of the loan will be the same.

C. You decide you really don’t want to buy a car after all. Instead, you will buy a house. In
order to buy the house you will borrow $100,000. The loan will be for 30 years. You
will make monthly payments with the first payment due one month after you buy the
house. The interest rate on the loan is 6.75 percent compounded monthly. How much
will the payments on the house be?

24. You recently purchased a new home. You paid $150,000 today for the home and financed the entire
purchase at an 8 percent nominal interest rate compounded monthly. You are scheduled to pay off
the loan by making monthly payments for the next thirty years. The first payment will be due one
month from today. As such, your monthly payment will be $1,100.65. You decide however, that
instead of making the required $1100.65 payment each month, you will increase the amount of the
payment you will make. How long will it take you to pay off the loan under each of the following
conditions.

A. Your make a $1,200 payment each month?


B. You make a $1,500 payment each month?
C. You make a $2,202 payment each month?
D. You make a $5,000 payment each month

25. What conditions must be met for a series of cash flows to be a perpetuity?

26. A. You are considering making an investment in a stock that will give you $100 each year
forever. You will receive the first payment one year from today. You have determined
that you require a 10 percent nominal return on this type of investment. What is the
present value of these cash flows?

B. You have re-evaluated the above decision and determined that you require a 12 percent
nominal return on this investment. What is the present value of these cash flows now?

C. You are looking into another investment that will give you $500 each year forever. You
will receive the first payment one year from today. You require a 25 percent nominal
return on this type of investment. What is the present value of these cash flows?

27. Rework the above problem assuming that you will get the first payment in 5 minutes rather than
one year from today.

28. What are the conditions necessary for a series of cash flows to be a growing perpetuity?

321
29.
A. Suppose you are considering investing in a stock. The stock will give you a $100
payment one year from today. The stock will continue to give you one payment per year
forever. However, each year the payment will become 10 percent larger. You require a
15 percent rate of return on this investment. What is the present value of the cash flows
you will receive from this stock?

B. Suppose you re-examine the above cash flows and determine that you will get $110 one
year from today rather than $100. The required rate of return is still 15 percent and the
payments will become larger by 10 percent per year. What is the present value of the
cash flows you will receive from this stock?

C. Suppose you re-examine the above problem and determine that you will get $110 one
year from today. However, the required rate of return should be 20 percent instead of 15
percent and the growth rate should be 10 percent. What is the present value of the cash
flows you will receive from this stock?

D. Suppose yet again you re-examine the above problem and determine that you will get
$110 one year from today. The required rate of return is 20 percent, but the growth rate
will only be 5 percent. What is the present value of the cash flows you will receive from
this stock?

30. That **0(///^^5%%% finance professor. He gave you the wrong information for the above
problem. The payments that are given (100 and 110 in the above problem) are the payments that
were just received 5 minutes ago, not the payments you will receive one year from today. Re-
compute the answers to parts A-D using the correct information.

31. You will receive $100 from an investment 5 minutes after you purchase it. You will continue to
receive payments each year forever, however the payments will become 7 percent larger in each
successive year. You require a 14 percent return on this type of investment. What should you be
willing to pay to buy this investment today?

32. What are the conditions for a series of cash flows to be an uneven cash flow stream?

33. What is the present value (at time 0) of the following cash flow stream?

Time Cash Flow


0 ($1000)
1 $2000
2 ($500)
3 $4000
4 ($100)

A. The appropriate interest rate is 12 percent nominal compounded annually.

B. The appropriate interest rate is 14 percent nominal compounded annually.

C. The appropriate interest rate is 16 percent nominal compounded annually.

D. The appropriate interest rate is 18 percent nominal compounded annually.

322
34. Using the information from the above problem. Compute the future value of the cash flows at the
end of the fifth year for parts A-D.

35. How long will it take for your money to double at each of the following interest rates?

A. 10 percent nominal compounded annually.


B. 12 percent nominal compounded annually.
C. 14 percent nominal compounded annually.
D. 16 percent nominal compounded annually.

36. You are considering buying a house. In order to buy the house you will borrow $100,000 today at
a nominal interest rate of 6.75 percent, compounded monthly. You will make monthly payments
with the first payment due one month after you buy the house.

A. Compute the payments on the loan if the loan is for 30 years.


B. Compute the payments on the loan if the loan is for 15 years.
C. Create an amortization table for the first three months of the loan in part A.
D. Create an amortization table for the first three months of the loan in part B.

37. You will put $1000 in the bank at the end of each of the next 4 years. You are saving the money
to go to graduate school. You have consulted your finance professor who says the type of degree
you are interested in will take two years to earn. You will start graduate school 4 years from
today. You want to take two equal payments out of the account, one at the beginning of each year
of graduate school. You will take the first payment out on the day that you start graduate school
and the second payment out one year later. You can earn 10 percent interest compounded
annually on your investments. How much will you be able to take out of the account each year to
pay for graduate school?

38. We will deposit $500 in to an account at the end of each of the next 5 years. We will stop making
deposits at the end of the 5th year, however, we will not take any money out of the account. How much
money will we have in the account at the end of the 10th year. We will earn 10 percent interest on the
account. Show how to solve the problem in two ways. The first way will be to use an ordinary
annuity, and the second way will be to use an annuity due.

39. Your son Bob is 12 years old today. You are planning for his college education. Bob will start school
on his 19th birthday. You wish to set aside some money early to send Bob to four years of school.
You have decided that you will give Bob 16,000 per year for each of his first two years of college, and
20,000 per year for each of his last two years of college. You will give these amounts to Bob at the
beginning of each school year.

You will make 7 equal annual deposits to fund the account. The first payment will be made one year
from today and the last payment will be made the day Bob leaves for college. You wish to have just
enough money in the bank to fund Bobs entire education on the day that he leaves for school. Any
money that is in the account will continue to earn interest while Bob is in school. Because of a new
program, the bank has agreed to give you a 10 percent, nominal compounded annually, return on your
investments throughout the entire time period. How much do you need to deposit into the account in
each of the 7 years in order to fund Bobs education?

40. Marvin Urlacher is 25 years old today. Urlacher wants to be prepared for his retirement, and so is
doing his planning early. He has already saved $10,000 toward retirement. Urlacher plans to

323
save $4,000 per year at the end of each of the next 15 years and $5,000 per year at the end of each
of the following 15 years, and 6,000 per year for the following 5 years. Urlacher plans to retire on
his 60th birthday. On the day he retires, he plans to spend $200,000 of the money he has
accumulated to purchase a house. Urlacher will live 25 years beyond retirement. He wishes to
have a contingency fund of $100,000 available (in addition to his home) at the time of his death to
pay for funeral and other expenses that may arise in relation to his death. How much money can
Urlacher withdraw at the beginning of each year of retirement and still meet his other goals? Each
of the withdrawals will be for an equal amount. Assume a 10 percent nominal interest rate
compounded annually throughout the entire problem. Make sure to draw a time line to show when
the payments are being made and received.

41.

Suppose you will receive $200 from an investment at the end of each of the next four years. After that you
will continue to receive payments each year forever, but the payments will become 10 percent larger in
each successive year. You require a 15 percent return on this type of investment. What should you be
willing to pay to buy this investment today? (that is, What is the present value of these cash flows at time
0?)

42. Using the following financial statements:


A. calculate the compound annual sales growth from 2014-2016.
B. Calculate the Net Income Growth from 2014-2016.

324
MICROSOFT CORP
BALANCE_SHEET
Period End: Jun 30, 2016
(In Millions)
2016 2015
Assets
Current assets:
Cash and cash equivalents $ 6,510 $ 5,595
Short-term investments (including securities loaned of $204 and $75) 106,730 90,931
------------------------------------------------------------------------------------------------------------------------------- -----------------
Total cash, cash equivalents, and short-term investments 113,240 96,526

Accounts receivable, net of allowance for doubtful accounts 18,277 17,908


Inventories 2,251 2,902
Other 5,892 5,461

Total current assets 139,660 122,797


Property and equip., net of accum. depreciation of $19,800 and $17,606) 18,356 14,731
Equity and other investments 10,431 12,053
Goodwill 17,872 16,939
Intangible assets, net 3,733 4,835
Other long-term assets 3,642 3,117

Total assets $ 193,694 $ 174,472

Liabilities and stockholders' equity


Current liabilities:
Accounts payable $ 6,898 $ 6,591
Short-term debt 12,904 4,985
Current portion of long-term debt 0 2,499
Accrued compensation 5,264 5,096
Income taxes 580 606
Short-term unearned revenue 27,468 23,223
Securities lending payable 294 92
Other 5,949 6,555
------------------------------------------------------------------------------------------------------------------------------- -----------------
Total current liabilities 59,357 49,647
Long-term debt 40,783 27,808
Long-term unearned revenue 6,441 2,095
Deferred income taxes 1,476 1,295
Other long-term liabilities 13,640 13,544

Total liabilities 121,697 94,389

Stockholders' equity:
Common stock and paid-in capital - shares authorized 24,000;
outstanding 7,808 and 8,027 68,178 68,465
Retained earnings 2,282 9,096
Accumulated other comprehensive income 1,537 2,522

Total stockholders' equity 71,997 80,083

Total liabilities and stockholders' equity $ 193,694 $ 174,472

325
MICROSOFT CORP
INCOME_STATEMENT
Period End: Jun 30, 2016

(In millions, except per share amounts)


----------------------------------------------------------------------------------------------------------------------------

Year Ended June 30, 2016 2015 2014

Revenue:
Product $ 61,502 $ 75,956 $ 72,948
Service and other 23,818 17,624 13,885
------------------------------------------------------------------------------------------ ---------------- ----------------
Total revenue 85,320 93,580 86,833
------------------------------------------------------------------------------------------ ---------------- ----------------
Cost of revenue:
Product 17,880 21,410 16,681
Service and other 14,900 11,628 10,397
------------------------------------------------------------------------------------------ ---------------- ----------------
Total cost of revenue 32,780 33,038 27,078
------------------------------------------------------------------------------------------ ---------------- ----------------
Gross margin 52,540 60,542 59,755
Research and development 11,988 12,046 11,381
Sales and marketing 14,697 15,713 15,811
General and administrative 4,563 4,611 4,677
Impairment, integration, and restructuring 1,110 10,011 127
------------------------------------------------------------------------------------------ ---------------- ----------------
Operating income 20,182 18,161 27,759
Other income (expense), net (431) 346 61
------------------------------------------------------------------------------------------ ---------------- ----------------
Income before income taxes 19,751 18,507 27,820
Provision for income taxes 2,953 6,314 5,746
------------------------------------------------------------------------------------------ ---------------- ----------------
Net income $ 16,798 $ 12,193 $ 22,074
----------------- ---------------- ----------------

Earnings per share:


Basic $ 2.12 $ 1.49 $ 2.66
Diluted $ 2.10 $ 1.48 $ 2.63

Weighted average shares outstanding:


Basic 7,925 8,177 8,299
Diluted 8,013 8,254 8,399

Cash dividends declared per common share $ 1.44 $ 1.24 $ 1.12


----------------------------------------------------------------------------------------------------------------------------

Solutions to Chapter 8 Homework

326
1. To set the number of decimal places press 2nd Format 9 Enter
To set the number of payments per year 2nd P/Y 1 Enter

2. A. CE/C
B. 2nd CLR TVM
C. CF, 2nd CLR Work

3. A. PV = 1,000
N=1
I=6
FV = ? = $1,060

B. PV = 1,000
N=2
I=6
FV = ? = $1,123.60

C. PV = 1,000
N = 30
I=6
FV = ? = $5,743.49

D. PV = 1,000
N = 40
I=6
FV = ? = $10,285.72

E. Yes, it tells us that it is important to begin planning for retirement early.

4.

A. PV = 1,000
N=1
I = 10
FV = ? = $1,100

B. PV = 1,000
N=2
I = 10
FV = ? = $1,210

C. PV = 1,000
N = 30
I = 10
FV = ? = $17,449.40

D. PV = 1,000
N = 40
I = 10
FV = ? = $45,259.26

E. Yes, not only is it important to begin planning for retirement early, it is important to
monitor carefully the return we are getting on our investments.

5. A. PV = 1,000

327
N = 2 (2X1)
I = 5 (10/2)
FV = ? = $1,102.50

B. PV = 1,000
N = 4 (2X2)
I = 5 (10/2)
FV = ? = $1,215.51

C. PV = 1,000
N = 60 (30X2)
I = 5 (10/2)
FV = ? = $18,679.19

D. PV = 1,000
N = 80
I = 5 (10/2)
FV = ? = $49,561.44

6. A. FV = 1,000,000
N=1
I=6
PV = ? = $943,396.23

B. FV = 1,000,000
N=2
I=6
PV = ? = $889,996.44

C. FV = 1,000,000
N = 30
I=6
PV = ? = $174,110.13

D. FV = 1,000,000
N = 40
I=6
PV = ? = $97,222.19

E. You would like the money as soon as possible because money received today is worth more
than money received in the future.

7.

I.
A. FV = 1,000,000
N=1
I = 10
PV = ? = $909,090.91

B. FV = 1,000,000
N=2
I = 10
PV = ? = $826,446.28

C. FV = 1,000,000

328
N = 30
I = 10
PV = ? = $57,308.55

D. FV = 1,000,000
N = 40
I = 10
PV = ? = $22,094.93

II. The higher the discount rate is, the lower the present value will be.

8.
I.
A. FV = 1,000,000
N = 12
I = 0.8333333333
PV = ? = $905,212.43

B. FV = 1,000,000
N = 24
I = 0.8333333333
PV = ? = $819,409.54

C. FV = 1,000,000
N = 360
I = 0.833333333333
PV = ? = $50,409.83

D. FV = 1,000,000
N = 480
I = 0.833333333333
PV = ? = $18,621.74

II. The more frequently we compound the interest rate, the lower the present value will be.

9. You can solve this problem by using the interest rate conversion function in your calculator.

2nd Iconv

Nom = 10 ↵ ↓↓
C/Y = 12 ↵ ↓
EFF = CPT = 10.47

A. 10 percent nominal compounded annually = 10 percent compounded annually


10 percent nominal compounded monthly = 10.47 percent compounded annually.
So 10 percent nominal compounded monthly better.

B. 9.5 percent nominal compounded monthly = 9.925 percent compounded annually.


10 percent nominal compounded semi-annually = 10.25 compounded annually.
So 10 percent nominal compounded semi-annually is better.

C. 10 percent nominal compounded daily = 10.52 percent compounded annually.


10.5 percent nominal compounded quarterly = 10.92 percent compounded annually.
So, 10.5 percent nominal compounded quarterly is better.

329
D. 10 percent nominal compounded quarterly = 10.38 percent compounded annually.
10.2 percent nominal compounded monthly = 10.69 percent compounded annually.
So, 10.2 percent nominal compounded monthly is better.

E. 7 percent nominal compounded weekly = 7.25 percent compounded annually.


7.5 percent nominal compounded semi-annually = 7.64 percent compounded annually.
So 7.5 percent nominal compounded semi-annually is better.

10. 𝐹𝐹𝐹𝐹 = 𝑃𝑃𝑃𝑃𝑒𝑒 𝑖𝑖𝑖𝑖 = $10,000e0.05(10) = $16,487.21

11. 𝑃𝑃𝑃𝑃 = 𝐹𝐹𝐹𝐹𝑒𝑒 −𝑖𝑖𝑖𝑖 = $2,000,000𝑒𝑒 −0.09(30) = $134,411.03

12. A. There must be a series of cash flows


B. Each of the cash flows must be for an equal dollar amount
C. There must be an equal time spacing between each of the cash flows.
D. There must be a limited number of cash flows.

13.

0 1 2 3 4 5 6 7

|----------|----------|----------|----------|----------|----------|----------|

First Last
PMT PMT

PV PV FV FV
OA AD OA AD

14. THIS QUESTION HAS BEEN REMOVED AND WILL BE REPLACED NEXT SEMESTER

15. THIS QUESTION HAS BEEN REMOVED AND WILL BE REPLACED NEXT SEMESTER

16. Your calculator should be set in the beg mode

PMT = 7,500,000
N = 20
I = 7.5
PV = ? = $82,193,086.58

17. Your calculator should be set in the beg mode.

PMT = 7,500,000
N = 20
PV = -87,600,000
I = ? = 6.57 percent

18. Your calculator should be set in the end mode

A. PMT = 1,000
N = 10
I = 10

330
PV = ? = $6,144.57

B. PMT = 1,000
N = 10
I = 20
PV = ? = $4,192.47

C. PMT = 1,000
N = 20
I = 10
PV = ? = $8,513.56

D. PMT = 1,100
N = 10
I = 10 PV = ? = $6,759.02
19. Your calculator should be set in the beg mode

A. PMT = 1,000
N = 10
I = 10
PV = ? = $6,759.02

B. PMT = 1,000
N = 10
I = 20
PV = ? = $5,030.97

C. PMT = 1,000
N = 20
I = 10
PV = ? = $9,364.92

D. PMT = 1,100
N = 10
I = 10
PV = ? = $7,434.93

20. Your calculator should be set in the end mode.

A. PMT = 1,000
N = 20
I = 10
FV = ? = $57,275

B. PMT = 2,000
N = 20
I = 10
FV = ? = $114,550.00

C. PMT = 1,000
N = 10
I = 10
FV = ? = $15,937.42

D. PMT = 1,000

331
N = 20
I = 12
FV = ? = $72,052.44

21. Your calculator should be set in the beg mode.

A. PMT = 1,000
N = 20
I = 10
FV = ? = $63002.50

B. PMT = 2,000
N = 20
I = 10
FV = ? = $126,005.00

C. PMT = 1,000
N = 10
I = 10
FV = ? = $17,531.17

D. PMT = 1,000
N = 20
I = 12
FV = ? = $80,698.74

22. You want to know the value one time period after the last cash flow. Thus you compute the future
value of an annuity due (FVAD, BEG).

PMT = 10,000
N=6
I = 10
CPT FV = ?

FV = $84,871.71

23. Your calculator should be set in the beginning mode

A. PV = 20,000
N = 60 (12X5)
I = 0.5825 (6.99/12)
PMT = ? = $393.64

B. PV = 14000
N = 60 (12X5)
I = 0.5825 (6.99/12)
PMT = ? = $275.55

C. Your calculator should be set in the end mode

PV = 100,000
N = 360 (12X30)
I = 0.5625 (6.75/12)
PMT = ? = $648.60

332
24. Solving this problem is slightly tricky. In order to solve the problem, you must make sure that you
enter the information into your calculator with the correct signs. In this case, we have both a
present value and a future value (the future value in this case is in the form of payments). Just as
we did before, we need to enter either the present value or the future value with a negative sign.
Recall that the loan was originally set up to have 360 payments.

A. PV = -150,000
I = 8/12 = .666666667
PMT = 1,200
N = ? = 269.66 Payments

Note, that by increasing our payment by $99.35 per month, the number of payments we
need to make decreased by (360-269.66) 90.34 payments.

B. PV = -150,000
I = 8/12 = 0.666666667
PMT = 1,500
N = ? = 165.34 Payments

C. PV = -150,000
I = 8/12 = 0.666666667
PMT = 2,202
N = ? = 91.10 Payments

D. PV = -150000
I = 8/12 = 0.666666667
PMT = 5000
N = ? = 33.58 Payments

25 A. There must be a series of payments.


B. The payments must be equal
C. There must be an equal time spacing between each of the payments.
D. The payments must continue forever

26. A. PV = PMT1/I = 100/0.1 = $1,000

B. = 100/0.12 = $833.33

C. = 500/0.25 = $2,000

27. A. PV = PMT1/kreq + PMT = 100/0.1 + 100 = $1,100

B. = 100/0.12 + 100 = $933.33

C. = 500/0.25 + 500 = $2,500

28. A. There must be a series of payments.


B. There must be an equal time spacing between each of the payments.
C. The payments must grow by some constant percentage rate in each time period.
D. The payments must continue forever.

29. A. PVGOP = PMT1/(kreq-g) = 100/(0.15-0.10) = $2,000

333
B. = 110/(0.15-0.10) = $2,200

C. = 110/(0.20-0.10) = $1,100

D. = 110/(0.20-0.05) = $733.33

30. A. PVGOP = PMT-0(1+g)/(kreq-g) = 100(1+0.1)/(0.15-0.10) = $2,200

B. = 110(1+0.10)/(0.15-0.10) = $2,420

C. = 110(1+ 0.10)/(0.20-0.10) = $1,210

D. = 110(1+0.05)/(0.20-0.05) = $770.00

31. PVGPD = (PMT0(1+g)/(kreq-g))+PMT0

= (100(1+0.07)/(0.14-0.07))+100

= $1,628.57

32. A. There must be a series of payments.


B. The payments may or may not be for an equal amount
C. There may or may not be an equal time spacing between each of the payments
D. There must be a limited number of payments.

33. In order to solve this problem, you must first enter the cash flows into your cash flow register.
You do this by first pressing the CF button on your calculator. Then enter the cash flows as
follows.

CF0 1,000 +- Enter Arrow down


CF1 2,000 Enter Arrow down, Arrow down
CF2 500 +- Enter Arrow down, Arrow down
CF3 4,000 Enter Arrow down, Arrow down
CF4 100 +- Enter

Now press the NPV key on your calculator

Input the interest rate and press enter.


Press the arrow down key
Press compute.

A. PV = $3,170.69
B. PV = $3,010.33
C. PV = 2,859.96
D. PV = 2,718.77

34. Because you have calculate the present value of the cash flows in the previous problem, you must
simply compute the future value of a single sum.

A. PV = 3,170.69
N=5
I = 12
FV = ? = $5,587.84

B. PV = 3010.33

334
N=5
I = 14
FV = ? = $5,796.13

C. PV = 2,859.96
N=5
I = 16
FV = ? = $6,006.89

D. PV = 2,718.77
N=5
I = 18
FV = ? = $6,219.88

35. A. PV = -1
FV = 2
I = 10
N = ? = 7.27 years

B. PV = -1
FV = 2
I = 12
N = ? = 6.12 years

C. PV = -1
FV = 2
I = 14
N = ? = 5.29 years

D. PV = -1
FV = 2
I = 16
N = ? = 4.67 years

36. A. PV = 100,000
N = 360
I = 0.5625
PMT = ? = $648.60

B. PV = 100,000
N = 180
I = 0.5625
PMT = ? = $884.91

C.

Month Beginning Balance Payment Interest Principle End Balance


1 100,000 648.60 562.50 86.10 99,913.90
2 99,913.90 648.60 562.02 86.58 99,827.32
3 99,827.32 648.60 561.53 87.07 99,740.25

D.

335
Month Beginning Balance Payment Interest Principle End Balance
1 100,000 884.91 562.50 322.41 99,677.59
2 99,677.59 884.91 560.69 324.22 99,353.37
3 99,353.37 884.91 558.86 326.05 99,027.32

0 1 2 3 4 5 6 7
37. |---------|-------|-------|-------|-------|-------|-------|
1000 1000 1000 1000
PMT PMT

The first thing we need to do to solve this problem will be to calculate the future value of the payments.
We will take the first payment out of the account at time 4, so it will be convenient to know the value of the
$1,000 payments at time 4. In order to do this we want to calculate the future value of an ordinary annuity.

1,000 = pmt
4 =n
I = 10
FV = ? = $4,641

So $4,641 is the value of the $1,000 payments at point 4 on the time line. In order to calculate the value of
the payments that we can take out of the account at time 4 and time 5, we will want to compute the
payments of an annuity due. We will use an annuity due. Why?, because we know the present value at
time 4. Time 4 is also the point in time that we want to take out the first payment. So we know the present
value at the time of the first payment. And thus we must use an annuity due.

Beg mode

PV = $4641
N=2
I = 10
PMT = ???? = $2,431

0 1 2 3 4 5 6 7 8 9 10
38. |---------|-------|-------|-------|-------|-------|-------|-------|-------|--------|
500 500 500 500 500

Using an ordinary annuity:

First calculate the future value of an ordinary annuity

PMT = 500
N=5
I = 10
FV = ?? = 3,052.55

336
This is the value of the payments at time 5 on the time line. There are 5 more years until the end of the 10th
year. Thus we will calculate the future value of a single sum as follows.

PV = 3052.55
N=5
I = 10
FV = ?? = 4,916.16

Using an annuity due

First calculate the future value of an annuity due


Beg mode
PMT = 500
N=5
I = 10
FV = ?? = 3,357.81

This is the value of the payments at time 6 on the time line. There are 4 more years until the end of the 10th
year. Thus we will calculate the future value of a single sum as follows.

PV = 3,357.81
N=4
I = 10
FV = ?? = 4,916.16

Note you may get a slightly different answer for this problem because of rounding differences.

39.

12 13 14 15 16 17 18 19 20 21 22
|-----------|----------|----------|----------|----------|-----------|----------|----------|----------|-----------|

PMT PMT PMT PMT PMT PMT PMT


16,000 16,000 20,000 20,000

Step 1 is to calculate the present value of all of the money he will need to have when Bob turns 19
years old. You can do this by using the cash flow register in your calculator.

CF0 = 16,000
CF1 = 16,000
CF2 = 20,000
CF3 = 20,000

I = 10
NPV = $62,100.68

Step 2: Now we can calculate the amount of the payments that the father will have to deposit into the
account. We can use an ordinary annuity to do this. The reason that we use an ordinary annuity is
because we know the future value ($62,100.68) at the time that the father will make the last deposit
into the account.

FV = $62,100.68
N=7
I = 10
PMT = ? = $6,545.75

337
40.

25 26…….. 40 41…….. 55 56……. 60 61 84 85


|-----------|-----------|----------|----------|----------|----------|-------------|-----------|-----------|
10,000 4000… 4000 5000…..5000 6,000….6,000
-$200,000
PMT…. PMT -100,000

There are numerous potential ways to solve this problem. I will show you one method,
However, you may use any method that gives you the correct answer.

Step 1 Calculate the future value of the deposits Urlacher will make into the account. We can do this
by calculating the future value of an uneven cash flow stream. Recall that in order to
calculate the future value of an uneven cash flow stream, we must first calculate the present
value of an uneven cash flow stream then calculate the future value of a single sum.

Calculating the PV of an uneven cash flow stream.

CF0 = 10000
CF1 = 4000
F01 = 15
CF2 = 5000
F02 = 15
CF3= 6,000
F03=5
NPV
I = 10
NPV = ? = $50,831.96

Calculating the Future value of a single sum 35 years from today.

PV = $50,831.96
N = 35
I = 10
FV = $1,428,501.95

Step 2: Subtract the amount of money used to purchase the house

Money Saved $1,428,501.95


House Purchase $200,000
Available for Retirement needs $1,228,501.95

Step 3: Calculate the present value of the 100,000 contingency fund that we want to have available at
when we die at the end of the 85 th year. We want to know the value of this money on
Marvin's 60th birthday.

FV = 100000
N = 25
I = 10
PV = ? = $9,229.60

Step 4: Subtract the Present Value of the Contingency fund from the money available for
retirement needs.

Available for Retirement needs $1,228,501.95


- Value of Contingency fund at Age 60 $9,229.60

338
= Money that can be used for retirement living $1,219,272.35

So, on Marvins’s 60th birthday he will have $1,228,501.95 in the bank. Of this $1,228,501.95,
Marvin will have to set aside $9,229.60. By setting aside $9,229.60 into an account that pays
10 percent interest, Marvin will have the 100,000 available upon his death. So, Marvin can
use $1,228,501.95 - $9,229.60 = $1,219,272.35 for his retirement.

Step 5: Now we can calculate the amount of the payments that Marvin can take out of the
account each year. We will use an annuity due for this calculation, because Marvin
wants to take the first payment out of the account on his 60th birthday, the time that
we know the present value to be $1,219,272.35.

PV = 1,219,272.35
N = 25
I = 10
PMT = ? = $122,113.53

Thus, Marvin can take $122,113.53 out of the account each year for retirement and
still have $100,000 available upon his death.

41. This problem is a combination problem

Step 1: Draw a time line

0 1 2 3 4 5 ∞
|--------|--------|--------|--------|--------|-------
$200 $200 $200 $200 220 ………

Step 2: Calculate the present value of a growing ordinary perpetuity.


PVGOP4 = 220/.15-.10 = $4400

Step 3: Redraw the Time Line to reflect our computations

0 1 2 3 4 5 6 7 8 9
|--------|--------|--------|--------|--------|--------|--------|--------|--------|
$200 $200 $200 $200
4,400
4,600

0 1 2 3 4 5 6 7 8 9
|--------|--------|--------|--------|--------|--------|--------|--------|--------|
$200 $200 200 $4,600

Step 4: The above cash flows are an uneven cash flow stream. So
we can compute the present value of an uneven cash flow stream.

C01 = 200
C02 = 200
C03 = 200
C04 = 4600

I = 15 NPV = ? = $3086.71

339
42. A. This problem involves solving for the interest rate in a single sum problem.

0 1 2
|------------------------|------------------------|
-86,833 85,320

PV = -86,833
FV = 85,320
N=2
I=?

I = -0.87504 percent.

So Microsoft’s sales growth was negative, declining by 0.87504 percent per year.

*. Notice that the numbers were not converted to millions. This is because it would create a
number too large to input into the calculator. However, the result is the same if we use the “in
millions” numbers.

B. This problem involves solving for the interest rate in a single sum problem.

0 1 2
|------------------------|------------------------|
-22,074 16,798

PV = -22,074
FV = 16,798
N=2
I=?

I = -12.766 percent.

So Microsoft’s net income growth was negative, declining by 12.766 percent per year.

Thus, Microsoft’s net income declined faster than sales. This indicates issues with both sales
and profitability.

340
Chapter 10: Stock Valuation

341
The General Valuation Model
The General Valuation Model tells us that the value of any asset
can be computed by calculating the present value of the cash flows
(benefits) that the asset will provide us in the future.

The value of your house can be computed as the present value of


the benefit of living in the house plus any appreciation in value.

The value of a painting is the joy you attach to viewing the


painting in your home plus any price appreciation.

The value of your car is the present value of the benefits you
receive from driving the vehicle

The value of your children is the joy you receive from watching
them grow up.

-- Some cultures, children are expected to provide for their


parents in retirement. In this case the value of your children is
the present value of the proceeds your children will provide to
you.

-- Some cultures, children are viewed as inexpensive labor. The


size of farms in years past was related to the number of
children in the family to work the operation.

342
Valuing Your Business
The amount that you could realize in disposing your business is the
value of your company. There are several methods available to
dispose of your business. The best method depends upon the
answer to several questions.

If the Company is worth more dead than alive – You should look
to the balance sheet to determine the value of your company.

1. Liquidation of the Assets


2. Declare Bankruptcy
a method of forcing the people who have lent money to
the company to purchase a portion of your company
from you at a high price.

If the Company is worth more alive than dead – you should look to
the income statement to determine the value of your company.

1. Sell your ownership stake in the company to someone else

a. other investors
b. employees

2. Merge your company into another company

In order to determine which method is best for you, you should


compare the amount of money you will receive for the business
under each of the four possibilities.

343
Notes:

Liquidation
-- Do not ignore the costs of disposing of the assets
a. real estate transaction fees
b. cost of having an employee monitor the sales
c. insurance costs

Approximate cost of selling the assets: 5-25 percent of


the sales price.

Sell your ownership stake in the company to someone else

a. determining a fair price for the company


Use a valuation model.

a. Find someone to purchase your company


make sure their money is good

344
Valuation Models:
1. Dividend Valuation Models:

The general idea behind dividend valuations models is to


estimate the current value of what the business will earn in
the future.

To Value your company using a Dividend (Earnings)


Valuation Model you need three pieces of information.

A. Information about current Dividends (Earnings)


-- Obtained from Financial Statements

B. Information about the appropriate discount rate


-- estimated based on the riskiness of the firm

C. Information about how earnings or dividends will


change in the future

-- requires an assumption

Note: You can complete this analysis using dividends or earnings.


Either approach, applied correctly, will produce exactly the same
answer. For convenience, the most common approach is to use
dividends, which is what we will use in class.

345
Constant Growth Valuation Model

Also often called the Gordon Model

-- Assumes that Dividends will grow at some constant


rate forever. (That is that g is constant forever)
-- Assumes Kreq >g

Suppose we want to value the shares of a company that paid a $10


dividend yesterday. Dividends are expected to grow at a constant
rate of 6 percent per year forever. You require a 12 percent rate of
return on this type of investment.

What is the value of this stock today?

^ D − 0(1 + g ) D1
Po = =
Kreq − g Kreq − g

Where:
^
P 0 = the value of a share of the firms stock
D-0 = the amount of the most recent dividend of the company
D1 = the dividend we will receive at time 1
g = the growth rate in earnings or dividends
Kreq = the required rate of return of investors
Kreq = Ks (your book uses Ks, I use kreq) they both refer to the
same thing.

For our example:

^ 10(1 + .06)
Po =
.12 − .06

= $176.67

346
The Zero Growth Dividend Valuation Model

Assumes that dividends will never change

-- that is: it assumes that dividends will stay at the same


amount every year.

-- A special Case of the Constant Growth Model

The Constant Growth Model:

^ D − 0(1 + g )
Po =
Kreq − g

If we set g = 0 because there is no growth in dividends

^ D − 0(1 + 0)
Po =
Kreq − 0

And we can simplify this model by eliminating the zeros so that:

^ D − 0(1 + 0)
Po =
Kreq − 0

^ D1
Po = D − 0 =
Kreq Kreq

347
Suppose that we want to determine the value of a stock that just
paid a dividend of $10 per share. These dividends are expected to
be the same amount each year forever. You require a 12 percent
return on this type of investment.

What is the Value of this stock today?

^ $10
Po =
0.12

= $83.33

348
Supernormal Growth Model

Assume that dividends grow at some rate for a limited amount of


time followed by growth at a constant rate forever.

The equation is complex. However, we can solve this problem by


calculating the present value of a constant growth firm, then
calculating the present value of an uneven cash flow stream.

Suppose we are considering investing in a stock that just paid a


dividend of $10 per share. The dividends will grow at a 30 percent
rate for the next 2 years. After that, they will grow at a 6 percent
rate forever. You require a 12 percent return on this type of
investment.

What is the value of a share of stock in this Company?

Step 1: Calculate the dividends for one more year than the
company will have supernormal growth.

In this case the supernormal growth will last for 2


years. So we must calculate the dividends for the first 3
years.

Dt = Dt-1(1+g)
D1 = D-0(1+g) = $10 (1+0.3) = $13.00
D2 = D1(1+g) = $13.00 (1+0.3) = $16.90
D3 = D2(1+g) = $16.90 (1+0.06) = $17.91

0 1 2 3
|--------------|--------------|--------------|-------------> .....
13.00 16.90 17.91 ..........

349
Step 2: Calculate the value of the shares at time that the
dividends start growing at a constant rate.

In this case, we will calculate the value of the company at


the end of the second year.

Recall that after the second year the dividends will grow at a
constant rate, so we can use the constant growth model to
calculate the value of the company at the end of the second
year.

^ D3
P2 =
Kreq − g

$17.91
=
.12 − .06

= $298.50

This is what we would expect the shares to be worth at the end of


the second year.

350
Step 3: Calculate the present value of an uneven Cash Flow
Stream.

0 1 2
|--------------|--------------|
$13.00 $16.90
$298.50
$315.40

CF0 = 0
CF1 = $13.00
CF2 = $315.40

NPV: I = 12: NPV = ????: NPV =$263.04

The shares are worth $263.04

351
2. Comparables Method

a. Find a similar company that has been recently traded in


the market.

Suppose that your firm is similar in many respects to


Subway. So you select this firm as your comparison firm.

b. Obtain the price to earnings ratio the comparable firm that


has recently traded in the market

Price to earnings ratios are published each day for each


stock in the Wall Street Journal.

Suppose that you find that the Price to Earnings ratio for
Subway is 18.

c. Determine the Value of your firm.

Suppose that your firm recently reported earnings of $10


per share for the past year

^
P o = EPS X PEC

EPS = earnings of your firm per share


PEC = P/E ratio for comparison firm.

= $10 X 18

= $180.00

352
From Share Price to Firm Value

Suppose that we have just computed the value of a share of stock


in the Lockheed Martin Company to be $250.29. We also obtain
information from the balance sheet of Lockheed Martin that tells
us that the firm has 100,000,000 shares outstanding. We wish to
know what the entire equity stake in the company is worth.

^ ^
V o = P o X SO

^
V o = Value of Equity Stake in Company
SO = Shares Outstanding

^
V o = $250.29 X 100,000,000

^
V o = $25,029,000,000

353
Computing the Required Rate of Return

A. Using Historical Average Returns

B. Using the Security Market Line (CAPM)


The Secturity Market Line Tells us that:

Kreq = KRF + B( Km − KRF )

Suppose that we have determined that the return on the market will
be 11 percent per year for the foreseeable future. The risk free rate
of interest is estimated to be 6 percent. We look in Value Line and
find the beta for the stock we are analyzing to be 1.2.

What is the required rate of return on this stock?

Kreq = 6 + 1.2(11 - 6)

= 12 percent.

354
Chapter 10: Homework Assignment and
Solution

355
Chapter 10 Homework:

Stock Valuation Problems:

Problem 1: You are considering investing in a common stock. You will receive a payment of $30 one year
from today from the stock. You will continue to receive payments from the stock each year
forever but the payments will become 10 percent larger each year. You require a 12 percent rate
of return on this type of investment. How much is the most you should be willing to pay to buy a
share of this stock?

Problem 2: I am considering buying a share of stock. The stock will pay me $100 per year forever with
the first payment to be made at the end of the first year. I think the appropriate discount rate
(required rate of return) for this stock is 12 percent. How much is this stock worth today?

Problem 3: Your broker offers to sell you some shares of Bahnsen and Co common stock that paid a
dividend of $2.00 yesterday. You expect the dividend to grow at the rate of 5 percent per year for
the next three years, and if you buy the stock, you plan to hold it for three years and then sell it for
$34.73. You require a 12 percent rate of return on this type of investment. What is the value of
the stock today?

Problem 4: The Vegas Gambling Company has just developed a new game of chance that is very popular
with customers, and very profitable for the casino. Because of this development, The Vegas
Gambling Company is expected to experience a 12 percent annual dividend growth rate for the
next 4 years. By the end of 4 years, other firms will have developed comparable technologies.
After which, The Vegas Gambling Company’s dividend growth rate will slow to 5 percent per
year indefinitely. Investors require a return of 12 percent on The Vegas Gambling Company
Stock. The Vegas Gambling Company paid a dividend yesterday of $2.00 per share. The firm has
20 million shares outstanding. What is the current value of a share of The Vegas Gambling
Company stock? What is the value of the equity stake in the entire company?

Problem 5: The Bah Bah Black Sheep Corporation is expanding rapidly. The company is considered to be
similar to the Microsoft Corporation by some investors, but is somewhat riskier. The firm
currently retains all of its earnings. It has not paid a dividend since the inception of the company.
However, investors expect that the company will begin paying dividends in the future.
Specifically investors expect the first dividend of $2.00 to be paid 8 years from today. The
dividend is expected to grow at a 50 percent per year rate during Years 9 and 10. After Year 10,
the company should grow at a constant rate of 5 percent per year. The required return on the stock
is 20 percent, what is the value of the stock today?

Problem 6: You are considering purchasing the Big Kine Doughnut Company. You are debating how
much to pay for the company. Last year the Big Kine Doughnut Company had earnings of $15
per share. You examine the Wall Street Journal and find that the Price to earnings ratio for Krispy
Kreme Doughnut Company, a company you think is similar to the Big Kine Doughnut Company
to be 22. The Big Kine Doughnut Company has 50,000 shares outstanding. Based on this
information, what is the appropriate price for the Big Kine Doughnut Company?

356
Solutions to Chapter 10 Homework
Problem 1: This is a constant growth valuation problem. We are given D1 = 30, Ks = 12, and g = 10

Po = D1/(kreq – g) = 30/(.12-.10) = $1,500

Problem 2: This is a zero growth stock

Po = D1/kreq = 100/.12 = 833.33

Problem 3: This is a supernormal growth problem. However, because you are given the stock price three
years from today, part of the work has been done for you.

Calculation of Dividends

D1 = Do (1 + g) = 2(1+.05) = 2.10
D2 = D1(1 + g) = 2.10 (1 + .05) = 2.205
D3 = D2 (1 + g) = 2.205 (1 + .05 = 2.315

So we will buy the stock today, after one year we will receive a dividend of 2.10. After the second
year we will receive a dividend of 2.205. After the third year we will receive the last dividend of
2.315 plus we will sell the stock for 34.73. So at the end of the third year we will get a total of
37.045. We want to know the present value of these cash flows. We can treat this as an uneven
cash flow stream and use our calculator to calculate the present value of the cash flows as follows.

CF0 = 0
C01 = 2.10
C02 = 2.205
C03 = 37.045

NPV, I = 12, Enter, ↓, CPT NPV = 30.00, So the stock is worth $30.00 today.

Problem 4: This is a supernormal growth model. The supernormal growth will last for 4 years so we need
to calculate the amount of the dividend for the next 5 years. (calculations are rounded to the
nearest penny.

D1 = D0 (1 +g) = 2.00 (1.12) = 2.24


D2 = D1 (1 +g) = 2.24 (1.12) = 2.51
D3 = D2 (1 +g) = 2.51 (1.12) = 2.81
D4 = D3 (1 +g) = 2.81 (1.12) = 3.15
D5 = D4 (1 +g) = 3.15 (1.05) = 3.31

Now we must compute the price of the stock at the end of the fourth year using the constant
growth model.

P4 = D5/(kreq-g) = 3.31/(.12-.05) = $47.29

So at the end of the fourth year we will get a dividend of $3.15 and the stock will be worth $47.29,
for a total of $50.44. We can draw a time line to show the cash receipts

0 1 2 3 4
|-----------|---------------|-----------------|--------------|
0 2.24 2.51 2.81 3.15
47.29
$50.44

357
Now we can use the cash flow function in our calculator to compute the present value of an
uneven cash flow stream.

CF0 = 0
C01 = 2.24
C02 = 2.51
C03 = 2.81
C04 = 50.44 NPV, I = 12, Enter, ↓, CPT NPV = $38.06, So the stock is worth $38.06
today.

The Value of the entire equity stake in the company is

Share Price X Shares Outstanding = Value of the Company

= 38.06 X 20,000,000 = $761,200,000

Problem 5:

This is a supernormal growth model. The supernormal growth will last for 10 years, so we need to
calculate the first 11 dividends:

D1 = given = 0
D2 = given = 0
D3 = given = 0
D4 = given = 0
D5 = given = 0
D6 = given = 0
D7 = given = 0
D8 = given = $2.00
D9 = D8 (1 +g) = 2 (1.5) = 3
D10 = D9 (1 +g) = 3 (1.5) = 4.5
D11 = D10 (1 +g) = 4.5 (1.05) = 4.73

Now we must compute the price of the stock at the end of the tenth year using the constant growth
model.

P10 = D11/(kreq-g) = 4.73/(.20-.05) = $31.53

So at the end of the tenth year we will get a dividend of $4.50 and the stock will be worth $31.53,
for a total of $36.03. We can draw a time line to show the cash receipts

0 1 2 3 4 5 6 7 8 9 10
|-----------|------------|-----------|----------|-----------|-----------|------------|-----------|----------|-----------|
0 0 0 0 0 0 0 0 $2.00 $3.00 $36.03

358
Now we can use the cash flow function in our calculator to compute the present value of an
uneven cash flow stream.

CF0 = 0
C01 = 0
C02 = 0
C03 = 0
C04 = 0
C05 = 0
C06 = 0
C07 = 0
C08 = $2
C09 = $3
C10 = $36.03

NPV, I = 20, Enter, ↓, CPT NPV = $6.87, So the stock is worth $6.87 per share today.

Problem 6

^
P o = EPS X PEC

= $15 X 22

= $330.00 per share

^ ^
V o = P o X SO

= $330 X 50,000

= $16,500,000

359
Chapter 9: Bonds and Their
Valuation

360
Chapter 9: Bonds and their Valuation

Bonds are Firms borrowing money directly from investors.

Two Ways a firm Can Borrow Money


1. Borrow From a Bank

When a firm borrows money from a bank, the bank


builds in a profit for itself.

Firm --- 8% ---- Bank------4%---Savers

Thus the bank has a 4 percent margin.

A very popular method of borrowing from banks today


is through the use of credit cards. By using a credit
card, your business is not subject to the same scrutiny
by the bank as you are when making a business loan.

About 2/3 of new businesses use credit card debt as a


source of money.

2. Issue Bonds

When a firm issues bonds, it avoids the middle man (the


bank) and borrows money directly from investors

Firm --- 6% ----Savers

The firm is happy because they borrowed at a 2 percent


lower interest rate. The investors are happy because
they received a 2 percent higher return.

361
Bond Terminology

Face Value, Maturity Value, Par Value:


-- The amount of money the firm agrees to pay to the
investors at the end of the loan. (almost always $1,000)
Maturity

-- The amount of time until the firm agrees to pay the face
value back to the investors.

Maturity Date

-- the specific date that the firm will pay the face value to the
investors

Issue Date: The day that the company issues the bonds (borrows
the money)

Coupon Payments

-- periodic interest payments made by the firm to the


investors

Coupon Rate

-- the rate used to determine the amount of any periodic


interest payments.
Suppose we have a bond with a $1,000 face value and a
9 percent annual coupon rate. What is the annual
coupon payment on the bond?
Computing the Coupon Payment

Face Value $1,000


X Coupon Rate .09
= Coupon Payment $90

362
Call Provision

-- Some bonds are callable, allowing the borrower to pay off


the loan early. These bonds are referred to as callable
bonds.

-- Call Protection

Usually Bonds that are callable have a period of time


that they may not be paid off.

-- Call Premium

Some callable bonds have a penalty for early payoff.


This penalty is referred to as a call premium.

Convertible Bonds

-- Convertible Bonds may be converted into a prespecified


amount of common stock.

Default Risk

-- The chance that the company that borrows the money will
not pay the investors as agreed upon.

Kreq = the required rate of return of investors for buying the bond
(making the loan)

363
Payoff to Investors From A Bond

Suppose that we have a $1,000 face value bond with 10 years to


maturity and a 10 percent annual coupon rate. Draw a timeline to
indicate the receipts that the investors will receive.

Step 1: Compute the Coupon Payments

Face Value $1,000


X Coupon Rate .10
= Coupon Payment $100

Step 2: Draw a timeline

0 1 2 3 4 10

|----------|----------|----------|----------|-----…-----|

$100 $100 $100 $100 $100


$1,000

364
Bond Valuation

Recall that The General Valuation Model tells us that the value of
any asset can be computed by calculating the present value of the
cash flows (benefits) that the asset will provide us in the future.

The General Valuation Model can be applied to Bonds.

Suppose that we have a $1,000 face value bond with 12 years


to maturity and a 10 percent annual coupon rate. Suppose
further that investors require a 10 percent return on this type
of investment. What is the value of this bond today?

0 1 2 3 4 12

|----------|----------|----------|----------|-----…-----|

$100 $100 $100 $100 $100


$1,000

Examining these cash flows to determine which technique to


apply to compute the present value:

-- There is a series of cash flows


-- There is a limited number of cash flows
-- There is an equal time spacing between each of the cash
flows
-- Each Cash Flow is not for an equal amount

365
Stream of Cash Yes Limited Number Yes Equal Time Yes Equal Dollar Yes
Annuity
Flows? of Cash Flows? Spacing? Amount?

No No No No

Uneven Cash Uneven Cash


Single Sum
Flow Stream Flow Stream

Equal Time Yes Equal Dollar Yes


Perpetuity
Spacing? Amount?

No No

Constant Yes Growing


Percentage
Perpetuity
Growth?

No

Combination Combination
Problem Problem

Thus the cash flows are an uneven cash flow stream

366
Computing the Present Value

CF
CF0 =0
C01 = 100 ↵ ↓
F01 = 11 ↵ ↓
C02 = 1100 ↵

NPV
I = 10 ↵ ↓
CPT NPV = ? NPV = $1,000

The present value is $1,000, so we would expect to pay $1,000 for


the bond in the market.

Notice that the current price of the bond is equal to the face value.

Whenever the coupon rate is equal to the required rate of


return, the current price of the bond will be equal to the face
value.

Whenever the coupon rate is not equal to the required rate of


return, the current price of the bond will be different than the
face value.

367
Bond Price Changes when Interest Rates
Change

When interest rates change, bond prices change.

There is an inverse relationship between interest rate levels


and bond prices.

When interest rates increase, bond prices go down.

When interest rates decrease, bond prices go up.

Suppose we buy the bond from the previous problem: That is, we
have a $1,000 face value bond with 12 years to maturity and a 10
percent annual coupon rate. One minute after the bond was issued
interest rates, and thus investors required rates of return, go up to
14 percent. What is the value of this bond now?

CF
CF0 =0
C01 = 100 ↵ ↓
F01 = 11 ↵ ↓
C02 = 1100 ↵

NPV
I = 14 ↵ ↓
CPT NPV = ? NPV = $773.59

368
Suppose we buy the bond from the previous problem: That is, we
have a $1,000 face value bond with 12 years to maturity and a 10
percent annual coupon rate. One minute after the bond was issued
interest rates, and thus investors required rates of return, go down
to 6 percent. What is the value of this bond now?

CF
CF0 =0
C01 = 100 ↵ ↓
F01 = 11 ↵ ↓
C02 = 1100 ↵

NPV
I=6↵ ↓
CPT NPV = ? NPV = $1,335.35

369
Notes: Bond Price Changes when interest rates change

The longer a bond has until maturity, the more the price of the
bond will change when interest rates change

Suppose that instead of having 12 years to maturity, the above


bond has 100 years to maturity. How will the price of the bond
change when interest rates change?

When Interest Rates go Up


CF
CF0 =0
C01 = 100 ↵ ↓
F01 = 99 ↵ ↓
C02 = 1100 ↵
NPV
I = 14 ↵ ↓
CPT NPV = ? NPV = $714.29

When Interest Rates go Down

CF
CF0 =0
C01 = 100 ↵ ↓
F01 = 99 ↵ ↓
C02 = 1100 ↵

NPV
I=6↵ ↓
CPT NPV = ? NPV = $1,664.70

Notice, the longer the bond had to maturity the more the bond
price increased when interest rates declined. The longer the bond
had to maturity, the more the price decreased when interest rates
increased.

370
Semi-Annual Coupon Payments and Bond
Valuation

Many bonds have semi-annual coupon payments. When this is the


case, we must adjust our calculations

Suppose that we are interested in a bond with a $1,000 face value,


12 years to maturity and a 10 percent annual coupon rate. Coupon
payments are made semi-annually. Investors require a 10 percent
annual rate of return on this investment. What is the value of this
bond today?

Step 1: Compute the Annual Coupon Payment

Face Value $1,000


X Annual Coupon Rate 0.10
= Annual Coupon Payment $100

Step 2: Compute the Semi-Annual Coupon Payment

Annual Coupon Payment $100


Number of Coupon Payments per Year 2
Periodic Coupon Payment $50

0 1 2 3 4 12

|----|-----|-----|----|----|-----|----|-----|-----…-----|

$50 $50 $50 $50 $50 $50 $50 $50 $50


$1,000

371
To solve this problem we must use the adjust the calculation
method that we learned in Chapter 8

Recall that

Ncalc = N X M = 12 X 2 = 24 (23 + 1)

I 10
Icalc = = = 5%
M 2

The payments will be the semi-annual payments as computed


of $50

CF
CF0 =0
C01 = 50 ↵ ↓
F01 = 23 ↵ ↓
C02 = 1050 ↵

NPV
I=5↵ ↓
CPT NPV = ? NPV = $1,000.00

372
Suppose we are interested in a bond with a $1,000 face value, 12
years to maturity and a 10 percent annual coupon rate. Coupon
payments are made semi-annually. One minute after the bonds
were issued, interest rates, and thus investors required rates of
return go up to 12 percent annually on this investment. What is the
value of this bond now?

CF
CF0 =0
C01 = 50 ↵ ↓
F01 = 23 ↵ ↓
C02 = 1050 ↵

NPV
I=6↵ ↓
CPT NPV = ? NPV = $874.50

373
Computing the Rate Earned on a Bond
Investment
We would like to know the return that we will receive if we
purchase a bond at a certain price.

1. Yield to Maturity

The interest rate that equates the current price of the


bond to the present value of all future cash flows that
will be received from the bond.

The interest rate that the investor will receive if he buys


a bond and holds it until maturity.

To solve for the yield to maturity, we solve for the


interest rate in an uneven cash flow problem

374
Suppose that I am considering purchasing a bond with a
$1,000 face value. The bond has a 9% annual coupon
rate, and coupon payments are made annually. The
bond has 10 years to maturity and a current price of
$887. What interest rate will we earn if we buy the
bond and hold it until it matures?

0 1 2 3 4 10

|----------|----------|----------|----------|-----…-----|

-$887 $90 $90 $90 $90 $90


$1,000

CF0 = -887 ↵ ↓
C01 = 90 ↵ ↓
F01 = 9 ↵ ↓
C02 = 1090 ↵ ↓

IRR
CPT

IRR = 10.91 %

So if we purchase this bond and hold it until it matures,


we will receive a 10.91 percent annual return on our
investment.

375
2. Yield to Call

Recall that some bonds are callable. That is they can be


paid off early at the option of the borrower. In this
case, the investor may not have the choice of holding
the bond to maturity.

The yield to call is interest rate that the investor will


receive if he buys a bond and holds it until it is called.

To solve for the yield to call, we solve for the interest


rate in an uneven cash flow problem

376
Suppose that I am considering purchasing a bond with a
$1,000 face value. The bond has a 9% annual coupon
rate, and coupon payments are made annually. The
bond has 10 years to maturity and a current price of
$914. Suppose that the bond can be called after 5 years
for 103% of par or $1,030. What interest rate will we
earn if we buy the bond and hold it until it is called?

0 1 2 3 4 5

|----------|----------|----------|----------|----------|

-$914 $90 $90 $90 $90 $90


$1,030

CF0 = -914 ↵ ↓
C01 = 90 ↵ ↓
F01 = 4 ↵ ↓
C02 = 1120 ↵ ↓

IRR
CPT

IRR = 11.85 %

So if we purchase this bond and hold it until it is called,


we will receive a 11.85 percent annual return on our
investment.

Usually investors will receive the lower of the Yield to Call or the
Yield to Maturity.

377
Advanced Problems

Suppose that you are considering purchasing a bond that has a


$1,000 face value, and a 3 percent annual coupon rate. When it
was issued, the bond had 20 years to maturity. It is currently14
years after the bonds were issued. The 14th coupon payment was
just made. If you buy the bond today for $1,059, what is your
yield to maturity?

The bond originally had 20 years to maturity. However, the first


14 years of the bond are gone, so they are irrelevant to you. The
first 14 years are history. You should consider only those cash
flows that occur after the 14th year. Time 14 on the following time
line is today.

14 15 16 17 18 19 20
|----------|----------|----------|----------|----------|----------|

-$1,059 $30 $30 $30 $30 $30 $30


$1,000

CF0 = -1,059 ↵ ↓
C01 = 30 ↵ ↓
F01 = 5 ↵ ↓
C02 = 1030 ↵ ↓

IRR
CPT

IRR = 1.949%

378
Suppose that you are considering purchasing a bond that has a
$1,000 face value, and a 3 percent annual coupon rate. Coupon
payments are made semiannually. When it was issued, the bond
had 15 years to maturity. It is 8 years after the bonds were issued.
The 16th coupon payment was just made. The bonds are callable
after they have been outstanding for ten years for $1050. If you
buy the bond today for $1,025, what is your yield to call?
The bond originally had 15 years to maturity. However, the first 8
years of the bond are gone, so they are irrelevant to you. The first
8 years are history. You should consider only those cash flows
that occur after the 8th year (which is today). The bond can be
called after 10 years, so there is two years remaining from today
until the bonds can be called.

8 9 10
|-------------|----------|----------|----------|

-$1,025 $15 $15 $15 $15


$1,050

CF0 = -1,025 ↵ ↓
C01 = 15 ↵ ↓
F01 = 3 ↵ ↓
C02 = 1065 ↵ ↓

IRR, CPT
IRR = 2.055%

However, because the coupon payments are semiannual, this


solution is the internal rate of return for 1/2 year. To identify
the Annual IRR we must multiply by 2.

2.055 X 2 = 4.11

379
Chapter 9 Homework and
Solutions

380
Chapter 9 Homework: Bond Valuation Problems:

Problem 1: Suppose the Master Blaster company wishes to issue some bonds. The bonds will have a
$1,000 face value, 30 years to maturity and a 6 percent annual coupon. How much will Master
Blaster receive for each of the bonds if?

a. Current market interest rates are 5 percent annually.


b. Current market interest rates are 6 percent annually.
c. Current market interest rates are 7 percent annually.

Problem 2: Now Suppose that Master Blaster bonds have an 8 percent coupon rate rather than a 6 percent
coupon rate. Re-compute your answers to parts A, B and C from problem 1.

Problem 3: Suppose that the Jerry Rigger Oil Company wishes to issue some bonds. The bonds will have
a $1,000 face value, 20 years to maturity and a 10 percent annual coupon rate. Coupon payments
will be made semi-annually. How much will the bonds sell for if

a. Current interest rates are 12 percent annually?


b. Current interest rates are 10 percent annually?
c. Current interest rates are 8 percent annually?

Problem 4: Suppose that Parker Ranch Company sold an issue of bonds that had an original maturity of 10
years. The bonds had a $1,000 face value and a 10 percent annual coupon rate. The bonds makes
semi-annual coupon payments.

a. Three years after the bonds were issued, (immediately after the 6th coupon payment), the
going rate of interest (the required rate of return) on bonds similar to this fell to 8 percent
annually. What price would you expect the bonds to sell for?

b. Suppose that two years after the bonds were issued (immediately after the fourth coupon
payment) the going annual rate of interest on bonds (the required rate of return) similar to this
increases to 12 percent annually. What price would you expect the bonds sell for?

Problem 5: The Pokemon Company has two bond issues outstanding. Both bonds have a 10 percent
annual coupon rate and a $1,000 face value. Bond long term has a maturity of 15 years. Bond
short term has a maturity of 1 year. Compute the value of each bond when the required rate of
return is:

a. 5 percent annually
b. 8 percent annually
c. 12 percent anually

Problem 6: The Hicory Dickory Dock Company’s bonds were issued with an original maturity of 10 years
and a $1,000 face value. The bonds have an 8 percent annual coupon rate. There are currently 5
years remaining until maturity. The fifth coupon payment was just made. What is the yield to
maturity on this bond if the current price of the bonds is:

a. $904
b. $1,144

Problem 7: The Bevis and B-head company issued bonds with an original maturity of 30 years and were
callable after being outstanding for 5 years at a price of 1,090. The face value of the bonds is
$1,000. The bonds have a 9 percent annual coupon. The bonds were issued three years ago. The
third coupon payment was just made. Today, you are considering purchasing one of the bonds at a
price of $1,175. You wish to know what rate of return you will be earning on the bonds.
a. Compute the yield to maturity for the bond.
b. Compute the yield to call for the bond.

381
Solutions to Chapter 9 Homework
Problem 1:

a. CF
CF0 =0 ↵ ↓
C01 = 60 ↵ ↓
F01 = 29 ↵ ↓
C02 = 1060 ↵

NPV I = 5 ↵ ↓ , CPT NPV = ? NPV = $1,153.72

b. CF
CF0 =0 ↵ ↓
C01 = 60 ↵ ↓
F01 = 29 ↵ ↓
C02 = 1060 ↵

NPV, I = 6 ↵ ↓, CPT NPV = ? NPV = $1,000.00

c. CF
CF0 =0 ↵ ↓
C01 = 60 ↵ ↓
F01 = 29 ↵ ↓
C02 = 1,060 ↵

NPV
I=7↵ ↓, CPT NPV = ? NPV = $875.91

Problem 2:

a. CF
CF0 =0 ↵ ↓
C01 = 80 ↵ ↓
F01 = 29 ↵ ↓
C02 = 1,080 ↵

NPV, I = 5 ↵ ↓, CPT NPV = ? NPV = $1,461.17

b. CF
CF0 =0 ↵ ↓
C01 = 80 ↵ ↓
F01 = 29 ↵ ↓
C02 = 1,080 ↵

NPV, I = 6 ↵ ↓, CPT NPV = ? NPV = $1,275.30

c. CF
CF0 =0 ↵ ↓
C01 = 80 ↵ ↓
F01 = 29 ↵ ↓
C02 = 1080 ↵

NPV, I = 7 ↵ ↓, CPT NPV = ? NPV = $1,124.09

382
Problem 3:

a. CF
CF0 =0 ↵ ↓
C01 = 50 ↵ ↓
F01 = 39 ↵ ↓
C02 = 1050 ↵

NPV
I=6↵ ↓
CPT NPV = ? NPV = $849.54

b.
CF
CF0 =0 ↵ ↓
C01 = 50 ↵ ↓
F01 = 39 ↵ ↓
C02 = 1050 ↵

NPV
I=5↵ ↓
CPT NPV = ? NPV = $1,000

c.
CF
CF0 =0 ↵ ↓
C01 = 50 ↵ ↓
F01 = 39 ↵ ↓
C02 = 1050 ↵

NPV
I=4↵ ↓
CPT NPV = ? NPV = $1,197.93

383
Problem 4:

The annual coupon payment can be computed as the par value * the coupon rate = 1000 * .1 =
100. However in this case we will get half of the payment 100/2 = 50 every 6 months.

The timeline for the bond when it was issued:

0 1 2 3 …. 7 8 9 10
|-----|------|-------|-------|------|-------|------|------|------|------|------|------|------|------|
50 50 50 50 50 50 50 50 50 50 50 50 50 50
1,000

a. The timeline for the bond after three years have passed:

3 … 7 8 9 10
|-------|-- ---|------|------|------|------|------|------|
50 50 50 50 50 50 50 50
? 1,000

CF
CF0 =0 ↵ ↓
C01 = 50 ↵ ↓
F01 = 13 ↵ ↓
C02 = 1050 ↵

NPV, I = 4 ↵ ↓, CPT NPV = ? NPV = $1,105.63

b. The timeline for the bond after 2-years have passed

2 3 …. 7 8 9 10
|------|-------|------|------|------|------|------|------|------|------|
50 50 50 50 50 50 50 50 50 50
? 1,000

CF
CF0 =0 ↵ ↓
C01 = 50 ↵ ↓
F01 = 15 ↵ ↓
C02 = 1050 ↵

NPV, I = 6 ↵ ↓, CPT NPV = ? NPV = $898.94

384
Problem 5:

Timeline for the one-year bond:

0 1
|--------|
1,000
100

Timeline for 15-year bond:

0 1 … 5 6 7 8 9 … 15
|--------|--------|--------|--------|--------|--------|--------|
1,000
? 100 100 100 100 100 100 100

One-year bond 15-year bond

a. CF CF
CF0 = 0 ↵ ↓ CF0 = 0 ↵ ↓
C01 = 1100 ↵ ↓ C01 = 100 ↵ ↓
F01 = 14 ↵ ↓
C02 = 1100 ↵ ↓

NPV NPV
I=5↵ ↓ I=5↵ ↓
CPT NPV = ? NPV = $1,047.62 CPT NPV = ? NPV = $1,518.98

b. CF CF
CF0 = 0 ↵ ↓ CF0 = 0
C01 = 1100 ↵ ↓ C01=100 ↵ ↓
F01 = 14 ↵ ↓
C02 = 1100 ↵ ↓

NPV NPV
I=8↵ ↓ I=8↵ ↓
CPT NPV = ? NPV = $1,018.52 CPT NPV = ? NPV = $1,171.19

c. CF CF
CF0 = 0 ↵ ↓ CF0 = 0 ↵ ↓
C01 = 1100 ↵ ↓ C01=100 ↵ ↓
F01 = 14 ↵ ↓
C02 = 1100↵ ↓

NPV NPV
I = 12 ↵ ↓ I = 12↵ ↓
CPT NPV = ? NPV = $982.14 CPT NPV = ? NPV = $863.78

385
Problem 6:

Timeline of bond when it was issued.

0 1 2 3 4 5 6 7 8 9 10
|-------|--------|--------|-------|-------|-------|--------|--------|--------|--------|
? 80 80 80 80 80 80 80 80 80 80
1,000

Timeline for the bond after it has been outstanding for 5 years.

5 6 7 8 9 10
|-------|--------|--------|--------|--------|
? 80 80 80 80 80
1,000

a. CF0 = -904 ↵ ↓
C01 = 80 ↵ ↓
F01 = 4 ↵ ↓
C02 = 1080 ↵ ↓

IRR
CPT

IRR = 10.57 %

b. CF0 = -1144 ↵ ↓
C01 = 80 ↵ ↓
F01 = 4 ↵ ↓
C02 = 1080 ↵ ↓

IRR
CPT

IRR = 4.70 %

386
Problem 7:

Timeline of the bond when it was issued:

0 1 2 3 4 5 6 7 8 9 … 30
|-------|--------|--------|-------|-------|-------|--------|--------|--------|--------|
? 90 90 90 90 90 90 90 90 90 90
1,000

There are 27 years remaining until the bond matures and two years remaining until the call date. The bond
was originally a 30 year maturity/5 year callable bond, however, three years have passed since the bond
was issued.

a. Timeline of the bond after three years have passed:

3 4 5 6 7 8 9 … 30
|-------|-------|-------|--------|--------|--------|--------|
90 90 90 90 90 90 90
-1,175 1,000

CF0 = -1,175 ↵ ↓
C01 = 90 ↵ ↓
F01 = 26 ↵ ↓
C02 = 1090 ↵ ↓

IRR
CPT

IRR = 7.47 %

b. Timeline if the bonds are called at the end of the 5th year. Notice there are two years remaining on the
bond until it can be called.

3 4 5
|-------|-------|
90 90
-1,175 1,090

CF0 = -1,175 ↵ ↓
C01 = 90 ↵ ↓
F01 = 1 ↵ ↓
C02 = 1180 ↵ ↓

IRR
CPT

IRR = 4.12 %

387
CHAPTER 14:
Cash Flow Estimation

388
Capital Budgeting:

The process of evaluating projects for your company. If you have observed
businesspersons calculating numbers on a napkin in Kens, they are probably
doing capital budgeting. Unfortunately, they are usually doing it incorrectly.
Today we will show you how to do it correctly.

Four Steps

1. Get an Idea

2. Gather information

3. Use that information to compute the cash flows for the


project

4. Evaluate the cash flows to determine if they are sufficient to


warrant starting the business.

389
Step 1:

Where to get Ideas for your business

A. Look around yourself and ask what do I need that I


can not get in Hilo.

Ask yourself if there is a sufficient number of other


people that would like the same thing.

Limousine Service in Hilo?

B. Talk to Friends

Listen carefully to what they tell you that you are


good at.

C. Look at what has been successful in other places.

i. Pokemon in Central America??


ii. Kava Café??

D. Entrepreneurship Magazines

i. Top 100 new small businesses

E. Watch Technological Developments


i. 3-D Printing
ii. Driverless Cars
iii. Nano Technology

390
Step 2:
a. You need good information. The analysis is a garbage in -
garbage out type of analysis.

a. Start-up costs
b. Operating Cash Flows
i. revenues
ii. expenses
c. Terminal cash flows

b. Information can come from a variety of sources

i. potential suppliers
ii. the internet
iii. government provided information (SBDC)
iv. ask the competition (be prepared for a limited
response

Step 3: Identify the relevant cash flows

Some of the cash flows you have identified may not be relevant for
Capital Budgeting Purposes.

Sunk Costs should not be considered. Sunk costs are those costs
that have already been expended, and cannot be recovered if you
decide not to do the project.

Suppose you spend $1,000 to conduct a survey to determine if


your idea for a butterfly farm in Hilo would be a valuable
tourist attraction.

Your survey suggests that it would be a good idea, so you


develop a capital budget. The $1,000 should not be considered
as part of the capital budget.

391
Information for the Project:

You are considering starting an embroidery project in your company. In


order to start the project you will purchase machines from a mainland
supplier that cost a total of $300,000. In addition, freight to Hawaii and
installation of the machines will cost $30,000. These machines will last for
four years. You will use 3-year MACRS depreciation on the machines. For
depreciation purposes, the machine will have a salvage value of $35,000. In
order to operate the business you will need inventory of $43,000. This
inventory will need to be available on the day you start the business and
remain available through the life of the business. You will purchase some of
your supplies on credit. You will owe your suppliers $10,000 at any given
time. You will make the first credit purchase the day you start the business.
You will sell some of your shirts on credit to select customers. You expect
that your customers will owe you $5,000 at any given time. Customers will
make the initial purchase on credit the day you start the business. You
expect all customers that owe you money will pay as required. You will
need $7,000 in working cash on hand at all times for the day-to-day
operations of the firm. You will sell 115,000 shirts at a price of $3.50 each,
per year. 50% of sales will be Cost of Goods Sold. Sales and administrative
expenses will be an additional 20 percent of sales. Your firm’s tax rate is
20%. The cost of capital for your firm is 10% regardless of how much
money that you need. At the end of the fourth year you will sell the machine
and close the business. You will sell the machine for $60,000. When you
close the business you will recover the inventory investment, the accounts
receivable and the working cash and will pay off all outstanding accounts.
Create a capital budget to determine if you should invest in this project.

Step 3: You must Compute three different Cash Flows for the Project

Part 1: Initial Cash Flow: The cash flows associated with starting
your project

Part 2: Operating Cash Flows: The year-to-year cash flows from the
project

Part 3: Terminal Cash Flows: The cash flows associated with closing
your project

392
Annual Cash Flows for the Project

Year 0 1 2 3 4
Total Revenues $402,500 $402,500 $402,500 $402,500
- Cost of Goods Sold $201,250 $201,250 $201,250 $201,250
- Sales and Administrative Costs $80,500 $80,500 $80,500 $80,500
- Depreciation $108,900 $148,500 $49,500 $23,100
= Earnings before Interest & Tax (EBIT) $11,850 -$27,750 $71,250 $97,650
- Interest $0 $0 $0 $0
= Earnings Before Taxes (EBT) $11,850 -$27,750 $71,250 $97,650
- Tax (@20 percent) $2,370 -$5,550 $14,250 $19,530
= Net Income $9,480 -$22,200 $57,000 $78,120
+ Depreciation $108,900 $148,500 $49,500 $23,100
=Net Operating Cash Flows $118,380 $126,300 $106,500 $101,220

Equipment Cost -$300,000


Installation -$30,000
Increase in Net Working Capital (NWC) -$45,000

+ Sales Price $60,000


- Tax on Gain on Sale -$12,000
+ Return of Net Working Capital (NWC) $45,000

Net Cash Flows -$375,000 $118,380 $126,300 $106,500 $194,220

393
Part 1: Initial Cash Flows

A. Computation of Working Capital Requirements

Increase in Inventory ($43,000)


Increase in Accounts Payable $10,000
Increases in Accounts Receivable ($5,000)
Other Working Assets ($7,000)
- Other Working Liabilities $0
Required Working Capital ($45,000)

B. Computation of Initial Cash Flow

Required Working Capital ($45,000)


Purchase of Machine ($300,000)
Freight and Installation ($30,000)
Total Initial Cash Flow ($375,000)

394
Part 2: Operating Cash Flow

A. Depreciation Computations
When we buy a machine, we spend money. The money is spent
today, however; the machine will last for more than one year.
So we calculate the part of the machine that wore out this year.
That part of the machine that wore out this year is an expense
and is called depreciation.

B. Unfortunately, it is difficult to estimate precisely how much our


machine will wear out in each year. There are several methods
available for estimating the amount that your machine wore out.
Regardless of the method used, we need to estimate two things:

i. How long the machine will last. The IRS


provides some guidelines in this regard.

Semi-Trucks 3 Years
Automobiles 5 Years
Office Furniture 7 Years
Residential Real Estate 27.5 Years
Commercial Real Estate 39 Years

Note: Except in a few unusual circumstances


Land is not Depreciated.

Note: I will tell you the classification of the


equipment on the exam. You will not need to
make this determination on the exam.

ii. We also need to compute the cost of the


machine (The Depreciable Base).

Purchase price of Machine $300,000


Freight and Installation $30,000
Depreciable Base $330,000

395
Method I: Straight Line Depreciation. The concept behind
straight line depreciation is that our machine will wear out by
the same amount each year.

DepreciableBase − SalvageValue
AnnualDepreciation =
DepreciableLife

$330,000 − 35,000
AnnualDepreciation =
3

AnnualDepreciation = $98,333.33

396
Method II: MACRS Depreciation

Step 1: Obtain Depreciation percentages from IRS


Publications

Year 3 Year 5 Year 7 Year


1 33% 20% 14%
2 45% 32% 25%
3 15% 19% 17%
4 7% 12% 13%
5 11% 9%
6 6% 9%
7 9%
8 4%

Note: The MACRS percentages will be provided


to you on the exam.

Step 2: Compute Annual Depreciation

Annual Dep. = Dep. Base X MACRS Rate

Year 1 $330,000 X 0.33 = $108,900


Year 2 $330,000 X 0.45 = $148,500
Year 3 $330,000 X 0.15 = $49,500
Year 4 $330,000 X 0.07 = $23,100

When Using MACRS, the salvage value is ignored in the


computations.

Summary of Depreciation Calculations

Straight Line MACRS Section 179

Year 1 $98,333.33 $108,900 $330,000


Year 2 $98,333.33 $148,500 $0
Year 3 $98,333.33 $49,500 $0
Year 4 $0.00 $23,100 $0

397
Terminal Cash Flows:

Sources of Terminal Cash Flows

1. Recovery of Inventory
2. Recovery of Accounts Receivable
3. Recovery of Working Cash
4. Payment of outstanding accounts
5. Sale of Equipment
6. Tax on Sale of Equipment
5. Various other terminal cash flows might occur.

398
Tax on Sale of Equipment

Suppose we depreciate our machine using MACRS 3 year depreciation.


Recall that after four years we sell the machine for $60,000. We must pay
taxes on the difference between what we told the IRS the machine would be
worth and what we actually sold the machine for.

Computation of Tax on Disposal of Machine

Step 1: Calculate the Depreciation taken to Date

Year 1 $108,900
Year 2 $148,500
Year 3 $49,500
Year 4 $23,100
Depreciation Taken $330,000

Step 2: Calculate the Book Value

Depreciable Basis $330,000


- Depreciation Taken $330,000
= Book Value $0

Step 3: Calculate the Taxable Gain or (Loss)

Sales Price $60,000


- Book Value 0
Taxable Gain $60,000

Step 4: Calculate Tax Owed

Taxable Gain $60,000


X Tax Rate 0.20
Taxes Owed (Recovered) $12,000

399
Terminal Cash Flow

Sales Price of Machine $60,000


Tax on Sale of Machine -$12,000
Recovery of Net Working Capital $45,000
Terminal Cash Flow $93,000

Methods for determining if the cash flows are sufficient to warrant starting
the business or the project:

1. Pay Back Period


2. Internal Rate of Return
3. Net Present Value
4. Profitability Index

400
Chapter 14: Homework and
Solutions

401
Chapter 14 Homework

1. You are planning on purchasing a new sewing machine for your business. The sewing
machine costs $10,000 and is expected to have a $1,000 salvage value. The
sewing machine is classified as three year MACRS property. How much is the
depreciation for each of the first four years?

2. Use the same information as from the above problem, but you have decided to use
straight line depreciation instead of MACRS. The machine will be depreciated
over three years. What is the depreciation for each of the three years?

3. On August 15, 2010 you purchased a machine that cost $15,000. You have been
using the MACRS (3 year life) depreciation on the machine and are in the 34
percent tax bracket. On August 15, 2012 you decide to sell the machine. You sell
the machine for $12,000. You will take the regular 2012 depreciation in your
operating cash flow in the normal fashion. How much in taxes will you have to
pay (or will recover) in 2012 because you sold the machine?

4. You are trying to determine how much working capital you will need for your new
salsa factory. You expect to buy most of your supplies on credit, so you expect to
owe an average of $10,000 to your suppliers at any given time. You expect to
have about 25,000 of inventory on hand at any given time. You will also sell
some of the salsa on credit. You expect that your customers will owe you
$15,000 at any given time. You will need to have $2,000 available for use in the
cash registers at the retail outlet. Compute the net working capital requirements
for this company.

5. You are considering starting a new doughnut shop in the Prince Kuhio Mall. Based
on your sales estimate, you believe that you will be able to sell 220,000 doughnuts
per year at a price of $1.10 per doughnut. You have estimated labor costs to be
$50,000 for the year and it will cost $48,000 per year to rent the space in the Mall
(rent will be paid at the end of each year). The cost of the ingredients to make the
doughnuts will be $.25 per doughnut. You will need to have $10,000 of inventory
and you will need to have $3,000 in cash for the cash registers. Inventory and
cash for the registers will need to be on hand the day you start the business. You
will have to purchase $50,000 of equipment with cash you have saved to
manufacture the doughnuts. This equipment is classified as three year MACRS
property. You have organized your company as a sole proprietorship, so you will
pay 34 percent taxes on the earnings of the firm. You are planning to quit the
business at the end of the third year, at which time you will sell the equipment for
$25,000. When you close the business, you will recover the original inventory
investment & the cash register cash. Create a cash flow schedule for the above
project. You require a 12 percent return on this type of investment.

402
6. You wish to evaluate the possible purchase of a gas chromatograph and total
hydrocarbon analyzer for your geology company. The equipment’s base price is
$40,000, and it would cost another $10,000 to modify it for special use by your
firm. The equipment qualifies for three-year MACRS depreciation, which is what
you will use for your capital budgeting analysis. The equipment, as modified,
would be sold after three years for $20,000. In order to operate the equipment,
you would need to increase the firm’s net working capital (in the form of spare
parts and inventory) by $4,000. The purchase of the equipment would have no
effect on the firm’s revenues. However, it would reduce the expenses of the firm
by $12,000 per year as the firm would no longer need to contract certain work
out. When you sell the equipment, you will recover the $4,000 of net working
capital. The firm’s marginal tax rate is 20 percent. Create a cash flow schedule
for the above project.

403
Chapter 14 Homework Solutions

Question 1

Cost 10000
3 year depreciable life

MACRS
Year Allowance Depreciation
1 0.33 3300
2 0.45 4500
3 0.15 1500
4 0.07 700

Question 2

Depreciation = (Cost-Salvage Value)/Depreciable


Life

= (10000-1000)/3

= 3000

Year Depreciaiton
1 3000
2 3000
3 3000
4 0

404
Question 3

Step 1: Calculate the depreciation taken to date

Depreciable Base 15000

MACRS
Year Year Allowance Depreciation
2010 1 0.33 $4,950
2011 2 0.45 $6,750
2012 3 0.15 $2,250
2013 4 0.07 $1,050

As of the end of 2012, we have taken a total depreciation so far of

= 4950 + 6750 + 2250 = $13,950

Step 2: Compute Book Value

Cost $15,000
- Depreciation Taken $13,950
= Book Value $1,050

Step 3: Calculate Taxable Gain or Loss

Sales Price $12,000


- Book Value $1,050
= Taxable Gain $10,950

Step 4: Compute Tax

Taxable gain $10,950


X Tax Rate 0.34
Tax Owed $3,723

Note: In this example, depreciation in our operating cash flows


would be reported as $2250 in 2012

Question 4

Inventory use $25,000


Accounts Receivable use $15,000
Cash Register use $2,000
-
Assounts Payable source $10,000
Net Operating Cash
Flow $32,000

405
Question 5

Year 0 1 2 3
Total Revenues $242,000 $242,000 $242,000
Ingredients $55,000 $55,000 $55,000
Labor $50,000 $50,000 $50,000
Rent $48,000 $48,000 $48,000
Depreciation $16,500 $22,500 $7,500
Earnings before Interest and Taxes
(EBIT) $72,500 $66,500 $81,500
Interest $0 $0 $0
Earnings Before Taxes (EBT) $72,500 $66,500 $81,500
Tax (@34 percent) $24,650 $22,610 $27,710
Net Income $47,850 $43,890 $53,790
+ Depreciation $16,500 $22,500 $7,500
Net Operating Cash Flows $64,350 $66,390 $61,290

Equipment Cost -$50,000


Inventory -$10,000
Cash Register -$3,000

Sales Price $25,000


Tax on Gain on Sale -$7,310
Recovery of Inventory $10,000
Recovery of Cash Register Funds $3,000
Net Cash Flows -$63,000 $64,350 $66,390 $91,980

406
Question 5 Continued
Depreciation Computations

Depreciable
Base $50,000

MACRS
Year ALLOW Depreciation
1 0.33 $16,500
2 0.45 $22,500
3 0.15 $7,500
4 0.07 $3,500

Tax on Gain on Sale Computations

Step 1: Calculate the depreciation taken to date

We have taken a total depreciation so far of

= 16500 + 22500 + 7500 = $46,500

Step 2: Compute Book Value

Cost $50,000
- Depreciation Taken $46,500
= Book Value $3,500

Step 3: Calculate Taxable Gain or


Loss

Sales Price $25,000


- Book Value $3,500
= Taxable Gain $21,500

Step 4: Compute Tax

Taxable gain $21,500


X Tax Rate 0.34
Tax Owed $7,310

407
Question 6 Cash Flows for the
Project

Year 0 1 2 3
Total Revenues $0 $0 $0
Operating Expenses -$12,000 -$12,000 -$12,000
Depreciation $16,500 $22,500 $7,500
Earnings before Interest and Taxes
(EBIT) -$4,500 -$10,500 $4,500
Interest $0 $0 $0
Earnings Before Taxes (EBT) -$4,500 -$10,500 $4,500
Tax (@20 percent) -$900 -$2,100 $900
Net Income -$3,600 -$8,400 $3,600
+ Depreciation $16,500 $22,500 $7,500
Net Operating Cash Flows $12,900 $14,100 $11,100

Base Price -$40,000


Modificaitons -$10,000
Net Working Capital -$4,000

Sales Price $20,000


Tax on Gain on Sale -$3,300
Recovery of Net Working Capital $4,000
Net Cash Flows -$54,000 $12,900 $14,100 $31,800

408
Question 6 Continued

Depreciation Computations

Depreciable
Base $50,000

MACRS
Year ALLOW Depreciation
1 0.33 $16,500
2 0.45 $22,500
3 0.15 $7,500
4 0.07 $3,500

Tax on Gain on Sale Computations

Step 1: Calculate the depreciation taken to date

Deprecation taken to date

= 16,500 + 22,500 + 7,500 $46,500

Step 2: Compute Book Value

Cost $50,000
- Depreciation Taken $46,500
= Book Value $3,500

Step 3: Calculate Taxable Gain or Loss

Sales Price $20,000


- Book Value $3,500
= Taxable Gain $16,500

Step 4: Compute Tax

Taxable gain $16,500


X Tax Rate 0.2
Tax Owed $3,300

409
CHAPTER 13:

Evaluating Cash Flows


When Cost of Capital is
Fixed

410
The Required Rate of Return

The appropriate required rate of return for Capital Budgeting is the


Weighted Average Cost of Capital (WACC).

The WACC can be computed as follows

WACC = Wst(Kreq) + Wd(Kd)(1-T)

Where:
Wst is the percentage of funds we will obtain from stock
Wd is the percentage of our funds that we will borrow
Kreq is the cost of obtaining money from stock issues (the required rate of
return)
Kd is the before tax cost of borrowing money
T is the firms tax rate

Suppose that our company will obtain 75 percent of needed funds from stock
issues. We will obtain 25 percent of our needed funds by borrowing money.
Suppose further that the cost of obtaining money from stocks is 12 percent
and the before tax cost of obtaining money by borrowing is 6.67 percent.
The firms tax rate is 40 percent.

WACC = .75(12) + .25(6.67)(1-.4)

= 9+1

= 10

411
Where do you get the parameters for computing the WACC

1. The Before Tax Cost of Debt, KD

This is the rate that the bank will charge you on a bank loan

2. The tax rate, T

This is the tax rate of the firm or the individual. You can
estimate this from tables provided by the IRS, based on the income of
the company or the individual.

3. The cost of obtaining money from stock issues, Kreq

This is the difficult parameter to estimate. There is no one method


available to do this.

A. Return on a Risk Free Asset Plus a Risk Premium

Suppose that the current risk free rate of interest (CD rate) is 3
percent. Suppose the additional return required to make this
investment rather than a risk free investment is 4 percent.

Then Kreq = 3 +4 = 7

B. Return on an Alternative Investment

In this Kreq is equal to the rate of return you could receive if you
invested in some alternate project, maybe the stock market. The
stock market has historically produced about a 12 percent return.
Thus you might take the return on the stock market as Kreq.

You might adjust this figure to reflect

i. liquidity premium
ii. risk premium

412
C. CAPM method

The CAPM argues that the appropriate required rate of return can
be computed as:

Kreq=Krf+B(Km-Krf)

Suppose that the return on the market is 12 percent and the risk
free rate of interest is 3 percent. Suppose that the project you are
considering is about as risky as the stock of the Pride Petroleum
Company. Pride Petroleum company has a Beta of 1.15. What is
the required rate of return

Kreq=Krf+B(Km-Krf)

= 3 + 1.15(12-3) = 13.35%

D. Cost of Obtaining Money from others

Venture capitalists who look for 30-40 percent returns

413
Classifications of Projects

Mutually Exclusive:
Accepting one project implies that you will not accept another project

Independent:
You might accept both projects.

Normal Project
You have only one cash flow sign change

0 -$10,000
1 $5,000
2 $7,000
3 $9,000

Non-Normal Project
The project has more than one cash flow sign change

0 -$10,000
1 $5,000
2 $7,000
3 $9,000
4 -$3,000

414
Payback Period:

Tells us the number of years it will take us to get our money back.

Decision Rule:
a. For independent Projects: The decision rule is not defined
b. For Mutually Exclusive Projects: Accept the Project with the
lowest payback period

Strengths:

1. Easy to Compute
2. Easy to Understand
3. Always Gives one answer
Weakness:

1. Ignores the Time Value of Money


2. Ignores Cash Flows that occur after the Payback Period
3. No definition of what constitutes a good or bad payback period.
4. Not appropriate for non-normal projects

415
Computations:

Year Cash Flow Cumulative Cash Flow


0 -$375,000 -$375,000
1 $118,380 -$256,620
2 $126,300 -$130,320
3 $106,500 -$23,820
4 $194,220 $170,400

− LastNegCumCashFlow
The Payback Period is PB = BaseYear +
FollowingCashFlow

− (−23,820)
PB = 3 +
194,220

23,820
PB = 3 +
194,220

PB = 3.123Years

416
Significance of Ignoring the Time Value of Money

Suppose we have two projects, A and B

Period A B
0 -10,000 -10,000
1 5,000 2,000
2 2,000 5,000
3 3,000 3,000

Both projects have a payback period of 3 years. However A is the


better project because you get your money back quicker).

417
Significance of ignoring Cash Flows that occur after the payback period

Period A B
0 -10,000 -10,000
1 5,000 5,000
2 6,000 5,000
3 1,000,000

According to the Payback Period, A is better project. But


project B is clearly the more desirable project.

418
Internal Rate of Return:

The interest rate that equates all future cash flows to the cost of
starting the project.

Decision Rule:

a. Independent Projects: Accept all projects where the Internal Rate


of Return is higher than the WACC (Cost of Obtaining Money
for the company)
b. Mutually Exclusive Projects: Accept the project with the highest
Internal Rate of Return as long as it is higher than the WACC

Strengths:

1. Considers all cash flows


2. Considers the Time Value of Money
3. Assumes that we can reinvest the funds we get back from the
project at the internal rate of return.
4. Considers the riskiness of the cash flows

Weaknesses:

1. Will give multiple answers for non-normal projects.


2. Assumes that we can reinvest at the internal rate of return
3. Requires a Financial Calculator

419
Computations: Assume that the Cost of Obtaining Money for the Project is
10 percent.

CF0 = -$375,000
C01 = $118,380
C0 2 = $126,300
C0 3 = $106,500
C0 4 = $194,220

IRR IRR = 15.55 percent

IRR > Cost of Funds, Accept the project

420
Net Present Value

Computes the Present Value of All Cash Flows from the Project and
Compares them to the cost of the Project

Generally considered to be the best method

Decision Rule:

a. For Independent Projects: Accept all projects with a positive NPV


b. For Mutually Exclusive Projects: Accept the Project with the
Largest NPV as long as it is positive

Strengths:

1. Considers all cash flows


2. Considers the Time Value of Money
3. Always gives one answer
4. Assumes that we can reinvest the funds we get back from the
project at the required rate of return.
5. Considers the riskiness of the cash flows

Weaknesses:
1. Gives misleading answers when projects have different sizes.
2. Gives misleading answers when projects have different lives.
3. Requires a Financial Calculator
4. Assumes that we can reinvest the funds we get back from the
project at the required rate of return.

421
Computations: Assume that the Cost of Obtaining Money for the Project is
10 percent.

CF0 = -$375,000
C01 = $118,380
C0 2 = $126,300
C0 3 = $106,500
C0 4 = $194,220

NPV I = 10
NPV = $49,668.25

Because the Net Present Value is Above $0, we should accept the project.

422
Significance of Misleading answers

Suppose we have two projects;

Project Cost NPV


A 1,000,000 100,000
B 100,000 12,000

A has a higher NPV but it is misleading because it is a much larger project.

423
Profitability Index.

Allows us to compare NPV on project of different sizes.

PI = NPV
Cost of Starting Project

Suppose we have 2 projects:

Project Cost NPV


A 1,000,000 100,000
B 100,000 12,000

PIA= 100,000 = .10


1,000,000

PIB= 12,000 = .12


100,000

Select the project with the highest PI.

424
What to do when projects have unequal lives

Sometimes projects have unequal lives. That is: If you buy the good
machine you can expect it to last 6 years. If you buy the inexpensive
machine, you should expect it to last 3 years.

Suppose that we have the following cash flows. Further suppose that we
have a 11 percent WACC
Year Cash Flow___
A B
0 -$40,000 -$80,000
1 $26,000 $29,000
2 $10,000 $16,000
3 $28,000 $15,000
4 $24,000
5 $10,000
6 $52,000

If we compute the NPV and IRR, we get the following

A B
NPV $12,013.01 $19,625.34
IRR 27.59% 18.55%

As in projects with differing sizes the NPV will be misleading in such cases.
In addition, the Profitability Index may be misleading. So we must adjust
our calculations.

What to do?

Replacement Chain Approach:

In this case, you find the lowest common denominator for the
two projects. If we have a project with a 6-year life and another
with a 3-year life, the lowest common denominator is 6 (two
three year projects makes one 6-year project)

What this involves is assuming that we will repeat project A 3 years


from today: So we get the following Cash Flows

425
Year Cash Flow___
A B
0 -$40,000 -$80,000
1 $26,000 $29,000
2 $10,000 $16,000
3 $28,000 -$40,000 $15,000
4 $26,000 $24,000
5 $10,000 $10,000
6 $28,000 $52,000

Combining the Project A cash Flows we get

Year Cash Flow___


A B
0 -$40,000 -$80,000
1 $26,000 $29,000
2 $10,000 $16,000
3 -$12,000 $15,000
4 $26,000 $24,000
5 $10,000 $10,000
6 $28,000 $52,000

Now we compute the criteria for each of the projects

A with repetition B
NPV $20,796.81 $19.625.34
IRR 27.59% 18.55%

So Project A is the better project

426
Chapter 13 Homework and Solutions

427
Chapter 13 Homework

Problem 1: We are considering two projects. The first project is a restaurant. The restaurant will cost
$10,000 to start. The restaurant will provide a return of $3,000 per year each year for the next
5 years. The second project is a tourism bus. The bus will cost $25,000 to start. The bus will
provide a return of $7,400 per year, each year for the next five years. The cost of capital for the
company is 12 percent. Because your funds are limited, you feel that it is feasible to undertake
only one of the projects.

A. Compute the payback period for each project.


B. Compute the internal rate of return for each project.
C. Compute the net present value for each project.

Problem 2:

You are considering buying a delivery truck. The truck you are considering buying costs $52,125 to buy
and will return $12,000 per year for 8 years. The cost of capital for your company is 12 percent.

A. Compute the payback period


B. Compute the internal rate of return
C. Compute the net present value.

Problem 3:

The CR Taxi Co is considering two mutually exclusive investments. The projects expected cash flows are
as follows:

Year Cash flow for project A Cash Flow for Project B


0 (3000) (3000)
1 1100 1000
2 1100 1000
3 1100 1000
4 1100 1000
5 1100 1000
6 1100 1000
7 (100) 1000

Compute the Net Present Value of each project when


a. the cost of capital is 5 percent
b. the cost of capital is 20 percent

Problem 4: Suppose you are considering investing in the following two projects. The projects are mutually
exclusive. Use the replacement chain approach that we discussed in class to determine which
project should be accepted. The cost of capital for the project is 16%

Year Project Short Project Long


0 $(20,000) $(40,000)
1 $7,000 $9,000
2 $10,000 $14,000
3 $19,000 $15,000
4 $8,000 $15,000
5 $0 $6,000
6 $0 $15,000
7 $0 $8,000
8 $0 $7,000

428
Problem 5: Suppose that our company will get 30 percent of the funds we need from stock issues and
70 percent of the funds we need from borrowing. The firms tax rate is 40 percent. The cost of
obtaining money from stocks is 10 percent, and the cost of obtaining money from borrowing is 5
percent. What is the weighted average cost of capital for our company?

429
Solutions to Chapter 13 Homework:
Problem 1:

A.

Restaurant
Year Cash flow cumulative cash flow
00 (10,000) (10,000)
01 3000 (7,000)
02 3000 (4,000)
03 3000 (1,000)
04 3000 2,000
05 3000 5,000

The payback period is 3 +1,000/3,000 = 3.33 years

Tourism Van

Year Cash flow cumulative cash flow


0 (25,000) (25,000)
1 7,400 (17,600)
2 7,400 (10,200)
3 7,400 (2,800)
4 7,400 4,600
5 7,400 12,000

The payback period is 3+2800/7,400 = 3.378

B Restaurant

First, you must enter the cash flows for the problem into the calculator

Hit the CF button


CF0 = 10000 +- ↵ ↓
C01 = 3000 ↵ ↓↓
C02 = 3000 ↵ ↓↓
C03 = 3000 ↵ ↓↓
C04 = 3000 ↵ ↓↓
C05 = 3000 ↵ ↓↓

Now hit the IRR button then the CPT button to get the internal rate of return of 15.238 percent

Tourism Van

CF0 = 25000 +- ↵ ↓
C01 = 7400 ↵ ↓↓
C02 = 7400 ↵ ↓↓
C03 = 7400 ↵ ↓↓
C04 = 7400 ↵ ↓↓
C05 = 7400 ↵ ↓↓

IRR = 14.67 percent

430
C. Restaurant

You must enter the cash flows into the calculator in the same manner as you did for part B (if they
are already entered, you do not need to re-enter them)

Now hit the NPV button and input I = 12 and hit the enter button ↵ followed by the arrow down
button ↓, followed by the CPT button to get the net present value of $814.33.

Tourism Van

NPV = $1,675.34

Notice that the various methods give us conflicting results the payback period, the discounted payback
period and the internal rate of return suggest that the restaurant is a better project. The net present value
suggests that the tourism van is a better project. The Internal Rate of Return suggests that the restaurant is
a better project. However we must recognize that the NPV is somewhat distorted in this example because
of the different sizes of the projects. When there is a large difference in the scale of the projects, I
recommend following the suggestion provided by the internal rate of return. So we should start the
restaurant.

Problem 2:

Year Cash flow cumulative cash flow


0 (52,125) (52,125)
1 12,000 (40,125)
2 12,000 (28,125)
3 12,000 (16,125)
4 12,000 (4,125)
5 12,000 7,875
6 12,000
7 12,000
8 12,000

The payback period is 4 + 4125/12000 = 4.34 years

B. IRR = 16.00

C. NPV = 7486.68

431
Problem 3

a. Project A NPV @ 5 percent cost of capital = $ 2512.19


Project B NPV @ 5 percent cost of capital = $ 2,786.37

b. Project A NPV @ 20 percent cost of capital = $ 630.15


Project B NPV @ 20 percent cost of capital = $ 604.59

Notice that project B is a better project when the cost of capital is low. Project A is a better project
when the cost of capital is high.

4. We begin by assuming that we can repeat project Short, four years from today, giving us the following
cash flows

Year Project Short Short Repeated Project Long


0 $(20,000) $(40,000)
1 $7,000 $9,000
2 $10,000 $14,000
3 $19,000 $15,000
4 $8,000 $(20,000) $15,000
5 $0 $7,000 $6,000
6 $0 $10,000 $15,000
7 $0 $19,000 $8,000
8 $0 $8,000 $7,000

Combining Project short with the repeated project short gives us:

Year Project Short/ with Repetition Project Long


0 $(20,000) $(40,000)
1 $7,000 $9,000
2 $10,000 $14,000
3 $19,000 $15,000
4 $(12,000) $15,000
5 $7,000 $6,000
6 $10,000 $15,000
7 $19,000 $8,000
8 $8,000 $7,000

Project Short Project Long


NPV $15,611.29 10,036.26
IRR Multiple Potential Answers 23.79%

Project Short is the better project and should be accepted.

Note that if you have the BAII+ Professional, the IRR on project short will come out to 37.29 percent. If
you have the BAII+, you will get an error.

5. The weighted average cost of capital is given by: WACC = Wst(Kreq) + Wd(Kd)(1-T).

In this case the computations are

WACC = 0.30(10) + 0.70(5)(1-0.4) = 3+ 2.1 = 5.1

432
Chapter 11: Capital Budgeting when
the Cost of Capital is Not Fixed

433
Capital Budgeting when the Cost of Capital is Not Fixed
So far we have assumed that the cost of capital is fixed regardless
of how much money our company uses for its projects.

In many situations, the cost of capital is not fixed but rather


increases as you need more money.

When the Cost of Capital varies, our capital budgeting procedures must be
modified.

Example #1:

Suppose that our company has the following projects that it is considering.

Project Cost to Start the Project IRR of the project


1 $100,000 20 percent
2 $300,000 18 percent
3 $200,000 16 percent
4 $150,000 15 percent

Suppose also that our company can obtain funds as follows: (Marginal Cost
of Capital schedule)

Funding Level Amount of Money Cost of Funds (WACC)


A First $400,000 10 percent
B Additional $200,000 11 percent
C any additional amounts 18 percent

Which Projects Should our Company Accept?

434
Steps in Finding the Solution:

Step 1: Draw a Opportunity Graph


A graph of the projects that are available to your company

Step 2: Draw a Marginal Cost of Capital Graph


A graph of the cost of funds for your company at different levels of
funding

Step 3: Combine the above two graphs into one graph

Step 4: Select projects where the return on the project is higher than the
marginal cost of obtaining funds for the project

435
Step 1: Create an Opportunity Graph
IRR
P1
20
P2
18

P3
16
15 P4

Total
Funds
$100,000 $400,000 $600,000 $750,000 Used

Step 2: Create a Marginal Cost of Capital Graph

WACC

20
F-C

18

16
15

F-B

11 F-A

10
Total
Funds
Raise
$100,000 $400,000 $600,000 $750,000

436
Step 3: Combine the two graphs

IRR &
WACC P1
20
F-C
P2
18

P3
16
15 P4

F-B
11 F-A
10
Total
Funds
Raised
$100,000 $400,000 $600,000 $750,000 & Used

Step 4: Select projects where the return on the project is higher than the
marginal cost of obtaining funds for the project.

In this case we should accept Projects 1, 2 and 3. We should not


accept Project 4.

437
Example #2:

Suppose that our company has the following projects that it is considering.

Project Cost to Start the Project IRR of the project


1 $100,000 20 percent
2 $350,000 18 percent
3 $200,000 10.5 percent
4 $150,000 9.5 percent

Suppose also that our company can obtain funds as follows: (Marginal Cost
of Capital schedule)

Funding Level Amount of Money Cost of Funds (WACC)


A First $500,000 10 percent
B Additional $200,000 11 percent
C any additional amounts 18 percent

Which Projects Should our Company Accept?

We should accept projects 1 and 2. We should not accept project #4. In this
case, it is not clear if we should accept Project #3 or not. How do we
determine if we should accept project 3?

438
Step 1: Create an Opportunity Graph
IRR
P1
20
P2
18

16
15

P3

P4
10

Total
Funds
$100,000 $450,000 $650,000 $800,000 Used

Step 2: Create a Marginal Cost of Capital Graph

WACC

20
F-C

18

16
15

F-B

11 F-A

10
Total
Funds
Raised
$100,000 $500,000 $700,000

439
Step 3: Combine the two graphs

IRR &
WACC P1

20
F-C
P2
18

16
15

F-B
11 F-A
10 P3
Total
P4
Funds
Raised
$100 $450 $500 $650 $700 $800 & Used

Step 4: Select projects where the return on the project is higher than the
marginal cost of obtaining funds for the project.

In this case we should accept Projects 1 and 2. We should not accept


project 4. It is not clear if we should accept Project 3 or not.

440
To determine if we should accept project 3, we must compute the Decision
Criteria as follows:

DC = $LC(IRR – WACCL) + $HC(IRR-WACCH)

Where

DC = Decision Criteria
$LC = The number of dollars to be invested in the project that will be
financed by low cost funds
$HC = The number of dollars to be invested in the project that will be
financed by high cost funds
IRR = the Internal Rate of Return on the project in question
WACCL = the WACC when low cost funds are used
WACCH = the WACC when the high cost funds are used

Decision Rule:
If DC is greater than or equal to 0, you should accept the project.
If DC is less than 0, you should reject the project.

DC = 50,000(.105-.10) + 150,000(.105-.11)

= 250 + (-750)

= -500

As DC < 0, we should not accept Project #3

441
There are seven steps in completing the Marginal Cost of Capital Graph.

#1. Computing the Break Points


#2. Starting a Marginal Cost of Capital Schedule
#3. Computing the Component Costs of Capital
#4. Summarize the Component Costs of Capital
#5. Computing the cost of capital (the WACC) for each level of funding.
#6. Completing the Cost of Capital Schedule
#7. Drawing the Marginal Cost of Capital Graph

Example #3:

Suppose that we have the following projects available to our company:

Project Cost to Start the Project IRR of the project


1 $100,000 20 percent
2 $300,000 18 percent
3 $200,000 16 percent
4 $150,000 15 percent

Suppose that our optimal capital structure is 40 percent debt and 60 percent
equity. Our company wants to be sure to maintain this mix of financing. We
have contacted our bank and they have told us that we can borrow $100,000
at an interest rate of 10 percent. We can borrow any amount of additional
money at an interest rate of 11 percent. Our company has $10,000 of
retained earnings. The current price of the company’s stock is $10 per share.
Next years dividend is expected to be $0.75 per share. This dividend is
expected to grow at a 10 percent rate forever. If we issue new common
stock, the float cost will be $2.00 per share. We can issue an unlimited
amount of new common stock at this price. The firms tax rate is 30 percent

Which Projects should our company accept:

442
Step 1: Computing the Break Points

A break point is the total amount of capital raised when a given source of
financing runs out.

We are answering the question:

Debt is 40 percent $100,000


Equity is 60 percent ????????
Total Capital ????????

BPdebt = (amount of debt available at this cost)/percentage of funds we will


obtain from debt

Bpequit = (amount of equity available at this cost)/percentage of funds we


will obtain from equity

443
Computing Break Points (Note in this example there are two break points,
other examples may have more or less than two break points)

Recall that there are two sources of debt for our company.

Source #1. We can borrow $100,000 at a 10 percent interest rate.


Source #2: We can borrow additional funds at an 11 percent interest
rate.

Also recall that we will obtain 40 percent of our money from debt.

There are two sources of debt for our company, so we must compute a break
point for debt.

BPdebt = $100,000/.40 = $250,000

The Break point can be interpreted as follows:

If we obtain 40 percent of our money from debt and 60 percent of our money
from equity. We will have a total of $250,000 when we must begin to use
the expensive debt.
Alternative Interpretation: If we need more than $250,000 for our projects,
we will need to borrow some money at 11 percent interest.

444
We must also compute a break point for equity. There are two sources of
equity for our company.

Source #1: Retained Earnings ($10,000 available)


Retained earnings is money our company has earned but has
not yet given to the owners of the company.

Source #2 New Common Stock Issue


There is a limit to the amount of retained earnings that a
company has. If you need more equity than is available from
retained earnings you must have a new equity issue.

Recall that our company wants to obtain 60 percent of its funds from equity

Bpequity = $10,000/.6 = $16,666.67

Interpretation: If we obtain 40 percent of our money from debt and 60


percent of our money from equity, we will have a total of $16,666.67 before
we must start issuing new common stock.

Alternative interpretation: If we want to accept project a total of more than


$16,666.67 we will have to issue new stock.

445
Step 2: Starting a Marginal Cost of Capital Schedule:
(note: always start from the lowest cost sources of funds and go to the higher
cost sources of funds)

(note: there will always be one more level of funding than there was Break
Points)

Total Funding Level: Source of Equity Source of Debt


0-$16,666.67 Retained Earnings 10 percent int
$16,666.67 – $250,000 New Common Stock 10 percent int
above $250,000 New Common Stock 11 percent int

446
Step 3: Computing the Component Costs of Capital

Part #1. For Debt – the component cost of debt will be given to you
as the interest rate the bank will charge you on a loan.

Part #2. Component Cost of Equity

A. The cost of Retained Earnings

For our purposes, we will always assume that dividends


will grow at some constant rate forever. As such, we can
use the constant growth model.

The constant Growth Model Says that:

Po = D1/(kreq-g)

We want to solve the above equation for kreq

Kreq = (D1/Po) + g

Kreq = ($0.75/$10) + .10

= 0.175 or 17.5 percent

B. The Cost of New Common Stock Issues

The cost of New Common Stock is higher than the cost of


Retained Earnings. When we issue new common stock, we
must pay the cost of finding the new investors. That cost of
finding new investors is called a Float Cost.

Kreq = (D1/(Po-F)) + g

Kreq = ($0.75/($10-$2) + .10

= 0.19375 or 19.375 percent

447
Step 4: Summary of Component Costs of Capital
This is a summary of the information computed in Step 3

Borrowing (up to $100,000) 10 percent


Borrowing (above $100,000) 11 percent
Cost of Retained Earnings ($10,000 available) 17.5 percent
Cost of New Common Stock Issues 19.375 percent

448
Step 5: Computing the WACC

You must compute a WACC for each level of funding that you identified in
your Cost of Capital Schedule. So for our example we must compute the
WACC 3 times.

The WACC can be computed as:

WACC = Ws(Kreq) + Wd(Kd)(1-T)

Where:

Ws = the percentage of our funds that we will get from equity


Wd = the percentage of our funds that we will get from debt
Kreq = the return investors require for investing in our stock
Kd = the before tax cost of borrowing money
T = the firms tax rate

You need to compute a WACC for each level of funding that you identified
in your Cost of Capital Schedule.

449
A. When we need from 0 to $16,666.67, we will obtain our funds from
retained earnings and the low cost debt.

The cost of retained earnings was 17.5 percent


The low interest debt cost 10 percent

WACC = 0.6(17.5) + 0.4(10)(1-.3)

= 10.5 + 2.8

= 13.3 percent

B. When we need from $16,666.67 to $250,000 we will obtain our funds


from New Common Stock and from low cost debt.

The cost of new common stock is 19.375 percent


The low interest debt cost 10 percent

WACC = 0.6(19.375) + 0.4(10)(1-.3)

= 11.625 + 2.8

= 14.425 percent

C. When we need more than $250,000 we will obtain funds from New
Common Stock and High Cost Debt

The cost of New Common Stock is 19.375


The high interest debt cost 11 percent

WACC = 0.6(19.375) + 0.4(11)(1-.3)

= 11.625 + 3.08

= 14.705

450
Step 6: Complete the Marginal Cost of Capital Schedule

Total Funding Level: Source of Equity Source of Debt WACC


0-$16,666.67 Retained Earnings 10 percent int 13.3
$16,666.67 – $250,000 New Common Stock 10 percent int 14.425
above $250,000 New Common Stock 11 percent int 14.705

Step 7: Create a Marginal Cost of Capital Graph


WACC

20

18

16 F-C
15 F-B

11 F-A

10
Total
Funds
Raised
$16.7 $250

451
Create an Opportunity Graph
IRR
P1
20
P2
18

P3
16
15 P4

Total
Funds
Used
$100,000 $400,000 $600,000 $750,000

Combine the two graphs

IRR & WACC


P1
20
P2
18

P3
16
P4

15
F-B

F-C

11
F-A
Total
10 Funds
Raised
16.7 100 250 400 600 750 & Used

In this case we should accept Projects 1, 2, 3 and 4.

452
Chapter 11 Homework

453
Homework For Chapter 11

Problem 1:

You have the following projects available for you company.

Suppose that our company has the following projects that it is considering.

Project Cost to Start the Project IRR of the project


1 $500,000 25 percent
2 $500,000 14 percent
3 $400,000 10 percent

Suppose also that our company can obtain funds for our company (Marginal
Cost of Capital schedule)

Funding Level Amount of Money Cost of Funds (WACC)


A First $400,000 10 percent
B Additional $300,000 11 percent
C Additional $300,000 13 percent
D any additional amounts 18 percent

Which projects should our company accept?

454
Problem #2

Suppose that our company has the following projects that it is considering.

Project 1
Year CF
0 -$100,000
1 $50,000
2 $95,000

Project 2
Year CF
0 -$150,000
1 $75,000
2 $120,000
3 $90,000

Project 3
Year CF
0 -$300,000
1 $100,000
2 $500,000

Project 4:
Year CF
0 $-200,000
1 $100,000
2 $200,000
3 $300,000

Suppose also that our company can obtain funds for our company (Marginal
Cost of Capital schedule)

Funding Level Amount of Money Cost of Funds (WACC)


A First $400,000 10 percent
B Additional $100,000 11 percent
C Additional $150,000 40 percent
D any additional amounts 45 percent

455
Which projects should our company accept?
Problem #3:

Suppose that we have the following projects available to our company:

Project Cost to Start the Project IRR of the project


1 $300,000 20 percent
2 $500,000 18 percent
3 $400,000 16 percent
4 $200,000 10 percent

Suppose that our optimal capital structure is 30 percent debt and 70 percent
equity. Our company wants to be sure to maintain this mix of financing. We
have contacted our bank and they have told us that we can borrow $200,000
at an interest rate of 10 percent. We can borrow any amount of additional
money at an interest rate of 11 percent. Our company has $100,000 of
retained earnings. The current price of the companies stock is $15 per share.
Next years dividend is expected to be $1.10. This dividend is expected to
grow at a 11 percent rate forever. If we issue new common stock, the float
cost will be $2.00 per share. We can issue an unlimited amount of new
common stock at this price. The firms tax rate is 40 percent. Which projects
should the firm undertake

456
Problem #4

Suppose that we have the following projects available to our company:

Project Cost to Start the Project IRR of the project


1 $500,000 30 percent
2 $700,000 25 percent
3 $600,000 20 percent
4 $800,000 15 percent

Suppose that our optimal capital structure is 20 percent debt and 80 percent
equity. Our company wants to be sure to maintain this mix of financing. We
have contacted our bank and they have told us that we can borrow $200,000
at an interest rate of 10 percent. We can borrow an additional $300,000 at
an 11 percent interest rate. We can borrow additional money at an interest
rate of 12 percent. Our company has $100,000 of retained earnings. The
current price of the companies stock is $15 per share. Next years dividend is
expected to be $1.10. This dividend is expected to grow at a 11 percent rate
forever. If we issue $150,000 of new common stock, the float cost will be
$2.00 per share. If we issue more than $150,000 of new common stock, the
float cost will be $3.00 per share We can issue an unlimited amount of new
common stock with the $3.00 float. The firms tax rate is 40 percent. Which
projects should the firm undertake?

457
Problem #5

Suppose that our company has the following projects that it is considering.

Project Cost to Start the Project IRR of the project


1 $100,000 20 percent
2 $350,000 18 percent
3 $200,000 10.5 percent
4 $150,000 9.5 percent

Suppose also that our company can obtain funds for our company (Marginal
Cost of Capital schedule)

Funding Level Amount of Money Cost of Funds (WACC)


A First $300,000 10 percent
B Additional $300,000 19 percent
C any additional amounts 20 percent

Which Projects Should our Company Accept?

458
Solution to Chapter 11 Homework

Problem #1 Opportunity Graph


IRR
P1
25

18

16 P2

14

P3

10
Total
Funds
Used
500 1,000 1,400

Problem 1: Marginal Cost of Capital Graph


WACC

25 F-D

18

16

15
F-C

F-B

11 F-A

10

Total
Funds
400 700 1,000 Raised

459
Problem 1: Combined graphs

IRR & WACC P1


25
F-D

18

P2
16
15

F-B
F-C P3
11 F-A
10
Total
Funds
Raised
400 500 700 1,000 1,400 & Used

In this case we should accept Projects 1 and 2. We should not accept


Project 3.

460
Problem #2

In order to solve this problem, we must first compute the internal rate of
return for each of the projects

For Project 1

CF0 = $100,000 +- ↵ ↓
C01 = $50,000 ↵ ↓↓
C02 = $95,000 ↵ ↓↓

Now press the IRR button followed by the compute button on your
calculator

You should get an answer of 25.62 percent

Project 2: IRR = 38.79 percent


Project 3: IRR = 46.84 percent
Project 4: IRR = 65.35 percent

Thus we have the following

Project Rank Project # Cost to Start the Project IRR


1 4 $200,000 65.35
2 3 $300,000 46.84
3 2 $150,000 38.79
4 1 $100,000 25.62

Now Draw a Combined Graph to determine which projects should be


accepted.

461
Problem 2 Combined Graph
IRR & WACC
P4

65

P3

46 F-C

F-D
38
P2
25
F-B P1
11 F-A
10
Total
Funds
Raised &
200 400 500 650 750 Used

In this case we should accept Projects 3 and 4. We should not accept


Project 1 or 2.

462
Problem 3 Solution

Step 1: Computing the Break Points

There are two sources of debt for our company.


Source #1. We can borrow $200,000 at a 10 percent interest rate.
Source #2: We can borrow additional funds at an 11 percent interest
rate.

Also recall that we will obtain 30 percent of our money from debt.

There are two sources of debt for our company, so we must compute a break
point for debt.

BPdebt = $200,000/.30 = $666,666.67

We must also compute a break point for equity. There are two sources of
equity for our company.

Source #1: Retained Earnings ($100,000 available)

Source #2 New Common Stock Issue

Recall that our company wants to obtain 70 percent of its funds from equity

Bpequity = $100,000/.7 = $142,857.14

463
Step 2: Starting a Marginal Cost of Capital Schedule:
(note: always start from the lowest cost sources of funds and go to the higher
cost sources of funds)

(note: there will always be one more level of funding than there was Break
Points)

Total Funding Level: Source of Equity Source of Debt


0-$142,857.14 Retained Earnings 10 percent int
$142,857.14-$666,666.67 New Common Stock 10 percent int
above $666,666.67 New Common Stock 11 percent int

Step 3: Computing the Component Costs of Capital

Part #1. For Debt – already computed

Part #2. Component Cost of Equity

A. The Cost of Retained Earnings

Kreq = (D1/Po) + g

Kreq = ($1.10/$15) + .11

= 0.1833 or 18.33 percent

B. The Cost of New Common Stock Issues

The cost of New Common Stock is higher than the cost of


Retained Earnings. When we issue new common stock, we
must pay the cost of finding the new investors. That cost of
finding new investors is called a Float Cost.

Kreq = (D1/(Po-F)) + g

Kreq = ($1.1/($15-$2) + .11

464
= 0.1946 or 19.46 percent
Step 4: Summary of Component Costs of Capital
This is a summary of the information computed in Step 3

Borowing (up to $200,000) 10 percent


Borrowing (above $200,000) 11 percent
Cost of Retained Earnings ($100,000 available) 18.33 percent
Cost of New Common Stock Issues 19.46 percent

Step 5: Computing the WACC

You must compute a WACC for each level of funding. For this problem we
must compute the WACC 3 times. (Because there are two break points).

A. When we need from 0 to $142,857.14, we will obtain our


funds from retained earnings and the low cost debt.

The cost of retained earnings was 18.33 percent


The low interest debt cost 10 percent

WACC = 0.7(18.33) + 0.3(10)(1-.4)

= 12.831 + 1.8

= 14.631 percent

B. When we need from $142,857.14 to $666,666.67 we will


obtain our funds from New Common Stock and from low
cost debt.

The cost of new common stock is 19.46 percent


The low interest debt cost 10 percent

WACC = 0.7(19.46) + 0.3(10)(1-.4)

= 13.622 + 1.8

= 15.422 percent

465
C. When we need more than $666,666.67 we will obtain funds
from New Common Stock and High Cost Debt

The cost of New Common Stock is 19.46


The high interest debt cost 11 percent

WACC = 0.7(19.46) + 0.3(11)(1-.4)

= 13.622 + 1.98
= 15.602

Step 6: Complete the Marginal Cost of Capital Schedule

Total Funding Level: Source of Equity Source of Debt WACC


0-$142,857.14 Retained Earnings 10 percent int 14.631
$142,857.14-$666,667 New Common Stock 10 percent int 15.422
above $666,667 New Common Stock 11 percent int 15.602

Problem 3 Combined Graph


IRR & WACC
P1

20

P2
18

16 P3

15 F-C
F-B Total
14 P4 Funds
F-A Raised
& Used
142 300 667 800 1,200 1,400

In this case we should accept Projects 1, 2 and 3. We should not accept


Project 4.

466
Problem 4 Solution

Step 1: Computing the Break Points

There are three sources of debt for our company.


Source #1. We can borrow $200,000 at a 10 percent interest rate.
Source #2: We can borrow an additional $300,000 at an 11 percent
interest rate.
Source #3 We can borrow additional amounts at an interest rate of
12 percent

We will obtain 20 percent of our money from debt.

There are three sources of debt for our company, so we must compute two
break point for debt.

BPdebt = $200,000/.20 = $1,000,000


Bpdebt = ($200,000 + $300,000)/.20 = $2,500,000

Notice that when there are multiple break points from one source of
financing, we must include both amounts in computing the second break
point.

We must also compute two break points for equity. There are three sources
of equity for our company.

Source #1: Retained Earnings ($100,000 available)

Source #2 New Common Stock Issue with a $2.00 float cost

Source #3 New Common Stock Issue with $3.00 float cost

Recall that our company wants to obtain 80 percent of its funds from equity.

Break point when Retained Earnings runs out

Bpequity = $100,000/0.8 = $125,000

467
Break Point when New Common Stock with a $2.00 float cost runs
out.

Bpequity = ($100,000 +$150,000)/0.8 = $312,500

Step 2: Starting a Marginal Cost of Capital Schedule:


(note: always start from the lowest cost sources of funds and go to the higher
cost sources of funds)

(note: there will always be one more level of funding than there was Break
Points)

Total Funding Level: Source of Equity Source of Debt


0-$125,000 Retained Earnings 10 percent int
$125,000-$312,500 New Common Stock($2) 10 percent int
$312,500-$1,000,000 New Common Stock($3) 10 percent int
$1,000,000-$2,500,000 New Common Stock($3) 11 percent int
above $2,500,000 New Common Stock($3) 12 percent int

Step 3: Computing the Component Costs of Capital

Part #1. For Debt – already computed

Part #2. Component Cost of Equity

D. The Cost of Retained Earnings

Kreq = (D1/Po) + g

Kreq = ($1.10/$15) + .11

= 0.1833 or 18.33 percent

468
C. The Cost of New Common Stock Issues

i. When the float cost is $2

Kreq = (D1/(Po-F)) + g

Kreq = ($1.1/($15-$2) + .11

= 0.1946 or 19.46 percent

ii. When the float cost is $3

Kreq = (D1/(Po-F)) + g

Kreq = ($1.1/($15-$3) + .11

= 0.2017 or 20.17 percent

Step 4: Summary of Component Costs of Capital


This is a summary of the information computed in Step 3

Borowing (up to $200,000) 10 percent


Borrowing ($200,000-$500,000) 11 percent
Borrowing (above $500,000) 12 percent
Cost of Retained Earnings ($100,000 available) 18.33 percent
Cost of New Common Stock Issues ($2 Float) 19.46 percent
Cost of New Common Stock Issues ($3 Float) 20.17 percent

469
Step 5: Computing the WACC

You must compute a WACC for each level of funding. For this problem we
must compute the WACC 5 times.

A. When we need from 0 to $125,000 we will obtain our


funds from retained earnings and the low cost debt.

The cost of retained earnings was 18.33 percent


The low interest debt cost 10 percent

WACC = 0.8(18.33) + 0.2(10)(1-.4)

= 14.66 + 1.2

= 15.86 percent

B. When we need from $125,000 to $312,500 we will obtain


our funds from New Common Stock ($2 float) and from low
cost debt.

The cost of new common stock is 19.46 percent


The low interest debt cost 10 percent

WACC = 0.8(19.46) + 0.2(10)(1-.4)

= 15.57 + 1.2
= 16.77 percent

C. When we need from $312,500 to $1,000,000 we will obtain


funds from New Common Stock ($3.00 Float ) and Low
Cost Debt

The cost of New Common Stock is 20.17


The low interest debt cost 10 percent

WACC = 0.8(20.17) + 0.2(10)(1-.4)

470
= 16.14 + 1.2
= 17.34 percent

D. When we need from $1,000,000 to $2,500,000 we will


obtain funds from New Common Stock ($3.00 Float ) and
Medium Cost Debt

The cost of New Common Stock is 20.17


The medium interest debt cost 11 percent

WACC = 0.8(20.17) + 0.2(11)(1-.4)

= 16.14 + 1.32
= 17.46 percent

E. When we need more than $2,500,000 we will obtain funds


from New Common Stock ($3.00 Float ) and High Cost
Debt

The cost of New Common Stock is 20.17


The medium interest debt cost 11 percent

WACC = .8(20.17) + .2(12)(1-.4)

= 16.14 + 1.44
= 17.58 percent

Step 6: Complete the Marginal Cost of Capital Schedule

Total Funding Level: Source of Equity Source of Debt WACC


0-$125,000 Retained Earnings 10 percent int 15.86
$125,000-$312,500 New Common Stock($2) 10 percent int 16.77
$312,500-$1,000,000 New Common Stock($3) 10 percent int 17.34
$1,000,000-$2,500,000 New Common Stock($3) 11 percent int 17.46
above $2,500,000 New Common Stock($3) 12 percent int 17.58

471
Problem 4 Combined Graph
IRR & WACC
P1

30
P2

25
P3 F-E

20
F-C
F-B
F-D P4

15

F-A
Total
Funds
Raised
& Used
125 500 1,000 1,200 1,800 2,500 2,600

In this case we should accept Projects 1, 2 and 3. We should not accept


Project 4.

472
Problem 5 Solution

Problem 5: Combined Graphs

IRR & WACC


P1
F-B F-C
20
P2
18

16
15

P3
11 F-A P4
10
Total
Funds
Raised
$100 $450 $600 $650 $800 & Used

We should accept projects 1. We should not accept projects #3 or #4. In


this case, it is not clear if we should accept Project #2 or not. To determine
if we should accept project #2, we must compute the decision criteria.

Funding for project #2 will be as follows


The first $200,000 of the project can be financed with the funds that cost 10
percent. The remaining $150,000 of the project can be financed with the
funds that cost 19 percent.

DC = $200,000(18 – 10) + $150,000(18-19)

= 1,600,000 + (-150,000)

= $1,450,000

As DC > 0, we should accept Project #2

473

You might also like